Anda di halaman 1dari 160

Náboj 2011 Problems

Problem 1 J. The numerator and denominator of Juan’s fraction are positive integers whose
sum is 2011. The value of this fraction is less than 13 . Find the greatest such fraction.

502
Answer. 1509
.

Solution outline. We clearly have a < 2001, so the relation can be rewritten as

a 1
< .
2011 − a 3

This rewrites as 4a < 2011 , from which we get that the biggest value of a is 502, so the smallest
502
value of b is 1509, giving us the answer 1509 .

Problem 2 J. Rectangle ABCD intersects a circle at points E, F , G, H as shown in the diagram.


If AE = 3, DH = 4, and GH = 5, find EF .

D
H
A
G C
E

F B

Answer. 7.

Solution outline.

D 4 H 5 G C

3 1 1
A E H1 G1 F B

Let G1 , H1 be the projections of G and H on the line AB. Then we obtain H1 G1 = HG = 5. We


also have EH1 = G1 F and EH1 = DH − AE = 1, thus EF = EH1 + H1 G1 + G1 F = 7.
2

Problem 3 J. Find the sum of digits of the number 1 + 11 + 101 + 1001 + 10001 + · · · + 10 . . . 01.
| {z }
50

Answer. 58.

Solution outline. Our number is equal to

1 + 11 + 101 + 1001 + · · · + 10 . . . 01 =
| {z }
50
= 1 + (10 + 1) + (100 + 1) + (1000 + 1) + · · · + (10 . . . 0 + 1) = 11 . . . 110 + 52 = 11 . . . 1162.
| {z } | {z } | {z }
51 51 50

From this we get that the sum of digits of our number is 58.

Problem 4 J. T-shirts were made in three colors, red, gray and blue. The number of red T-shirts
is six smaller than the total number blue and gray T-shirts. Also, the number of gray T-shirts is
ten smaller than the total number red and blue T-shirts. How many blue T-shirts were made?

Answer. 8.

Solution outline. Let a, b, c be the number of red, white and respectively blue T-shirts. We then
obtain the system of equations
a = b + c − 6,
b = a + c − 10.

By adding the two relations we obtain c = 8.

Problem 5 J. There are 33 walnuts on the table in at least two piles. Each pile consists of at
least 2 walnuts. After moving one walnut from each pile to the first one, all piles will have the same
number of walnuts. What was the original number of piles? Find all possibilities.

Answer. 3.

Solution outline. Let p1 , p2 , . . . pn+1 be all the piles. Then p2 = p3 = · · · = pn+1 = b, and let
p1 = a. We then obtain a + nb = 33 and a + n = b − 1, and by subtracting these two relations
we get bn − n = 34 − b, so (b − 1)(n + 1) = 33. If n + 1 = 11, then b = 4, and we cannot have
a + n = b − 1, so we are only left with n + 1 = 3, which works.

Problem 6 J. A rectangle is divided by two segments parallel to its sides into four smaller
rectangles. Let’s label them A, B, C, D as in the diagram. Given that the perimeters of rectangles
A, B, C are 2, 4, and 7, respectively, find all possible values of the perimeter of rectangle D.

A B

C D

Answer. 9 cm.

Solution outline.
3

A x B

t y

C z D

Let x, y, z, t be the lengths of the sides of the four rectangles, as in the figure, and let oA , oB , oC ,
oD be the perimeters of the rectangles A, B, C, D respectively. Then

oA + oD = 2(t + x) + 2(y + z) = 2(x + y) + 2(t + z) = oB + oC ,

therefore oD = 4 + 7 − 2 = 9 cm.

Problem 7 J. Find pairwise distinct digits A, B, C such that


A
AB
ABC
BCB
Answer. A = 6, B = 7, C = 4.
Solution outline. Let’s write, for conveniance, a = A, b = B, c = C. Then we get a + 10b + c +
100a + 10b + c = 100b + 10c + b, so 111a = 90b + 9c. This means that 9 divides 111a, so 3 divides a.
If a = 3, then b = 3 and c = 7, so a, b, c are not pairwise distinct.
If a = 6, then b = 7 and c = 4.
If a = 9, then we obtain 111 = 10b + c, which doesn’t satisfy the property that b and c are
digits. So the only solution is a = 6, b = 7, c = 4.

Problem 8 J. Find the area of a rectangle with perimeter 10 cm and diagonal 15 cm.
Answer. 5 cm2 .
Solution outline. Let a and b be the sides of our rectangle. Then we obtain 2(a+b) = 10, a2 +b2 =
15, so 
2ab = (a + b)2 − a2 + b2 = 52 − 15 = 10.
Therefore the area is 5 cm2 .

Problem 9 J. Andrei took N 3 equally sized white cubes and used them to form one big cube
N × N × N . Then he colored its surface in red. Given that now one tenth of the total surface of
the cubes is red, find N .
Answer. 10.
Solution outline. Let S be the area of one face of a small cube. Then the sum of the surface areas
of all the cubes is 6N 3 S, and the surface area of the big cube is 6N 2 S. The problem statement
tells us that 6S · N 2 = 6S
10
N 3 , so N = 10.

Problem 10 J. What is the least possible number of members of a math circle, where girls form
more than 48.5% but less than 50% of the members?
Answer. 35.
Solution outline. Let g be the number of girls in the math circle, and let N be the total number
of people in the circle. Then the problem statement tells us that 48,5
100
N <g< N2
, so we can write
4

N = 2g + a, with a an integer, a > 0, and 97N = 200g − b, for an integer b with b > 0. The last
equation can be rewritten as 97(2g + a) = 200g − b, or equivalently 97a + b = 6g. If a = 1, then
the smallest value of b which satisfies our equation is b = 5, which gives us g = 17 and N = 35.
If a ≥ 2, then 6g > 194, so g ≥ 33, which means N ≥ 67. Therefore a = 1 gives us the least
value of N , which is indeed 35.

Problem 11 J / 1 S. If you increase the number of this problem by n, you obtain the number
of the most shocking problem. Whereas if you increase it by a two-digit number k, you obtain the
number of the most playful problem. Moreover, we have n3 = k2 . Find n and k, given that you
have 30 problems left (including this one).

Answer. n = 9, k = 27.

Solution outline. For k2 to be a cube, k must be a cube itself. The only two-digit cube less than
41 is 27, which gives us n = 9.

Problem 12 J / 2 S. Find positive integer n such that 66662 + 88882 = n2 .

Answer. 11110.

Solution outline. We regroup the terms to get


p √
n= 11112 · 62 + 11112 · 82 = 1111 36 + 64 = 11110.

Problem 13 J / 3 S. Find the smallest positive integer, which ends with number 17, which is
divisible by 17, and whose sum of digits is 17.

Answer. 15317.

Solution outline. We write the number in the form 100·a+17 for some a ∈ N0 . Since 17 is coprime
with 100, we get that a is divisible by 17 and has sum of digits 9. Thus it is also a multiple of 9.
Smallest such a is 153, thus the solution is 15317.

Problem 14 J / 4 S. Each pair of consecutive digits of a 2011-digit number is a multiple of


either 17 or 23. Its last digit is 1. Find its first digit.

Answer. 3.

Solution outline. Write all two-digit multiples of 17 and 23 to find out that each unit digit appears
exactly once. In this way we can restore the original number from its end as . . . 92346 92346 851.
Noticing the periodicity, we easily calculate that the first digit is three.

Problem 15 J / 5 S. A positive integer is called awesome if any other positive integer with the
same sum of digits is greater. How many three-digit awesome numbers exist?

Answer. 9.

Solution outline. Realize that for each value k ∈ N of the sum of the digits there is exactly one
smallest positive integer with this sum of its digits, i.e. one awesome number. Let’s denote it ak .
The number a1 . . . a18 will be two digit integers, whereas a28 , a29 , . . . will have more than three
digits. Also the sums of digits of three-digit integers attain all numbers between 19 and 27, thus
a19 , . . . , a27 will indeed have three digits and thus we have 9 awesome three-digit numbers.
5

x−y
Problem 16 J / 6 S. Tim has found real numbers x, y, z satisfying z−y
= −10. What are the
x−z
possible values of y−z
?
Answer. 11.
Solution outline.
x−z (x − y) + (y − z) x−y y−z
= = + = 10 + 1 = 11.
y−z y−z y−z y−z

Problem 17 J / 7 S. The numbers 1, 2, . . . , 9 are arranged in some order to form a nine-digit


integer. Consider all triplets of consecutive digits and add the corresponding seven three-digit
numbers. What is the largest result that can be obtained?
Answer. 4648.
Solution outline. Let the digits be a1 , a2 , . . . , a9 . The triplet then equal:
100a1 + 10a2 + a3 , 100a2 + 10a3 + a4 , 100a3 + 10a4 + a5 , ... , 100a7 + 10a8 + a9 ,
so their sum equals
100a1 + 110a2 + 111a3 + · · · + 111a7 + 11a8 + 1a9 .
To maximize the sum we set a3 to a7 the highest digits and a2 = 4, a1 = 3, a8 = 2, a9 = 1. The
result is
111 · (5 + · · · + 9) + 4 · 110 + 3 · 100 + 2 · 11 + 1 · 1 = 4648.

Problem 18 J / 8 S. A real number is written in each cell of a 10 × 10 square. Emily wrote down
all products of two numbers from two distinct cells of the table and noticed that exactly 1000 of
these products were negative. How many times did number 0 appear in the original square? Find
all possibilities.
Answer. 30, 35.
Solution outline. Let p, n be the numbers of positive and negative numbers written in the square,
respectively. We observe that we must have p + n ≤ 100 and p · n = 1000, which has two solutions
20, 50 and 25, 40, which lead to the number of zeros equal to 30 or 35.

Problem 19 J / 9 S. Math kingdom started to produce a new set of coins. On the first day
they created coins with value 1 MD (Math Dollar). Every other day they created coins with the
smallest value which cannot be paid by at most ten existing coins. Which coins did they create on
the 2011th day?
Answer. 20101.
Solution outline. Prove by induction that during the k-th day they created coins with value
10(k − 1) + 1. The result follow by plugging in k = 2011.

Problem 20 J / 10 S. Let the number p be the solution to this problem. Find the probability
that a randomly chosen point inside a unit square is at least p units away from all sides.
1
Answer. 4
.
Solution outline. A point is not closer than p to any of its sides if and only if it lies inside a small
square with sidelength 1 − 2p. We calculate the probability as the ratio of areas
(1 − 2p)2
= p.
12
6

1
This equation has two solutions: p = 4
works, p = 1 does not.

Problem 21 J / 11 S. A 3 × 3 square is filled with integers such that the sums of the horizontal
rows increase by two going downwards and the sums of the vertical columns double from left to
right. Given that the sum of the numbers in one of the rows is 2011, find the sum of the numbers
in the leftmost column.
Answer. 861.
Solution outline. Let a be the sum of the numbers in the first row and let b be the sum of the
numbers in the leftmost column. We calculate the sum of the numbers in the square in two ways
to get 3a + 6 = 7b. Distuingishing the three cases based on which row has the sum of the numbers
2011, we get the only integer solution b = 861.

Problem 22 J / 12 S. There are 2 boats, one on each side of a river bank. They both sail
towards each other at a constant speed (not necessarily the same). The first time they meet, they
are 100 meters from one side of the bank. Once they reach the side of the bank, they turn around
and move towards each other again. This time they meet 70 meters from the other side of the bank.
How wide is the river?
Answer. 230 m.
Solution outline. Let S be the length of the river bank. When the boats first met, they had
travelled S meters altogether. When they met the second time, they had travelled 3S meters. For
the boat which had travelled 100 meters before the first meeting, we can use the constant speeds
to form the equation 3 · 100 = S + 70, which yields S = 230.

Problem 23 J / 13 S. Vertices of a star form a regular heptagon. What is the magnitude of the
marked angle?

3π 540◦ 1◦
Answer. 7
= 7
= 77◦ + 7
.
Solution outline.
B

A S

D
Denote points A, B, C, D, S as in the diagram. If we rotate the segment AC counterclockwise
about the center by angle 2π · 72 , we obtain the segment DB. Thus ∠DSA = 4π7
and ASB is
supplementary.
7

2
32 x
Problem 24 J / 14 S. Find x such that 22 = 44 .
2
Note: the order of operations is: 43 = 49 .
Answer. 40.
Solution outline. Rewrite the righthand side:
 22 x
44 = 22 ( ) = 22·2 = 22
x 2x 2x+1
.

2
So we need 2x + 1 = 32 = 34 = 81, or in other words x = 40.

Problem 25 J / 15 S. How many triplets of positive integers (a, b, c) exist, such that
a a
c
+ b
+1
b b
= 11
a
+ c
+1

and a + b + c ≤ 30?
Answer. 24.
a b
Solution outline. Write 1 = a
and 1 = b
to get

a a 1 1 1

c
+ b
+1 a c
+ b
+ a a
b b
= 1 1 1
 = = 11.
a
+ c
+1 b a
+ c
+ b
b

For b = 1, a = 11 we get c = 1, 2, . . . , 18 and for b = 2, a = 22 we get c = 1, 2, . . . , 6, which is 24


triplets altogether.

Problem 26 J / 16 S. A circle ω with radius 1, center O and diameter AC is given in the plane.
Draw a line ℓ through point O such that it is perpendicular to AC. Choose a point U on ℓ such
that U is outside ω. Denote by B the second intersection of AU and ω and assume BU = 1. Find
OU .

Answer. 3.
Solution outline.
U

A 1 O C

Since OB = 1 = BU , the triangle OU B is isosceles. Thus B lies on the perpendicular bisector


of AOU and also on its hypotenuse,
√ so it is the midpoint of the hypotenuse. Thus AB = 1 and
Pythagorean theorem gives OU = 3.

Problem 27 J / 17 S. Two nations A and B are in a battle with 1000 soldiers involved altoge-
ther. The armies take turns to attack. In each turn every living soldier from the attacking army
shoots a soldier from the enemy’s army. The battle ended (not necessarily by elimination of one of
8

the sides) after three turns (A was shooting first, then B and finally A again). What is the least
guaranteed number of survivors?
Answer. 200.
Solution outline. Assume n soldiers survived and deduce that there must have been at most 5n
soldiers at the beginning of the battle. Also note that 200 survivors are indeed possible.

Problem 28 J / 18 S. All six sides of a convex hexagon A1 A2 A3 A4 A5 A6 are colored in red.


Each of the diagonals is either blue or red. Find the number of such colorings that each triangle
Ai Aj Ak (i 6= j 6= k 6= i) has at least one red side.
Answer. 392 = 7 · 7 · 8.
Solution outline. Apart from A1 A3 A5 and A2 A4 A6 (dashed in the diagram) all other triangles
have a red edge. Every dashed triangle can be colored in 23 − 1 = 7 ways. Finally, we have 23 = 8
ways to color the dotted diagonals A1 A4 , A2 A5 , A3 A6 . Altogether, this gives us 7 · 7 · 8 = 392
suitable colorings.
A5 A4

A6 A3

A1 A2

Problem 29 J / 19 S. Malcom told each Michal and Shri a positive integer. Further, he told
them, that the numbers they heard were distinct and that their sum is a two-digit number. Then
the following conversation took place:
Michal: “I cannot determine which one of us has the greater number.”
Shri: “I can’t determine it either, but I will tell you that my number is divisible by 17.”
Michal: “Wow! Now, I can determine the sum of our numbers.”
Find the value of this sum, given that the logic Michal and Shri used was flawless.
Answer. 51.
Solution outline. Since neither of them can determine whose number is greater, neither has a
number greater than fifty. Moreover, since Shri’s number is divisible by 17 it can be either 17 or
34. For Michal to be able to determine the sum of the two numbers, he must have the other one
(the numbers are distinct!). Hence the sum is 51.

Problem 30 J / 20 S. There are 55 guests in a café, Turks and Indians. Each of them drinks
either tea or coffee. An Indian speaks truth if he drinks tea and lies if he drinks coffee, whereas
with the Turks it is the other way round. For questions “Do you drink coffee?”, “Are you Turkish?”
and “Is it raining outside?” the numbers of positive answers were 44, 33, and 22, respectively. How
many Indians drink tea? Find all possibilities.
Answer. 0.
Solution outline. Form equations and find two solutions (based on whether it is raining outside
or not) one of which does not work since there would be 11
2
Indians drinking tea, which is against
humanity. The other solution works and yields 0 Indians drinking tea.
9

Problem 31 J / 21 S. Three digits were written to the end of a positive integer A. The resulting
number was equal to the sum of numbers from 1 to A. Find all possible values of A.

Answer. 1999.

Solution outline. Let B be the three digit-integer written to the end of A. Since 1 + 2 + · · · + A =
1
2
A(A + 1), we have
A(A + 1)
1000A + B = ,
2
which we transform to 2B = A(A − 1999). The left handside is between 0 and 2 · 999, whereas the
righthand side is negative for A ≤ 1998 and greater than 2000 · 1 for A ≥ 2000. We are left with
A = 1999 which satisfies the problem for B = 000.

Problem 32 J / 22 S. Alice, Betty, Claudia, Daniel, and Eli were playing doubles tournament
in table-tennis. Each pair played against each other pair exactly once. Alice won 12 games and
Betty won 6 games. How many games could Claudia win? Find all possibilities.

Answer. 4.

Solution outline. We calculate that each player plays in 12 games. This implies Alice won all of
her games and also that Betty won all games which were not against Alice. This ensures we know
the result of every game and we find that Claudia won 4 games.

Problem 33 J / 23 S. Two players are playing a game on the given plan consisting of 30 cells.
The rules are the following.
• players take turns,
• in one move a player colors one cell,
• in the first move, only a cell neighbouring with the edge can be colored. In any other
moves, only a cell which is next to the last colored cell and is not further away from the
center, can be colored,
• once a cell is colored, it cannot be colored again.
• the player who can no longer make a move, loses.
How many cells will be colored by the end of the game, in which both players play perfectly and
the one who cannot win tries to make the game as long as possible?

Answer. 18.

Solution outline. Show that the second player has a winning strategy. The game is going to look
something like this with 18 moves.
10

Problem 34 J / 24 S. In triangle ABC with AC = BC, we find a point P (P 6= B) on the side


AB, such that P B < P A and ∠ACP = 30◦ . Further, we find point Q such that ∠CP Q = 78◦ and
points C and Q lie each on opposite side of AB. If the internal angles in triangles ABC and BQP
have integral values (in degrees), find all possible values of angle BQP .

Answer. 1◦ .

Solution outline.
C

30◦

78◦ P
A Q B

Since |∢P AC| = |∢CBA| < |∢CP A| and |∢P AC| + |∢CP A| = 180◦ − 30◦ = 150◦ , we must have
|∢P AC| < 75◦ . Using integral value of the angle we even get that |∢P AC| ≤ 74◦ a |∢CP A| ≥ 76◦ .
But also |∢AP Q| = |∢P BQ| + |∢BQP | ≥ 1◦ + 1◦ = 2◦ , thus

78◦ = |∢CP Q| = |∢CP A| + |∢AP Q| ≥ 76◦ + 2◦ = 78◦ .

So we must have equality everywhere, which yields |∢BQP | = 1◦ .

Problem 35 J / 25 S. Ten people in the theater are sitting next to each other in one row. After
a break they sit in a new arrangement, so that only two people remained in their original positions
and the remaining eight sat next to their former position. In how many ways could they have done
that?

Answer. 15 = 62 .

Solution outline. Show that any such seating can be represented by a sequence of six letters two
of which are S representing people who stay in their original position and four of which are C
representing a pair which changes (switches) their seats. There are 62 = 15 such sequences.

Problem 36 J / 26 S. A positive integer is written on each face of a cube. To each vertex we


assign the product of the numbers written on the three faces intersecting at that vertex. The sum
11

of the numbers assigned to the vertices is 165. What are the possible values of the sum of the
numbers written on the faces?

Answer. 19.

Solution outline. Denote the number on the faces by a, b, c, d, e, f so that the pairs a and f , b
and e, c and d are opposite on the cube. Then

3 · 5 · 11 = 165 = (a + f )(b + e)(c + d),

where the second equality is veryfied by expanding. Using that a to f are positive integers, we get
that the sum equals 3 + 5 + 11 = 19. Such numbering can easily be found.

Problem 37 J / 27 S. Two bicyclists are racing on a straight path at constant speeds. They
both start at one end and everytime they hit the end of the path, they turn around and go the
opposite direction. Eventually, they meet again at one of the endpoints. Before that, the slower
bicyclist had traveled the path 35 times (in one of the directions) and the faster one 47 times. How
many times did they meet head on?

Answer. 40.

Solution outline. A head on meeting requires two changes of direction to be made (regardless if
one bicyclist changes direction twice r both once). There are 34 + 46 = 80 changes of direction,
which means 40 head on meetings.

Problem 38 J / 28 S. Find the largest positive integer such that all its digits (other than the
first one and the last one) are smaller than the arithmetic mean of the two surrounding digits.

Answer. 96433469.

Solution outline. Show that there cannot be more than 4 consecutively decreasing or increasing
digits. Next show that the desired number consists of a decreasingsection of digits followed by an
increasing section. Playing aroung with such 8-digit numbers gives the result 96433469.

Problem 39 J / 29 S. Two tetrominoes made of 1 × 1 squares touch at points A, B, C as in the


diagram. Find the distance AB.

B C

5
Answer. 4
= 1,25.

Solution outline. Right triangle with hypotenuses AB and BC are congruent by ASA. Let x be
their shorter leg. Then 2 = x + |BC| = x + |AB| a by Pythagorean theorem also x2 + 1 = |AB|2 .
Solving the equations gives |AB| = 1, 25 dm.
12

Problem 40 J / 30 S. There are 100 points with integral coordinates given in the plane. We
connect each pair with a segment. How many of these segments are guaranteed to have a midpoint
with integral coordinates?

Answer. 1200 = 4 · 25 2
.

Solution outline. The midpoint has integer coordinates if and only if the x-coordinates of the two
points have the same parity as well as the y-coordinates. Now divide the points in four groups
depending on the parity of its x and y coordinates. Use power mean inequality to show that
extreme case is when all groups have same size. Each group gives you 252
midpoints with integers

coordinates so the answer is 4 · 25
2
= 1200

Problem 41 J / 31 S. A five-digit integer is called irreducible if it cannot be written as a pro-


duct of two three-digit integers. What is the maximum possible number of consecutive irreducible
integers?

Answer. 99.

Solution outline. To be translated.

Problem 42 J / 32 S. Real numbers x and y satisfy (x + 5)2 + (y − 12)2 = 142 . Find the
minimum possible value of x2 + y 2 .

Answer. 1.

Solution outline. To be translated.

Problem 43 J / 33 S. A sequence is definned as follows: a1 = 20, a2 = 11, and

1
an+2 = an − ,
an+1

as long as the right-hand is well-defined. Find the least t such that at = 0.

Answer. 222.

Solution outline. To be translated.

Problem 44 J / 34 S. Let ABC be an acute-angled triangle with altitudes AA′ , BB ′ , CC ′ ,


intersecting at H. Given that
|AH| |BH|
= 1, = 2,
|HA′ | |HB ′ |
|CH|
find |HC ′ |
.

Answer. 5.

Solution outline. To be translated.

Problem 45 J / 35 S. Every guest at a party (including Tim) knows exactly seven boys and
ten girls there. What is the least possible number of people at the party?

Answer. 34.

Solution outline. To be translated.


13

Problem 46 J / 36 S. Let S be the midpoint of side CD of rectangle ABCD. The incircles of


triangles ASD and BSC have radii both equal to 3 and the inradius of △ASB is 4. Find the sides
of the rectangle.
D S C

A B

Answer. 9, 24.

Solution outline.
D S C

Z
O2 O1
Y

A X D′ B

To be translated.

Problem 47 J / 37 S. We write out all divisors of a positive integer n which are less than n
from the greatest to the lowest. If n is the sum of the second and third divisor, we say that n is
additive. How many additive numbers are there which are less than 15000?

Answer. 1000.

Solution outline. To be translated.

Problem 48 J / 38 S. Find all real numbers x such that

x − 49 x − 50 50 49
+ = + .
50 49 x − 49 x − 50

4901
Answer. 99, 0, 99
= 49 50
99
.

Solution outline. To be translated.

Problem 49 J / 39 S. A position of a minute hand and an hour hand on the clock is called
valid, if it might occur during one 12 hour cycle. Find the number of valid positions which remain
valid after switching the two hands.

Answer. 143.

Solution outline. To be translated.


14

Problem 50 J / 40 S. Let a, b, c be nonzero real numbers, such that the quadratic equations
ax2 + bx + c = 0 and bx2 + cx + a = 0 have a common root. Find all possible real values of this
root.

Answer. 1.

Solution outline. To be translated.

Problem 51 J / 41 S. Find all integers n such that both 16n+9 and 9n+16 are perfect squares.

Answer. 0, 1, 52.

Solution outline. To be translated.

Problem 52 J / 42 S. A regular octahedron with side length 2 is given in space. One circle is
inscribed in one of the faces and another circle is circumscribed about an adjacent face. What is
the minimal distance between the two circles?

√ p √
Answer. 2−1= 3 − 2 2.

Solution outline. To be translated.

Problem 53 J / 43 S. Let ABC be a triangle with circumradius 5 and inradius 2. Three circles
with radius r are inscribed in angles BAC, CBA, ACB, respectively such that the circles are in
the interior of the triangle and there exists another circle with radius r which is tangent to all three
circles. Find r.
A

B C

10
Answer. 9
.

Solution outline. To be translated.


15

Problem 54 J / 44 S. Real numbers a, b, x, y satisfy

ax + by = 3,
ax2 + by 2 = 7,
ax3 + by 3 = 16,
ax4 + by 4 = 42.

Find ax5 + by 5 .
Answer. 20.

Solution outline. To be translated.


Problem 1. If we increase the length of each edge of a cube by 100%, by what percent does the volume increase?

Result. 700%

Solution outline. Increasing the length of an edge a by 100% is the same as multiplying it by 2. The volume of the
cube is now a3 , after multiplication it will be (2a)3 = 8a3 . There is an increase by 7a3 , which is 700%.

Problem 2. What is the largest number of pieces a ring can be divided into using three straight lines?

Result. 9

Solution outline. We want all intersections of the lines to be distinct, lie in the ring and every line to be a secant of
the inner circle.

Problem 3. A five digit number a679b is divisible by 72. Find the value of a · b.

Result. 3 × 2 = 6

Solution outline. Since a679b is divisible by 8, it follows b = 2. Since it is divisible by 9, 9 | a + 6 + 7 + 9 + 2 implying


that a = 3.

Problem 4. A math teacher decided to organize two rounds of math competition. Each team consisted of five
members. In the first round, the students divided themselves into teams on their own. In the second round the
teacher divided them so that nobody was in the same team with anyone he or she has played with in the first round.
Determine the minimum number of students in which such division is possible.

Result. 25

Solution outline. The number of students is divisible by 5 and is larger than 20 (Pigeonhole principle). It is easy to
verify that 25 is sufficient.

Problem 5. The whole surface of a rectangular prism-shaped vanilla cake with edges of lengths 10, 10 and 5 is
covered by a thin layer of chocolate. Let us cut the cake into cubes of volume 1. What percentage of cubes have no
chocolate on them?

Result. 38.4% = 48/125

Solution outline. Consider what is left after eating all the pieces with chocolate layer. It will be a rectangular prism
8 × 8 × 3 out of all pieces without chocolate. There must be then 192 pieces of cake without chocolate on it. From
the total amount of pieces with chocolate 10 · 10 · 5 = 500 we easily count the desired percentage: 100 · (192/500)%.

Problem 6. We are√given a square ABCD with side length 2 and a point X in its plane (outside the square) so
that |AX| = |XB| = 2. What is the length of the largest diagonal in the pentagon AXBCD?

Result. 10

Solution outline. We aim to find CX. Let E be the foot of perpendicular dropped from X onto BC. As 4AXB is
right and isosceles we have EB = EX = 1 and the rest follows from Pythagorean theorem.

1
D C

A B

X 1

Problem 7. Jacob had written 5 · 414 = 1121 on a blackboard. Now he is wondering how to increase or decrease
each of the digits by 1 so that the result is correct. What will be the right-hand side after the change?
Result. 2012
Solution outline. It suffices to go through 16 ways to vary the left-hand side and see if the right-hand side can be
changed accordingly. The process can be speeded up by various observations (for instance the result has to start with
2, hence the first number is 4 and the second one starts with 5).

Problem 8. The sum of integers x and y is at most 200 and their difference is smaller than 100. Find the maximum
value of the expression 2 · min(x, y) + max(x, y).
Result. 300
Solution outline. Without loss of generality let x ≥ y. Then 2 · min(x, y) + max(x, y) = 2y + x = y + (x + y) ≤
y + 200 ≤ x+y
2 + 200 = 300. This value is achieved for x = y = 100.

Problem 9. Let x, y, z be real numbers such that the arithmetic mean of x and 2y is equal to 7, and arithmetic
mean of x and 2z is equal to 8. What is the arithmetic mean of x, y, and z?
Result. 5
Solution outline. Adding the given equations we get
x + 2y x + 2z
15 = + = x + y + z.
2 2
Divide by 3 to get the result.

Problem 10. 81 trees grow on a 8 × 8 square grid. The gardener cut out one of the corner trees and is now standing
at that corner facing the rest of the grid. However, he does not see some of the trees because they are aligned with
the other trees. (A tree T is aligned with another tree if there exists a tree on the line segment between the gardener
and T .) How many trees does the gardener see?

Result. 45
Solution outline. Let us order the trees by their distance to the gardener and check each one for visibility. If a tree
is not crossed out, we mark it as visible and cross out all the trees aligned with it.

2
Problem 11. How many ways are there to color the faces of a cube with two colors? Two colorings are considered
identical if we can get one from the other by rotating the cube.

Result. 1 + 1 + 2 + 2 + 2 + 1 + 1 = 10

Solution outline. We shall distinguish the cases according to the number of black faces involved in the coloring. For
zero black faces we have one option as well as for one black face. For two black faces there are two ways—either
the faces are neighbouring or they are not. For three black faces there are again two colorings (either these three
faces share a common vertex or they do not). For four, five, and six the situation is analogous to the one with two,
one, and zero black faces, respectively (we can place the white faces instead of the black ones). Altogether, we have
1 + 1 + 2 + 2 + 2 + 1 + 1 = 10 ways to color the cube.

Problem 12. A rectangle ABCD is given with AB = 20 and BC = 12. Let Z be the point on the ray BC such
that CZ = 18 and E the point inside ABCD such that the distance from E to both AB and AD is 6. Let line EZ
intersect AB and CD at points X and Y , respectively. Find the area of AXY D.

Result. 72

Solution outline. Focus on trapezoid AXY D and observe that E is the midpoint of its side XY . Hence if we denote
the foot of perpendicular dropped from E onto AD by E 0 then EE 0 is the midline of the trapezoid AXY D and its
area is thus AD · EE 0 = 72.
Z

18

D Y C

6 12
E0 E

A X 20 B

Problem 13. For how many positive integers a (1 ≤ a ≤ 2012) is aa a square of a positive integer?

Result. 1028

Solution outline. If a = 2k is even then aa = (ak )2 is always a square. If a is odd then aa is a square if and only if a
itself is a square. As 442 < 2012 < 452 the answer is 21 2012 + 12 44 = 1028.

Problem 14. An equilateral triangle with side length 1 is lying on the floor, with one altitude perpendicular to the
floor. We color one of its vertices red and we “roll” the triangle on the floor (in the plane of the triangle) through one
full rotation. What is the length of the red vertex’s trajectory?
2
Result. 3 · 2π = 43 π

Solution outline. Observe that the red vertex traces circular arcs. Twice it traces 13 of a circumference of a circle
with radius 1 and once it remains fixed. Hence the total length of its trajectory equals ( 31 + 13 ) · 2π · 1 = 43 π.

Problem 15. What is the smallest positive integer consisting only of the digits 0 and 1 that is divisible by 225?

Result. 11111111100

Solution outline. The number has to be divisible by 25 so it ends with two zeroes. Moreover, it has to be divisible
by 9 (225 = 9 · 25), so the number of 1’s is a multiple of nine. The smallest positive integer with these properties is
11111111100.

3
Problem 16. Bill is old enough to vote but not old enough to use the senior discount. (His age is between 18 and
70.) It is known that x years ago, the square of his age was the same as his current age increased by x. Moreover,
Bill’s age is a square of an integer. Find x.

Result. 28

Solution outline. Bill’s age a is the arithmetic mean of the positive integer a − x and its square a + x. Therefore
all possible values of a are 12 (6 + 36) = 21, 12 (7 + 49) = 28, 21 (8 + 64) = 36, 12 (9 + 81) = 45, 12 (10 + 100) = 55, and
1 2
2 (11 + 121) = 66. The only square among them is 36, hence x = 36 − 8 = 8 − 36 = 28.

Problem 17. Let us fold the bottom left corner of a rectangular paper to its top right corner. The resulting figure
consists of three triangles created by the edges of the paper and the fold. For what ratio of side lengths of the paper
is the ratio of the areas of the triangles 1 : 2 : 1?
√ √
Result. 3 : 3 = 1 : 3 or in reverse order.

Solution outline. For the two corners to coincide we have to fold about the line passing through the center of the
rectangle perpendicular to one of its diagonals.

Focus on the resulting pentagon consisting of two congruent right triangles and an isosceles triangle in the middle.
The diagonal splits the isosceles triangle into two congruent right triangles. Using the ratio of the areas we learn
that in fact all four right triangles forming the pentagon are congruent. Hence the angle by their common vertex is
1 ◦ ◦
3 · 90 = 30 . The ratio of the side lengths of the rectangle is readily obtained invoking the symmetry about the fold
line.

Problem 18. How many three digit numbers are divisible by 6 if each digit is larger than 4?

Result. 16

Solution outline. The number has to be divisible by 2, thus it must end with 8 or 6. Also, it has to be divisible by
3, so its digit sum has to be divisible by 3, which implies that after dividing by 3 the sum of the first two digits gives
remainder 1 (if the last digit is 8) or 0 (if the last digit is 6). The remainder 1 can be achieved as a sum of numbers
with remainders 2 + 2, 3 + 1 (the order of the digits matters, so we have 4 + 4 = 8 combinations). Similarly the
remainder 0 can be achieved only as 0 + 0, 2 + 1 (again 4 + 4 = 8 combinations).

Problem 19. Three circles with radius 1 are given, such that each two are externally tangent. We put all the circles
into a greater circle ω. Each small circle is internally tangent to the greater circle. Find the radius of the circle ω.


√2 2 3
Result. 1 + 3
=1+ 3

Solution outline. Denote by A, B, C the centers of the smaller circles and by S the center √ of ω. Observe
√ that A,
B, C are vertices of an equilateral triangle with side length 2. Its altitudes are of length 22 − 1 = 3, and they

intersect at S. However, in equilateral triangle the altitudes are at the same time the medians. Hence AS = 32 · 3.
The radius of ω is then the sum of AS and the radius of the smaller circle.

4
C

A B

Problem 20. Let n be a positive integer. If n2 has 7 in the tens place, which digits can be in the units place?
Result. 6
Solution outline. Let n = 10x + y for an integer x ≥ 0 and a digit y. Then n2 = 100x2 + 20xy + y 2 . The digit in the
tens place is odd if and only if the digit in the tens place of the number y 2 is odd. Therefore y 2 is 16 or 36. The last
digit in both cases is 6.

Problem 21. If we write the numbers 1, 2, . . . , n in some order, we will get an n-chain. For example, one possible
11-chain is
3764581121910.
What is the smallest n with n > 1 such that there exists an n-chain that is a palindrome? (A number is a palindrome
if it may be read the same way in either direction.)
Result. 19
Solution outline. The following is a palindromic 19-chain:

9|18|7|16|5|14|3|12|1|10|11|2|13|4|15|6|17|8|19.

We will show that 19 is the smallest possible value of n. First note that only one digit can appear in a palindromic
chain odd number of times (namely the middle one). Clearly, for n ≤ 9 this condition cannot be satisfied. Similarly,
for 10 ≤ n ≤ 18 both digits 0 and 9 appear exactly once, thus such an n-chain cannot be a palindrome.

Problem 22. Find all triples (x, y, z) of positive real numbers for which (x+y)(x+y+z) = 120, (y+z)(x+y+z) = 96,
(z + x)(x + y + z) = 72.
Result. (4, 6, 2)
Solution outline. Adding the equations yields (x + y + z)(z + y + x + z + x + y) = 288. Since we are looking for x, y,
z positive, after dividing by 2 and taking the square root, we have x + y + z = 12. By substituting back to the original
equations we get (12 − z) = 10, (12 − x) = 8, (12 − y) = 6, from which we easily obtain the solution (x, y, z) = (4, 6, 2).

Problem 23. A circle is given with a radius 1 and two perpendicular chords inside it, dividing the circle into 4 parts.
We color the part with the greatest and the part with the smallest area in black and leave the rest white. We know
that the area of the white parts is the same as the area of the black parts. What is the maximum possible distance of
the longer chord to the centre?
Result. 0
Solution outline. Draw the chords symmetrical to the two we already have about the center of the circle and observe
that if the two areas are to be the same then one of the chords has to be a diameter of the circle.

c
a a

b d b

a a
c

5
Problem 24. An officer’s route consists of three circles (shown in the picture). He must start at A and travel the
entire route, without visiting any portion twice except perhaps the intersection points, and return to A. If his direction
matters, how many such routes exist?

Result. 16
Solution outline. The officer has to walk around the first building either in the very beginning or in the very end
of his route (in any of the two possible directions) which gives us 4 options. For any of the remaining two buildings
he may independently decide if he walks around them clockwise or counterclockwise. These decisions determine the
route uniquely, hence the answer is 4 · 2 · 2 = 16.

Problem 25. A trapezoid has bases of lengths 5 and 2 and legs of lengths 3 2 and 3. What is its area?
21
Result. 2

Solution outline. A line parallel to the shorter leg passing through


√ another vertex of the trapezoid splits it into a
parallelogram and a triangle with side lengths 3, 5 − 2 = 3, and 3 2, which is therefore isosceles and right. The area
is now simply 12 · 3 · 3 + 3 · 2 = 21
2 .


3 3 3 2

2 3

Problem 26. There are 42 people in a row. They want to order themselves according to their height, so that the
tallest one will stand in front. In one step, two people who are next to each other can switch position. At most, how
many steps are necessary for them to order as they want?
n(n−1)
Result. 2 = 21 · 41 = 861
Solution outline. Let us assign to each ordering a value H that denotes the number of (not necessary neighbouring)
pairs such that the taller person stands behind the shorter one. Every step decreases H by at most one and if H > 0
then there exists a step that decreases it. In the beginning, H is at most 12 · 41 · 42 (if every pair is switched, i.e. if
the people are in the opposite order). Thus, the “worst” ordering requires 861 steps.

Problem 27. Parsley lives in Vegetable State where one can pay only by coins with values 7 and 11. If Parsley had
an unlimited supply of both kinds of coins what would be the highest integer price he could not pay with them?
Result. 59
Solution outline. If he can pay some price, he can also pay this price plus a multiple of 7. Thus, it is enough to
observe the multiples of 11 and their remainders modulo 7. 0 mod 7 = 0, 11 mod 7 = 4, 22 mod 7 = 8, 33 mod 7 = 5,
44 mod 7 = 2, 55 mod 7 = 6, 66 mod 7 = 3. From 66 on, he can pay all the amounts, as the remainders repeat
themselves. Thus, the last price he cannot pay is 66 − 7. (Should he be able to pay this amount, it would hold
66 − 7 = 11k + 7m. The left-hand side modulo 7 is 3, while the right-hand side is not 3 as the first multiple of 11
(11k) with remainder 3 is 66 as shown above.)

Problem 28. Let us divide a circle with radius 1 into 4 parts. What is the smallest possible perimeter of the part
with the greatest area? If there is more than one part with the greatest area, we take into account the one with the
smallest perimeter.
Result. π
Solution outline. A shape with a given area has the smallest perimeter if it is a circle. So all that needs to be done
is to cut out the circle with area consisting of one fourth of the big circle and dividing the rest into three parts with
equal area.

6
Problem 29. Find the sum of all real numbers a for which the equations x2 + ax + 1 = 0 and x2 + x + a = 0 have
at least one common real root.
Result. −2
Solution outline. Let x be a common solution of both equations. By subtracting the equations we get (a−1)(x−1) = 0.
So either x = 1 or a = 1. If x = 1, a = −2 (substitute back). In the latter case the equations do not have any real
roots.

Problem 30. How many 8-digit numbers are there such that after crossing out its first digit (from the left), one
gets a number 35 times smaller than the original one?
Result. 0
Solution outline. Let m be a number that we get after crossing out the first digit and c be the crossed out digit. The
1
original number is then c · 107 + m. It should hold that m = 35 (c · 107 + m), which can be simplified to 17m = c26 57 .
However, the left-hand side is nonzero and divisible by 17, while the right-hand side is not for c ≤ 9.

Problem 31. Consider a right triangle with sides of integer length. One of the sides has length 2012. What is the
maximum area it can have?
Result. 1006 · (10062 − 1) = 1018107210
Solution outline. In order to maximize the area, the side of length 2012 is surely the shorter leg. From the Pythagorean
theorem we get b2 + 20122 = c2 . We want to maximize 2012 · b/2. That is the same as to minimize the difference
between the lengths of the hypotenuse and the longer leg. If c = b + 1, the parity does not match. If c = b + 2, we get
a simple equation for b.

Problem 32. Let ABC be a triangle with circumcentre O and orthocentre H in the Cartesian plane. No two of
these five points coincide and all of them have integer coordinates. What is the second smallest possible radius of a
circle circumscribed around triangle ABC?

Result. 10
√ √
Solution outline. Consider the possible √ radii √
of the excircle. For 1, 2, 2 and 2 2 we see that the orthocentre
coincides with one of the vertices. For 5 and 10 we can construct the desired triangles.

H
O O

A B A B

Problem 33. Find the largest natural number n such that the number 72048 − 1 is divisible by 2n .
Result. 14
Solution outline. Use the formula a2 − b2 = (a − b)(a + b) several times to get 72048 − 1 = (7 − 1)(7 + 1)(72 + 1)(74 +
1) . . . (71024 + 1). But for the (7 + 1), all the other factors in the product are divisible by 2 exactly once, since for every
positive integer k, 72k + 1 gives a remainder 2 when divided by 4.

7
Problem 34. If we calculate the product of the digits of some number, the product of the digits of the product,
and so on, we arrive after a finite number of steps at a one digit number. The number of required steps is called the
persistence of the number. For example, the number 723 has persistence 2 because 7 · 2 · 3 = 42 then 4 · 2 = 8. Find
the greatest even number with mutually distinct digits that has persistence 3.
Result. 98764312
Solution outline. Let us observe the highest number that satisfies the condition of different nonzero digits which is
987654321, but it has persistence only 2 because there is a five and an even number, so that in the second step we
obtain 0. To obtain the result, it suffices to get rid of 5, and switch the last two digits to satisfy the evenness.

Problem 35. If we extend the sides AD and BC of a convex quadrilateral, they intersect at a point E. Let us
denote H and G the midpoints of BD and AC, respectively. Find the ratio between the area of a triangle EGH and
the area of a quadrilateral ABCD. (We let you know that this ratio is the same for all convex quadrilaterals with
non-parallel sides.)
Result. 1 : 4
Solution outline. Let us consider a quadrilateral in which points C and D merge into one point. Then GH is a
midline in ABC and the ratio is obviously 41 .

Problem 36. Cube termites bore four straight square tunnels of sidelength 1 in each direction inside a cube (as you
can see on the picture) and now they have left the cube. How many cm2 of paint do we need to cover the surface of
what is left of the cube if the original cube had side length 5 cm?

Result. 270
Solution outline. Let us imagine that we are looking at the cube from above and count the area of the surfaces
oriented upwards. In the depth of 0 cm (the side of the cube) there are 21 cm2 , in the depth of both 2 and 4 cm there
are 12 cm2 . The situation is the same from all directions.

Problem 37. We are given a circle with radius 1. We are standing on the leftmost point of the circle. It is possible
to move only up and right. What is the length of the longest trajectory that we can travel inside the circle?

Result. 1 + 2
Solution outline. For each trajectory there exist a trajectory of the same length that first moves only right and
then only upwards, so it suffices to consider such trajectories only. The longest trajectory must clearly pass through
the centre. Let us denote a the length of the trajectory travelled to the right from the centre of the circle and
b the length of the trajectory that we will travel from the centre upwards. As the longest route will end at the
circumference of the circle, it holds that a2 + b2 = 1. We want to maximize a + b, which is equivalent to maximizing
(a + b)2 = a2 + b2 + 2ab = 1 + 2ab. From the trivial inequality 0 ≤ (a − b)2 = a2 √
− 2ab + b2 we have 2ab ≤ a2 + b2 = 1.
p
The equality occurs when we have equality for 0 ≤ (a − b)2 , so a = b = 1/2 = 22 . Adding the distance to the centre

of the circle, we get the result 1 + 2.

Problem 38. What is the greatest divisor of 15! = 1 · 2 · . . . · 15 that gives a remainder of 5 when divided by 6?
Result. 5 · 5 · 7 · 7 · 11 · 13 = 175175
Solution outline. The remainder of the product is the same as the product of the remainders of the factors, for
example 7! mod 6 = 1 · 2 · 3 · 4 · 5 · 0 · 1 = 0. In order to make the final remainder equal to 5, the number can be divisible
by neither 2 nor 3, so we have to exclude all the two’s and three’s from 15!. After that we are left with the number
53 · 72 · 11 · 13, which gives the remainder 1. Each factor gives a remainder either 1 or 5, so we just have to exclude
one number with remainder 5. The smallest such number is 5.

8
Problem 39. A cube and 27 of its points are given: the vertices, midpoints of the edges, centers of the faces, and
the center of the cube. How many lines are passing through exactly three of the given points?
Result. 49
Solution outline. We divide the lines into three groups: those that pass through the center of the cube (9 + 8/2 = 13),
those that pass through the center of the side but not the center of the cube (4 for each side), and those lines on which
the edges lie (12). We conclude that there are 13 + 24 + 12 lines altogether.

Problem 40. n participants took part in a competition that lasted for k days. Each day each participant received
an integer amount of points between 1 and n, inclusive. No two participants had the same score in a given day. At
the end of the competition (the k-th evening) each participant had a score of 26 points (when all the points for the
whole competition were added). Find the sum of all n for which this is possible (regardless of k).
Result. 1 + 3 + 12 + 25 = 41
Solution outline. It is clear that n < 26. Altogether, there were n(n+1)2 points allocated each day, that gives a total sum
of k· n(n+1)
2 points. However, that is also equal to 26n. Comparing these two expressions, we get k(n+1) = 52 = 2·2·13.
Trying out several possibilities, we find out that for k ∈ {2, 4, 13, 26} we can find a way in which the contestants could
receive points during the competition and for k ∈ {1, 52} we cannot find such a way. The sum of all n is then
(26 − 1) + (13 − 1) + (4 − 1) + (2 − 1).

Problem 41. We want to cut a cylindrical cake with 5 straight cuts. What is the maximum number of resulting
pieces? For example, by 3 cuts we can divide the cake into 8 pieces.
Result. 26
Solution outline. For the sake of simplicity, consider an infinite 3-dimensional space that we cut by planes. When the
space is cut in some way, the next plane adds as many new parts as it had cut through. Let us take the lines that are
intersections of the older planes with the new plane. The number of the new pieces is the same as the number of the
parts that the lines are dividing the new plane into. Thus, vn = vn−1 + pn−1 , where vn is the result for n intersections
and pn is the number of new parts, i.e. the number of parts into which we can divide a plane by lines. By drawing
on paper we can find the maximum values for p1 , p2 , p3 and p4 , which are 2, 4, 7 a 11. For v1 = 2 we get the result
2 + 2 + 4 + 7 + 11 = 26.
For curious souls, this is a generalization: n3 + n2 + n1 + n0 = 1/6(n3 + 5n + 6).
   

Problem 42. An eight-branched star (Stella octangula) is a solid, which results from sticking eight regular tetrahe-
drons to the faces of an octahedron. All the edges of each tetrahedron and the octahedron have length 1. What is the
volume of the eight-branched star?

Result. 2
Solution outline. We have two types of solids: A regular tetrahedron with side length 1 and a tetrahedral pyramid with
all sides of length 1. The base area of the tetrahedral pyramid is 1 and the height can be obtained from Pythagorean
q √ 
2
theorem as 1 − 22 = √12 , so its volume is 3·√
2 1
2
. Similarly, the base area of the tetrahedron (equilateral triangle)
√ q √ 
2
√ √ √ √ √
is 43 and the height 12 − 2 6 3 = √23 , so its volume is 3·42 . When we sum it up, we get 2 · 62 + 8 · 122 = 2.
Other solution: Watch what happens if you inscribe the star into a cube.

Problem 43. A hitchhiker is walking along the road. The probability that a car picks him up in the next 20 minutes
is 609
625 . What is the probability of a car picking him up in the next five minutes, if the probability that he gets picked
up by a car is the same in each moment?
Result. 3/5
Solution outline. The probability that the hitchhiker is not picked during the first 20 minutes is 1 − 609 16
625 = 625 . If
4
the probability of him not being picked during 5 minutes is p, then for the 20 minutes it is p . Hence p = 2/5 and the
probability of catching a car is 1 − 2/5.

Problem 44. A vandal and a moderator are editing a Wikipedia article. At the beginning, the article was without
a mistake and each day the vandal adds one mistake. At the end of each day the moderator has 23 chance of having
found each single mistake that is in the article. What is the probability that after three days the article will be without
a mistake?
25 ·13
Result. 2/3 · 8/9 · 26/27 = 36
Solution outline. For each mistake we find the probability that it won’t stay till the end. The probability that the
mistake will stay k days is (1/3)k , and the probability that some mistake will not stay k days is 1 − (1/3)k . The events
of not spotting a mistake are independent, so we can multiply: (1 − 1/3)(1 − 1/9)(1 − 1/27).

9
Problem 45. We have a large enough heap of red, blue, and yellow cards. We can receive the following number of
points:
• for each red card one point,
• for each blue card twice the number of red cards as points,
• for each yellow card three times the number of blue cards as points.
What is the maximum number of points we can receive when we have 15 cards?
Result. 168
Solution outline. Let R denote the number of red cards, B the number of blue cards and Y the number of yellow
cards. Each red card adds 1 + 2B points to the overall score (1 for itself and 2 for each blue card) and each yellow
card adds 3BY points. Therefore R + 2RB + 3BY is the overall score. For B = 0, the maximum score is 15. For
B = 1, the score is always R + 2R + 3Y = 3(R + Y ) = 42. For B > 1, it is worth to swap all red cards for yellow
cards. Therefore we choose R = 0 for B > 1 and get the overall score 3BY in this case. Respecting the condition
B + Y = 15, we get the maximum at B = 7 and Y = 8, which yields the score 168.

Problem 46. Matthew has one 20-sided die and his friend CD has three 6-sided dice. What is the chance that after
rolling all the dice, the value on Matthew’s die will be greater than the sum of the values on CD’s dice?
19
Result. 40
Solution outline. First observe that both rolls have a symmetric distribution, i.e., the probability that we get x on a
20-sided die is the same as the probability that we get 21 − x (from 1 to 20). Similarly, the probability that we get y
on three 6-sided dice is the same as the probability that we get 21 − y (from 3 to 18). By a similar line of thought
we may conclude that the probability of Matthew winning is the same as the probability of CD winning. Thus, the
answer is 1−p
2 , where p is the probability of a draw. And that is 1/20, because whatever CD rolls, Matthew always
has a 1/20 chance that he rolls the same.

Problem 47. Let us have a 10 by 10 board. The rows and columns are numbered from left to right and top to
bottom, respectively, by integers from 1 to 10. In each cell we write the product of the row number and column
number. A traveler stands on the top left cell and wants to arrive to the bottom right cell. However, she can only
travel right and down (not diagonally). A traveler’s number is the product of the numbers on the cells which she
had stepped onto (including the first and the last one). What is the greatest common divisor of all possible traveler’s
numbers?
Result. 10 × 10! · 10! = 217 · 38 · 55 · 72
Solution outline. During each route, the traveler gathers each of the numbers 2, 3, . . . , 9 at least twice (one time for
the column and one time for the row). Furthermore, for each i except for 1 and 10, there exists a route that crosses
the i-th row exactly once and i-th column exactly once. Thus, the 2, 3, . . . , 9 will occur in the result exactly once. As
for the 10, each traveler’s number contains at least three 10’s. (The traveler must step on the bottom right corner
and on the previous where there is 10, too.) It is easy to verify that there is a route that does not contain more than
three 10’s.

Problem 48. We have a triangle with altitudes of sizes 3, 4, and 6. What is its perimeter?

√72 24 15
Result. 15
= 5

Solution outline. From the fact that the area of the triangle is half of the product of its height and length of its side
we get that the triangle will be similar to the triangle with sides 2, 3, 4. We can express one of the altitudes using
its sides. For example, we can
√ use the Heron’s formula for sides of length242a, 3a a 4a and then compare this with the
2a·6 a2
area 2 . Thus, the A = 4 135 = 6a, from which it follows that a = √135 and perimeter is then 9a.


q p
Problem 49. Find the greatest integer n ≤ 4 000 000 for which n+ n+ n + . . . is rational.
Result. 1999 · 2000 = 3998000
√ √
q p
Solution outline. Denote s = n+ n+ n + . . ., then s = n + s. By solving the quadratic equation for s we

get s = 1± 21+4n , and because s is rational, 2
also 1 + 4n = a for some a rational. After simple manipulation we have
2
n = a 4−1 . In order for n to be an integer, a has to be an integer as well: If it were a = pq with p, q coprime, then
2 2
n = p 4q−q2 and q would divide p, which is a contradiction. Since n = a−1 2 ·
a+1
2 , a must be odd. Moreover, from
n ≤ 4 000 000 we have a+12 = 2000. Thus, n = 1999 · 2000 is the solution.

10
Problem 50. A 3 × 3 Maxi-square is a square divided into nine square tiles. Each tile is divided into four little
squares in which the numbers 1, 2, 3, and 4 are written (each of them exactly once). Two tiles can touch only if their
adjacent numbers match (as in dominoes). How many Maxi-squares exist?

4 3 3 4 4 2
1 2 2 1 1 3

Result. 24 · 7 = 168

Solution outline. First we choose the tile in the middle to be (1, 2 and 3, 4). Denote by A, B the numbers written in
the little square at the coordinates (4, 1) and (4, 6), respectively ((1, 1) being the upper left corner square). Observe
that A ∈ {3, 4} and B ∈ {1, 2}. If the couple (A, B) equals one of (3, 2), (4, 1), (3, 1), there is only one way to fill the
rest of the maxi-square. In the case (A, B) = (4, 2), there are four ways. As there are 4! = 24 possible middle tiles,
the result is 24 · 7.

Problem 51. Andrew calls his favourite number balloon. It holds for balloon that

• the sum of its digits is twice the number of its digits

• it does not have more than 12 digits

• its digits are in turn even and odd (it doesn’t have to start with an even digit)

• the number greater by one is divisible by 210.

Find the value of Andrew’s balloon.

Result. 1010309

Solution outline. Let us focus on the condition that the number increased by 1 is divisible by 210 = 2 · 3 · 5 · 7. From
this it immediately follows that the last digit has to be 9. Because of the divisibility by three, the digit sum of the
balloon is of the form 3k + 2 and also it should be twice the number of digits from the statement. So the digit sum
is even, of the form 3k + 2 and greater than 9, thus it can only be 14 or 20, to which correspond numbers with 7 and
10 digits, respectively. Since odd and even digits alternate, the smallest possible solutions relaxing the condition on
divisibility by 7 are 1010109 and 2101010109 with digit sums 12 and 15. The digit sum in the second case is odd, so
there is no way of achieving the digit sum of 20. In the first case we have to increase the digit sum by 2, which means
that we have to increase one digit by 2. We can easily check that the only solution (taking divisibility by 7 back into
consideration) is the number 1010309.

Problem 52. Find all four-digit positive integers n such that the last four digits of the number n2 is the number n.

Result. 9376

Solution outline. We seek n such that n2 − n = n(n − 1) = 10000x for some integer x. As 54 divides the right-hand
side, it must divide the left-hand side; thus 54 = 625 divides either n or n−1, since they are coprime. Similarly 24 = 16
divides either n or n − 1. The former divisibility implies that n is of the form 625k + b, where k ∈ {1, 2, . . . , 15} and
b ∈ {0, 1}. The remainder of 625 when divided by 16 is 1, therefore the remainder of n when divided by 16 is k + b.
Hence k + b ∈ {0, 1, 16} from the latter divisibility. It is then readily seen that n has 4 digits if and only if k = 15,
b = 1, n = 9376; we can easily verify that this n satisfies all the conditions.

Problem 53. Find the sum of all five-digit palindromes.

Result. 900 · 55000 = 49500000.

Solution outline. We establish a one-to-one correspondence between 5-digit palindromes and 3-digit numbers—the
number abc corresponds to the palindrome abcba for a non-zero digit a and arbitrary digits b and c. The total sum of the
palindromes can be then computed digit-wise; each digit a adds a·10001, each digit b adds b·1010 and each c adds c·100
to the total sum. Each possible value of a appears in the total sum 100 times and each possible value of b or c appears 90
times. The total sum is thus (1+2+· · ·+9)(100)(10001)+(0+1+· · ·+9)(90)(1110) = 45004500+4495500 = 49500000.

11
Problem 54. How many ordered quadruples of odd positive integers (a, b, c, d) satisfy a + b + c + d = 98?
Result. 50

3 = 19600

Solution outline. First we rewrite the problem to get rid of the condition of a, b, c, d being odd. Let a = 2A + 1, and
likewise for b, c, d. Then there is a one-to-one correspondence between quadruples (a, b, c, d) of odd positive integers
satisfying a+b+c+d = 98 and quadruples (A, B, C, D) of nonnegative integers satisfying A+B+C+D = 21 (98−4) = 47.
The solution is now a standard exercise on combinations with repetitions allowed; the answer is 50

3 (each quadruple
(A, B, C, D) corresponds to a sequence of 47 “balls” and 3 “separators”).

Problem 55. Find the only eleven-digit number such that


• it starts with a one
• when it it is written twice in a row, it is a perfect square.

Result. (1011 + 1) · 16/121 = 16 · 826446281 = 13223140496


Solution outline. Let n be the number satisfying the conditions above. The number N created by subsequently
writing n two times is equal to n(1011 + 1) = n · 112 · D for a suitable integer D. Therefore, if we let n = t2 · D, we
obtain N = (11 · t · D)2 , which is a perfect square. It remains to choose t in such a way that n has eleven digits and
its decimal representation starts with 1. The only possible choice is t = 4.

Problem 56. Two different triangles with side lengths 18, 24, and 30 are given such that their incircles coincide and
their circumcircles coincide. What is the area of the polygon that the triangles have in common?
Result. 132
Solution outline. Let ABC be one of the triangles (AC = 24, BC = 18). Let I be the incenter of ABC and O its
circumcenter. Since (18, 24, 30) = 6 · (3, 4, 5), the angle ACB is right and the radii r and R of the inscribed and the
circumscribed circle of ABC, respectively, are found as r = 12 (18 + 24 − 30) = 6 and R = 12 30 = 15. There is only one
diameter A0 B 0 of the circle (ABC) different from AB that is tangent to the incircle of 4ABC; it is the one symmetric
to AB with respect to the line OI. Note that A0 B 0 lies on the perpendicular bisector of AC. Indeed, this bisector
passes through O, and since its distance from BC is 12, its distance from I is 6. The other triangle A0 B 0 C 0 is then
symmetrical to ABC w.r.t. OI and A0 B 0 ⊥ AC and A0 C 0 ⊥ AB. Finally, the intersection of our two triangles can
be computed by taking the area of ABC and subtracting the areas of three small triangles similar to ABC; their
side lengths are 9, 12, 15 (the triangle at A), 6, 8, 10 (at B), and 3, 4, 5 (at C), respectively. Thus the area of the
intersection is
1
(18 · 24 − 9 · 12 − 6 · 8 − 3 · 4) = 132.
2
A0

O
I
C0

C A

B0

Problem 57. In how many ways can we color the cells of a 5 × 5 grid with black and white so that in each column
and row there will be exactly two black squares?
Result. 2040
Solution outline. Let us paint the ten cells one after another. Denote the cells A1, . . . , E5 (columns by letters).
Without loss of generality, the cells A1, B1 are painted (from symmetry, it is sufficient to multiply the answer by
5

2 = 10) and also the cell B2 (multiply by four). If the cell A2 is painted, it remains to determine the number of
complying paintings of 3 × 3 grid, which is 6. If a different cell in the second row is painted, w.l.o.g. let it be C2
(multiply by 3), and in the third column C3 (again multiply by 3). If A3 is now painted, there is only one possibility

12
how to finish (D4, D5, E4, E5 are painted). Otherwise there are four possibilities (without loss of generality in the
third row D3 is painted and D4 in the fourth column). To sum up, the overall number of colorings is

10 · 4 · 6 + 3 · 3 · (1 + 2 · 2) = 40 · 51 = 2040.

Problem 58. Two distinct points X and Y lie inside a square with side length 1. By remoteness of a vertex of the
square we mean its distance to the closer of the points X and Y . What is the smallest possible sum of the remotenesses
of the vertices of the square?
√ √
6+ 2
Result. 2

Solution outline. Let V be the value we are minimizing. If X = A and Y = B then V = 2 and using triangle
inequalities it is easy to show that if X is closer to exactly 0, 2 or 4 vertices than Y , then V ≥ 2. Next, let us assume
that the remoteness of A is measured to X and the remotenesses of B, C, D are measured to Y , i.e., let us minimize
the value V = AX + BY + CY + DY . The minimum is attained when X = A and Y is the Fermat point of the
triangle BCD, that is if ∠BY C = ∠CY D = ∠DY B = 120◦ . (The required property of the Fermat point can be
verified by rotating the triangle BY C around B by 60◦ into a triangle BY 0 C 0 and comparing the length of the broken
line DY Y 0 C to the length of the segment DC 0 ). Finally, we compute DC 0 using Pythagorean theorem. It turns out
to be less than two so it is the answer.
D C

Y0 C0
Y

A=X B

Problem 59. The parking lot consists of 2012 parking places regularly distributed in a row with numbers 1 to 2012.
2012 cars park there one after each other, in the following way:

• The first car chooses randomly one of the 2012 places.

• The following cars choose the place from which the distance to the closest car is the greatest (each of such places
with the same probability).

Determine the probability that the last car parks in the parking place number 1.

Result. 1/2012 · 1/1025 = 1/2062300

Solution outline. The place No. 1 can be taken as the last one only if place No. 2 is taken as the first one (with the
1
probability of 2012 ) and place No. 2012 right after it. Then some places from 3 to 2011 will be filled until gaps of size
1 and 2 remain. In that moment the probability of place No. 1 to be taken as the last one is equal to the inverse of
number of the places, since every place has equal probability of being taken as the last one. Note that by looking only
at sizes of the gaps and placing cars into the widest ones, we always end up with the same number of gaps of sizes 1
and 2. Thus we can uniquely determine their count.
Let f (n) be the number of places which remain free, if the cars park on n + 2 places in a row with first and last
places already taken. The first car will park in the middle, which splits the task into the cases f (b n−1 n−1
2 c) and f (d 2 e).
n−1 n−1
Thus we have a recurrence relation f (n) = f (b 2 c) + f (d 2 e) with initial conditions f (1) = 1 and f (2) = 2. After
a bit of work and computing values for small n we can derive closed form:
(
x − 2n−1 + 1, 2n ≤ x ≤ 23 · 2n − 2
f (x) = 3
2n , n n
2 · 2 − 1 ≤ x ≤ 2 · 2 − 1.

1 1 1
Thus f (2009) = 1024 and the overall probability is 2012 · 1024+1 = 2062300 .

13
Problem 60. Find all real numbers x that satisfy

(x2 + 3x + 2)(x2 − 2x − 1)(x2 − 7x + 12) + 24 = 0.


√ √
Result. 0, 2, 1 ± 6, 1 ± 8
Solution outline. Since

(x2 + 3x + 2)(x2 − 7x + 12) = (x + 1)(x + 2)(x − 3)(x − 4) =


= (x + 1)(x − 3)(x + 2)(x − 4) = (x2 − 2x − 3)(x2 − 2x − 8),

after substitution x2 − 2x = z we get an equation

0 = (z − 3)(z − 8)(z − 1) + 24 = z 3 − 12z 2 + 35z = z(z − 5)(z − 7).

For each z ∈ {0, 5, 7} we can easily determine the corresponding x.

14
Problem 1J. Given that there is exactly one way to write 2013 as a sum of two primes find the product
of these primes.
Result. 4022
Solution outline. As the sum is odd, one of the primes must be even and therefore equal to 2. Thus
2013 = 2 + 2011 is the only candidate which happens to work as 2011 is indeed a prime. The result then
is 2 · 2011 = 4022.

Problem 2J. Two circles with radius 1 intersect each other. The area of their intersection equals the
sum of the areas of the two outer parts. What is the area of the intersection?

2
Result. 3π

Solution outline. Denote the areas of the three parts as in the figure.

A B C

Combining the obvious A = C (symmetry) with the given B = A + C yields B = 2A. Thus the area of
the intersection is equal to two thirds of the area of the left circle, that is 23 π.

Problem 3J. We have several pins five of which are yellow, four red, three green, two blue, and the
remaining one is orange. In how many ways can we place them in the given triangular grid (see diagram)
so that no pair of pins with the same color is in the same row or column? The pins of the same color are
considered indistinguishable.

Result. 1
Solution outline. First we place the yellow pins and our only option is to place them on the hypotenuse
of the triangle. By the same argument we now have an only option to place the four red pins (again on
the hypotenuse of the triangle formed by empty spots). Continuing with this argument we see that there
is a unique arrangement which satisfies the given conditions.

Problem 4J. Find the smallest positive integer whose product of digits equals 600.
Result. 3558
Solution outline. Since 600 = 23 · 3 · 52 , we may only use the digits 1, 2, 3, 4, 5, 6, 8. Using the digit 1
only increases the number, so we will do better without it. Also, we must use two fives as it is the only
way to get 52 . The product of the remaining digits then must be 24, thus one digit is not enough and
from the pairs 3 · 8 and 4 · 6, the former contains the smallest digit and leads to the solution, which is
3558.

1
Problem 5J. Positive real numbers a and b satisfy

1 1
a+ =7 and b+ = 5.
b a
1
What is the value of ab + ab ?

Result. 33

Solution outline. We multiply the two equations and obtain

a b 1
ab + + + = 35,
a b ab
1
ab + = 33.
ab

Problem 6J. Luke has a six-digit number which satisfies the following conditions:
1. The number reads the same from left to right as from right to left.
2. It is divisible by 9.
3. After crossing out its first and last digit the only prime factor of the resulting number is 11.
What is Luke’s number?

Result. 513315

Solution outline. We start by analyzing the last condition. The only four-digit power of 11 is 1331,
therefore the sought number is of the form a1331a for some digit a. Since it is divisible by 9, so must
be also its sum of digits 2a + 8. Since 2a + 8 is even and less than or equal to 26, the only choice is
2a + 8 = 18, i.e. a = 5 and the answer is 513315.

Problem 7J. Point D lies on the diameter AB of the semicircle k. A perpendicular to AB through
point D intersects k at point C. The lengths of the arcs AC and CB of k are in the ratio 1 : 2. Find
AD : DB.

Result. 1 : 3

Solution outline. Since C trisects the arc AB, we can draw point E ∈ k such that A, C, E, and B (in
this order) are consecutive vertices of a regular hexagon.

C E

A D S B

Now if S is the center of k, triangle ASC is equilateral with D the midpoint of SA (altitudes coincide
with medians in equilateral triangles). Then we easily obtain AD : DB = 1 : 3.

Problem 8J. Two spoilt brothers Jim and Tim got a bag of chocolate chips and split them evenly. Both
of them eat from two to three chocolate chips a day. Jim finished his chips after fourteen days, Tim after
exactly three weeks. How many chocolate chips were there in the bag?

Result. 84

Solution outline. Note that Jim ate at most 3 · 14 = 42 chips while Tim ate at least 2 · 21 = 42. Since
they both started with the same amount of chocolate chips, it must have been 42, and the answer is
42 + 42 = 84.

2
Problem 9J. In how many ways can we read the word Náboj in the diagram?

á á

b b b

o o o o

j j j j j

Result. 16
Solution outline. From each of the letters N, á, b, o we can continue reading in exactly two ways.
Therefore, we can read the word Náboj in 24 = 16 ways.

Problem 10J. Inhabitants of an island are either liars or truth-tellers. Twelve of them sit around a
table. They all claim to be truth-tellers and also claim that the person to their right is a liar. What is
the maximum possible number of liars in such a group?
Result. 6
Solution outline. If two truth-tellers sat next to one another, the one on the left would not call his right
neighbour a liar. For the same reason two liars cannot sit side by side. It remains to check that an
arrangement in which truth-tellers and liars alternate satisfies the condition, making the number 6 our
answer.

Problem 11J / 1S. Jane has 11 congruent square tiles six of which are red, three blue, and two green.
In how many ways can she construct a 3 × 3 square from 9 of the 11 tiles such that the coloring of the
square remains intact if we rotate the square by 90◦ clockwise around its center? Two tiles of the same
color are considered indistinguishable.
Result. 0
Solution outline. For a coloring to be invariant under the 90◦ rotation, its four corner tiles must have the
same color and the same holds for the remaining four non-central tiles. Thus we either need eight tiles
of one color or two quadruplets of tiles with the same color. Since we have neither of that available, the
answer is 0.

Problem 12J / 2S. Two fifths of males and three fifths of females living on a certain island are married.
What percentage of the island’s inhabitants is married?
12
Result. 48% = 25

Solution outline. Denote by D the total number of married couples on the island. Then the total numbers
of males and females are 52 D, 53 D, respectively. Thus, we have 25 D + 53 D = 25
6 D inhabitants from which
2D are married. The fraction of married islanders is then
2D 12
25 = = 48%.
6 D
25

Problem 13J / 3S. What is the maximum side length of an equilateral triangle which can be cut from
a rectangular-shaped paper with dimensions 21 × 29.7 cm?
√ 42
Result. 14 3 = √ 3
cm
Solution outline. Any equilateral triangle lying between a pair of parallel lines can be suitably enlarged
so that two of its vertices lie on opposite borderlines. From all such equilateral triangles the longest side
length is obtained from one with the whole edge on one of the borderlines.
Then if the width of the strip
√ is 21 cm, the √largest fitting equilateral triangle has altitude 21 cm. The
21 cm
side-length is then sin 60◦ = 14 3 cm. Since 14 3 < 14 · 2 < 29.7, this triangle fits inside the rectangle.

3
Problem 14J / 4S. Kate took a square piece of paper and repeatedly folded it in half until it was done
four times. After every fold, she was left with an isosceles right triangle. How many squares can one see
after unfolding the paper?
Result. 10
Solution outline. The fold-lines are captured in the following figure.

Altogether, we see 10 squares: the whole paper, the square connecting the midpoints of the edges and in
each of these two, we see four smaller squares.

Problem 15J / 5S. How many pentagons are there in the figure?

Result. 35 = 243
Solution outline. Observe that each pentagon must have the center of the figure in its interior. We have
three choices for each of the five sides (outer, middle or inner line), thus there are 35 = 243 pentagons.

Problem 16J / 6S. Joseph wanted to add two positive integers but accidentally wrote a zero digit to
the end of one of them. The result he obtained was 3858 instead of the correct 2013. Find the value of
the larger summand.
Result. 1808
Solution outline. Denote the numbers by a, b. Since adding a zero digit is the same as multiplying by
ten, we only have to solve the following system of equations:

a + b = 2013,
10a + b = 3858.

Subtracting them we learn a = 205. Plugging it in the first one we get b = 1808, which is the answer.

Problem 17J / 7S. What is the radius of the smallest circle which can cover a triangle with side lengths
3, 5, and 7?
Result. 3.5
Solution outline. Since 32 + 52 < 72 , the angle opposite the 7-side is obtuse. Thus the triangle can be
covered with a circle of radius 3.5. On the other hand, no smaller circle can cover the longest side, so 3.5
is our final answer.

4
Problem 18J / 8S. Lisa, Mary, Nancy and Susan have 100 lollipops altogether. Each pair of them has
at least 41 lollipops. What is the minimum number of lollipops that Lisa can have?
Result. 12
Solution outline. If we denote the numbers of each girl’s lollipops by L, M , N , and S, respectively, we
know that L + M ≥ 41, L + N ≥ 41, and L + S ≥ 41. Summing these inequalities we get 2L + (L + M +
N + S) ≥ 123. Since L + M + N + S = 100, we learn 2L ≥ 23, or L ≥ 12 (L is a positive integer).
On the other hand, the distribution L = 12, M = N = 29, S = 30 satisfies the conditions of the
problem.

Problem 19J / 9S. The area of a rectangle ABCD is 80 and the length of its diagonal is 16. Find the
sine of the angle between its diagonals.
5
Result. 8 = 0.625
Solution outline. Denote by S the intersection of the diagonals and by h the length of the B-altitude in
△ABC. The area of ABCD can be found as AC · h which gives h = 80/16 = 5. The sought-after value
h
is then sin ∠ASB = sin ∠CSB = BS = 85 .

Problem 20J / 10S. What is the largest possible value of ab + cd , where a, b, c, and d are distinct
elements of the set {−7, −5, −4, −3, −2, −1}?
10
Result. (−1)−4 + (−3)−2 = 9

Solution outline. In order to obtain a positive number, we need to take even exponents. Also, since
(−2)−4 = (−4)−2 < (−1)−2 , we will take the exponents −2 and −4. Further, as a negative power is
decreasing, we will take −1 and −3 as bases. Out of the remaining two options, (−1)−4 + (−3)−2 = 10
9
has greater value.

Problem 21J / 11S. An angle of magnitude 110◦ is placed in the coordinate plane at random. What is
the probability that its legs form a graph of some function?
11
Result. 18

Solution outline. For an angle to form a graph of a function, it must not contain two points with the
same x-coordinate. We may assume that the vertex of the angle is at point (0, 0). We will rotate the
angle by one full circle (clockwise) and determine when it is a graph of a function. We choose to start in
a position in which one leg coincides with the positive ray of the y axis and the other lies to the right.
Observing the breaking point after 70◦ rotation, we see that the answer is 110 11
180 = 18 .

Problem 22J / 12S. James drew a regular 100-gon A1 A2 . . . A100 (numbered clockwise) on the last
year’s Náboj (23rd March, 2012) and randomly placed a chip on one of the vertices. Every following day,
he moved the chip clockwise by the number of vertices indicated by the number of the current vertex
(from A3 he would move it to A6 , from A96 to A92 ). Now the chip lies on A100 . What was the probability
this would happen?
1
Result. 0.04 = 25

Solution outline. We are in fact asking about numbers from 1 to 100 which, when repeatedly multiplied
by 2, become a multiple of 100. Such number must be a multiple of 25 and all four of these (25, 50, 75,
4 1
100) work. The answer is 100 = 0.04 = 25 .

Problem 23J / 13S. A positive integer is called differential if it can be written as a difference of squares
of two integers. How many of the numbers 1, 2, . . . , 2013 are differential?
Result. 1510
Solution outline. Odd numbers can be written as 2k + 1 = (k + 1)2 − k 2 and multiples of four as
4k = (k + 1)2 − (k − 1)2 . Numbers of the form 4k + 2 are not differential since a2 − b2 = (a + b)(a − b)
is a product of two positive integers of the same parity, therefore either odd or multiple of 4. There are
503 numbers of the form 4k + 2 from 1 to 2013, hence the answer is 1510.

5
Problem 24J / 14S. Three non-overlapping regular convex polygons with side length 1 have point A
in common and their union forms a (nonconvex) polygon M which has A in its interior. If one of the
regular polygons is a square and another is a hexagon, find the perimeter of M .

Result. 16

Solution outline. We look at the internal angles of the polygons by vertex A. One of them is 90◦ , another
is 120◦ , and since the three angles add up to 360◦ (A is in the interior), the remaining angle is 150◦ ,
which corresponds to a regular dodecagon (12 vertices). Since each pair of polygons shares a side, the
perimeter of M is 4 + 6 + 12 − (3 · 2) = 16.

Problem 25J / 15S. Mike wrote the numbers from 1 to 100 in random order. What is the probability
that for each i = 1, . . . , 50 the number on the position 2i − 1 is smaller than the number on the position
2i?

Result. 2−50

Solution outline. Imagine Mike writes the numbers in pairs: First he randomly chooses two available
numbers and then their order. The smaller of them going first thus has probability 21 (regardless of the
50
chosen numbers) and thus for 50 pairs the probability is 12 = 2−50 .

Problem 26J / 16S. Pink paint is made up of red and white paint in the ratio 1 : 1, cyan paint is made
up of blue and white in the ratio 1 : 2. Karen intends to paint her room with paint which is made up of
pink and cyan in the ratio 2 : 1. She has already mixed three tins of blue and one tin of red paint. How
many tins does she have to add to the mixture, provided there are only red and white tins left?

Result. 23

Solution outline. Since we can add only red or white paint, the resulting paint has to contain exactly
three tins of blue. Blue is not contained in pink paint, thus the ratio 1 : 2 in cyan implies that there need
to be exactly 9 tins of cyan paint (3 blue and 6 white). Finally, from the ratio 2 : 1 in the desired paint
we infer that it has to be mixed from 27 tins (9 of which are mixed to produce cyan and 18 to pink). As
we have already mixed four tins, we have to add 27 − 4 = 23 tins to complete the paint.

Problem 27J / 17S. We have a hat with several white, gray, and black rabbits. When a magician starts
taking them randomly out of the hat (without returning them), the probability of him taking a white
rabbit before a gray one is 34 . The probability of him taking a gray rabbit before a black one is 43 as well.
What is the probability of him taking a white rabbit before a black one?
9
Result. 10

Solution outline. The magician takes a white rabbit before a gray one with the probability of 34 , hence
the hat contains three times as many white rabbits as the gray ones. In like manner, there are three
times as many gray rabbits as the black ones. Therefore the hat is inhabited by nine times as many white
9
rabbits as the black ones and the sought-after probability is 10 .

Problem 28J / 18S. Positive integers a, b satisfy 49a + 99b = 2013. Find the value of a + b.

Result. 37

Solution outline. We add a + b to both sides of the equation and get 50(a + 2b) = 2013 + (a + b). The
left-hand side of the equation is divisible by 50, and so must be the right-hand side. It follows that
a + b = 50k − 13 for some k ∈ N.
If a + b ≥ 87, then 49a + 99b > 49a + 49b ≥ 49 · 87 > 2013, which is not possible. Thus the only
possibility is a + b = 37.

6
Problem 29J / 19S. In the corners of a square P QRS with side length 6 cm four smaller squares are
placed with side lengths 2 cm. Let us denote their vertices W , X, Y , Z like in the picture. A square
ABCD is constructed in such a way, that points W , X, Y , Z lie inside the sides AB, BC, CD, DA
respectively. Find the largest possible distance between points P and D.

S 2 2 2 R
D 2
Z Y
A
2
C
W X
B 2

P Q

Result. 6

Solution outline. Point D belongs to the circle with diameter ZY , the


√ center of which we denote by O.
This circle has radius 1 and by Pythagorean Theorem we have P O = 32 + 42 = 5. Hence from triangle
inequality we obtain P D ≤ P O + OD = 6. The equality is attained if P , O, D lie on one line, so the
maximum distance is 6.
S 2 2 2 R
D 2
Z Y
O
2
4
2

P 3 Q

Problem 30J / 20S. We have twenty boxes of apples that altogether contain 129 apples. We know that
in some of the boxes there are exactly 4 apples and all the remaining boxes contain exactly x apples.
Find all possible values of x.

Result. 11, 53

Solution outline. Denote by K the number of boxes that contain x apples (K ≤ 20). In the remaining
20 − K boxes we have four apples and hence

K · x + (20 − K) · 4 = 129, which implies K(x − 4) = 49.

We have K ≤ 20, hence the only possible choices are K = 1 and K = 7. From K = 1 we easily obtain
x = 53, and K = 7 corresponds to x = 11.

Problem 31J / 21S. Consider real numbers a, b such that a > b > 0 and

a 2 + b2
= 2013.
ab
a+b
Find the value of the expression a−b .
q
2015
Result. 2011

7
Solution outline. From the given equality we obtain a2 + b2 = 2013ab. Using this identity we have

(a + b)2 = a2 + 2ab + b2 = 2015ab,


(a − b)2 = a2 − 2ab + b2 = 2011ab.

a+b
From a > b > 0 we know that the expression a−b is positive and hence using the two relations above
yields
s r r
a+b (a + b)2 2015ab 2015
= = = .
a−b (a − b)2 2011ab 2011

Problem 32J / 22S. In how many ways can we fill in individual squares in the pictured heptomino piece
with numbers 1 to 7 (each of which should be used once only) so that the sum of numbers in the bottom
row is the same as the sum of numbers in the left column?

Result. 144 = 3 · 2 · 4!
Solution outline. Denote values in the individual squares as in the picture.

C D E F G

It follows from the problem statement that A + B + C + D + E + F + G = 1 + 2 + 3 + 4 + 5 + 6 + 7 = 28


and A + B = D + E + F + G = 12 (28 − C). Therefore C must be even.
If C = 2, then A + B = 13 = 6 + 7, which gives two alternatives how to fill in the squares in the left
column. For each of them we can reorder the numbers in the bottom row in 4! ways. If C = 4, then
A + B = 12 = 5 + 7 (6 + 6 is not legitimate since identical numbers are disallowed), and if C = 6, then
A + B = 11 = 4 + 7 (5 + 6 is also illegitimate since C = 6 already), which gives in both cases 2 · 4!
possibilities of finishing the table. Hence we obtain altogether 3 · 2 · 4! = 144 possibilities.

Problem 33J / 23S. Let ABC be an acute triangle with AB = 4π, BC = 4π + 3, CA = 4π + 6. Denote
by D the foot of the A-altitude. Find CD − BD.
Result. 12
Solution outline. Pythagorean Theorem for right triangles ADC and ADB gives CD2 = AC 2 − AD2
and BD2 = AB 2 − AD2 . After subtracting, we obtain

CD2 − BD2 = AC 2 − AB 2 = (4π + 6)2 − (4π)2 = 48π + 36.

Since D lies on the segment BC, we also have

CD2 − BD2 = (CD − BD) · (CD + BD) = (CD − BD) · (4π + 3),

hence CD − BD = 12.

8
Problem 34J / 24S. Trams have the same intervals all day and in both directions. A pedestrian walked
along the tram rails and observed that every 12 minutes he was overtaken by a tram, and every 4 minutes
one tram coming in the opposite direction passed him. What is the time interval between two trams?
Result. 6 minutes
Solution outline. We denote the speed of the tram by t, the speed of the pedestrian by c and the distance
between two trams by d. From the given information we deduce that
d
t−c= ,
12 min
d
t+c= .
4 min
By summing up the equations and dividing by 2 we obtain
 
1 1 1 d
v= + ·d= .
2 12 min 4 min 6 min
It follows that the tram travels the distance d in 6 minutes, which implies that the interval between two
trams is 6 minutes.

Problem 35J / 25S. How many nondegenerate triangles can be formed by means of joining some point
triplet from the picture?

Note: Points are aligned in the indicated grid.


Result. 148 = 11

3 − 17
Solution outline. We can pick a point triplet out of the given eleven ones in
 
11 11 · 10 · 9
= = 165
3 1·2·3
ways. It remains to compute the number of point triplets lying on a line and to subtract the resulting
number from the one above. There are 4 + 1 + 4 = 9 horizontal triplets, 3 + 3 + 1 + 1 = 8 transversal
triplets, hence the number of triangles is 165 − 17 = 148.

Problem 36J / 26S. Thomas got a box of chocolates with 30 sweets arranged in three rows by ten. He
is eating the sweets one by one in such a way that the number of sweets in two rows always differs at
most by one. In how many ways can he eat the whole box?
Result. 610 · (10!)3
Solution outline. The way of eating the sweets can be uniquely determined in the following way: For
each row we choose the order of its sweets in which Thomas will eat them and also choose the order of
the rows in which he will switch between them.
For each row, we have 10! possibilities to choose the order of sweets, which gives us (10!)3 possibilities
for three rows.
Now we need to count the number of the order of rows. We know that if there is the same number of
sweets in all rows, then Thomas can choose any of them, then he needs to choose one of the remaining
two (choosing the same row would result in having two sweets less in this row) and then he has to choose
the remaining one. After three steps he has again the same number of sweets in all three rows. It follows
that it is enough to choose ten times the order of the three rows, which can be done in (3!)10 = 610
different ways.
Thomas can eat the box of chocolates in 610 · (10!)3 different ways.

9
Problem 37J / 27S. We call a positive integer with six digits doubling if the first three digits are the
same (including the order) as the other three digits. For example the number 227227 is doubling, but
135153 is not doubling. How many six-digit doubling numbers are divisible by 2013?
Note: The first digit of a positive integer cannot be 0.
Result. 5
Solution outline. Any doubling number with six digits can be written as 1001 · k for some positive integer
k. Conversely, for any three digit positive integer k the formula 1001 · k gives us a six-digit doubling
number.
As 2013 = 3 · 11 · 61 and 1001 is divisible by 11, but not by 3 or 61, it follows that the six digit integer
is doubling if and only if the corresponding three digit k in 1001 · k is a multiple of 3 · 61 = 183. We
have exactly five three digit k divisible by 183 and hence we have five six-digit doubling numbers that
are divisible by 2013.

Problem 38J / 28S. Suppose that we have 4 × 4 chessboard and on each square we draw an arrow
aiming right or aiming down at random. We put a robot on the upper left corner and let it move along
the chessboard in the directions of the arrows. What is the probability that it will exit the chessboard
from the square in the lower right corner?
5 (63)
Result. 16 = 26

Solution outline. Let us count the number of paths from the upper left to the lower right corner and
the probability that the robot willgo along one such path. Each path consists of three steps down and
three steps right which gives us 63 possible paths. The probability that the robot will follow this path
−6
is always 2 as in each of the steps he must choose the correct direction. The overall probability is thus
−6 6
2 · 3 .

Problem 39J / 29S. Write


212121210
112121211
in the basic form (i.e. as the fraction ab , where a and b are positive integers with no common divisor).
70
Result. 37

Solution outline. It is easy to see (as the sum of the digits is divisible by three) that both numerator
and denominator are divisible by three. By division we get 212121210 = 3 · 70707070 and 112121211 =
3 · 37373737. Now observe that 70707070 = 70 · 1010101 and 37373737 = 37 · 1010101, which implies that
the basic form of our fraction is 70
37 .

Problem 40J / 30S. Let ABCD be a rectangle with side lengths AB = 30 and BC = 20. For how
many points X on its side AB is the perimeter of the triangle CDX an integer?
Result. 13
Solution outline. It is sufficient to find when is DX + XC an integer. Let us consider C ′ , D′ such that A,
B are midpoints of DD′ , CC ′ respectively. Then DX + XC = DX + XC ′ . The right-hand side attains
its minimum when X is the midpoint of AB and maximum when X is equal to A or B.

D 30 C

20

A X B

20

D′ C′

10
√ √ √
Using Pythagorean Theorem we compute DC ′ = 302 + 402 = 50 and AC ′ = 302 + 202 = 1300,
from which we have 36 < AC ′ < 37. So, if X moves from A to B, then DX + XC first decreases from a
number a little larger than 20 + 36 = 56 (X = A) to 50 (X is the midpoint of AB) and then increases to
a number a little larger than 56 (X = B). Hence, it is integer for 6 + 1 + 6 = 13 positions of the point X.

Problem 41J / 31S. In what order should we place the rows r1 , . . . , r11 of the table in the picture, if
we want the newly created table to be symmetric with respect to the marked diagonal? It is sufficient to
find one solution.

Result. in the reverse order (eventually complemented by a circular shift), i.e. r11 , r10 , r9 , . . . , r1 or
r10 , r9 , . . . , r1 , r11 , . . . , r1 , r11 , r10 , . . . , r2

Solution outline. Observe that the table is symmetric with respect to the other diagonal. To make it
symmetric with respect to the marked diagonal it is sufficient to reflect it over the horizontal axis.
Remark: For this table there are no solutions except the eleven mentioned above.

Problem 42J / 32S. For every positive integer n let

1p3
an = n3 + n2 − n − 1.
n
Find the smallest integer k ≥ 2 such that a2 · a3 · · · ak > 4.

Result. 254

Solution outline. Denoting An = a3n our problem is equivalent to finding the smallest integer k ≥ 2 such
that A2 · A3 · · · Ak > 43 = 64. We have

n3 + n2 − n − 1 (n + 1) · (n + 1) · (n − 1)
An = =
n3 n·n·n
and hence

3·3·1 4·4·2 (k + 1) · (k + 1) · (k − 1) 1 · (k + 1) · (k + 1) (k + 1)2


A2 · A3 · · · Ak = · ··· = = .
2·2·2 3·3·3 k·k·k 2·2·k 4k
It remains to solve the inequality
(k + 1)2 > 256k

for integer k. Substracting 2k from both sides and multiplying them by 1/k yields equivalent inequality
k + k1 > 254, the smallest integer solution of which is k = 254.

Problem 43J / 33S. Barbara cut the pizza into n equal slices and then she labeled them with numbers
1, 2, . . . , n (she used each number exactly once). The numbering had the property that between each two
slices with consecutive numbers (i and i + 1) there was always the same number of other slices. Then
came the fatty Paul and ate almost the whole pizza, only the three neighboring slices with numbers 11,
4, and 17 (in this exact order) remained. How many slices did the pizza have?

Result. 20

11
Solution outline. Suppose that between the slices with consecutive numbers we have exactly k − 1 other
slices, that is by moving by k slices we get from slice with number 1 to slice with number 2, from slice 2
to slice 3 and so on. All these movements have to be in the same direction, otherwise we would get from
slice i to the previous slice i − 1 and not to i + 1. From the slice n we get in this way necessarily to slice
number 1, because any other i has in distance k slices with numbers i + 1 and i − 1. By moving with the
step k slices we will finally pass through the whole pizza. Hence there is s such that moving s times by
k slices we will end exactly at the neighboring slice. Thus we get

11 − 4 ≡ s · k ≡ 4 − 17 (mod n),

which implies that 7 − (−13) = 20 is divisible by n. There is a slice with number 17 and hence n ≥ 17
which implies that the only solution is n = 20.

Problem 44J / 34S. In one of the lecture halls at Matfyz the seats are arranged in a rectangular grid.
During the lecture of analysis there were exactly 11 boys in each row and exactly 3 girls in each column.
Moreover, two seats were empty. What is the smallest possible number of the seats in the lecture hall?

Result. 144

Solution outline. We denote by r and s the number of rows and columns in the lecture hall. From the
instructions we have rs = 11r + 3s + 2, which is equivalent to

(r − 3)(s − 11) = 35.

The numbers in the brackets are either 5 and 7, or 1 and 35 in some order. By writing down all four
possibilities we can find that the smallest number rs corresponds to the case r − 3 = 5, s − 11 = 7, when
rs = 8 · 18 = 144. It remains to show that we can really put the students in the lecture hall as requested,
which can be seen in the picture.

Problem 45J / 35S. Circle k with radius 3 is internally tangent to circle l with radius 4 at point T .
Find the greatest possible area of triangle T KL, where K ∈ k and L ∈ l.
√ 27
Result. 9 3 = √ 3

Solution outline. Denote by [XY Z] the area of triangle XY Z.


Denote by M the intersection of T L with k (M 6= T ). Since T is the center of homothety with factor
4
3 sending k to l, points L and M correspond and thus T L = 34 T M and [T KL] = 34 [T KM ] (the triangles
share an altitude from K). Hence it suffices to maximize the area of T KM inscribed in circle k with
radius 3. From all such triangles the equilateral one has the greatest area, namely
  √
1 27 3
3· · 3 · 3 sin 120◦ = .
2 4

Hence the area of the corresponding triangle T KL is



4 27 3 √
· = 9 3.
3 4

12
L
M

T
k
l
K

Problem 46J / 36S. Alice and Bob are playing the following game: At the beginning they have a set of
numbers {0, 1, . . . , 1024}. First Alice removes any 29 elements, then Bob removes any of the remaining
28 elements, then Alice removes 27 elements and so on until at last Bob removes one element and there
are exactly two numbers remaining. The game ends then and Alice pays to Bob the absolute value of
the difference of these two numbers in Czech crowns. How many crowns will Bob get if they both play
in the best way they can?
Result. 32
Solution outline. Bob can double the smallest distance between any two numbers in each step by removing
every other element. In this way he wins at least 25 = 32 crowns. Alice can halve the difference of the
largest and the smallest number in each step by removing upper half (or lower half) of all remaining
numbers. This ensures that she loses at most 1024/25 = 32 crowns. Therefore if they both play in the
best way, Bob will get 32 crowns.

Problem 47J / 37S. Some students of Matfyz did not pass the exams and were expelled. They all
started to study at SOU (some other university). It had the following consequences:
1. The number of students of Matfyz decreased by one sixth.
2. The number of students of SOU increased by one third.
3. The average IQ on both schools increased by 2%.
How many times was the average IQ at Matfyz higher than average IQ at SOU?
6
Result. 5 = 1.2-times
Solution outline. We denote by 100m the original average IQ of students at Matfyz and 100v the original
average IQ of students at SOU. Further, we denote by p the average IQ of students who changed from
Matfyz to SOU. The average IQ at Matfyz increased by 2% and thus the average IQ of remaining students
at Matfyz is 102m. From the ratio 5 : 1 of remaining students and from the original average 100m we
obtain the equation 100m = 65 · 102m + 16 p, which is the same as p = 90m. Analogously the average IQ
at SOU was originally 100v and with the new students it has risen to 102v. From the ratio 3 : 1 we have
the equation 102v = 34 · 100v + 41 p, which implies p = 108v. Altogether this gives us
m 108 6
90m = 108v, and hence = = .
v 90 5

Problem 48J / 38S. A regular tetradecagon A1 A2 . . . A14 is inscribed in a circle k of radius 1. How
large is the part of the disc circumscribed by the circle k, which lies inside the angle ∠A1 A4 A14 ?
π
Result. 14
Solution outline. Let us focus on points A1 , A4 , A14 and A11 .
A14 A1
A13 A2

A12 A3

A11 O A4

13
Since 11 = 4 + 7, the line segment A4 A11 is a diameter of the circle k. On the other hand we have
4 − 1 = 3 = 14 − 11, so A1 A4 A11 A14 is an isosceles trapezoid and its bases A4 A11 and A1 A14 are parallel.
The area of the triangle A1 A4 A14 is therefore the same as the area of the triangle A1 OA14 , where O is
the center of k. The area to compute is then equal to the area of the sector A1 OA14 , i.e. one fourteenth
of the area of the disc.

Problem 49J / 39S. Bill and Carol saw the 24-element set {1, 2, . . . , 24}. Bill wrote down all its 12-
element subsets whose sum of the elements was even. Carol wrote down all its 12-element subsets whose
sum of the elements was odd. Who wrote more subsets and by how many?
Result. Bill, 12

6 = 924 subsets more

Solution outline. Consider an arbitrary 12-element subset B and assume there exists an integer i such
that B contains exactly one of the numbers 2i − 1, 2i. Let us take the smallest such i and construct a
12-element set f (B), which contains the same elements as B with the only exception: It contains the
other element from the couple 2i − 1, 2i.
It is easy to see that f (f (B)) = B and that by applying f on a Bill’s subset we obtain Carol’s subset
and vice versa. So, f is a bijection between Bill’s and Carol’s subsets, if we consider just those subsets
for which there exists an i from the previous paragraph. It remains to count the remaining subsets, for
which such i does not exist.
In each of the remaining subsets, there must be exactly six odd numbers and six even numbers which
are successors of the odd ones. Sum of the elements of such subsets is always even (hence they belong to
Bill’s list) and the number of such subsets is 126 .

Problem 50J / 40S. Jack thought of three distinct positive integers a, b, c such that the sum of two of
them was 800. When he wrote numbers a, b, c, a + b − c, a + c − b, b + c − a and a + b + c on a sheet
of paper, he realized that all of them were primes. Determine the difference between the largest and the
smallest number on Jack’s paper.
Result. 1594
Solution outline. Let us assume without loss of generality that b + c = 800. At least one of the numbers
a, b + c − a = 800 − a, a + b + c = 800 + a is divisible by three, so it can be prime number only if it is
equal to three. Since 800 + a > 800, there are only two possibilities, a = 3 or 800 − a = 3.
If a = 3, then we have 3 + (b − c) ≥ 2 and simultaneously 3 − (b − c) ≥ 2, so |b − c| ≤ 1. This is
impossible, since b + c = 800.
So, we know that 800 − a = 3, i.e. a = 797. The largest among Jack’s prime numbers is a + b + c =
797 + 800 = 1597. Since b + c = 800, none of the Jack’s primes is even. Therefore, the smallest number
is 800 − a = 3. The difference is then 1597 − 3 = 1594.
Let us remark that such numbers exist, we can take a = 797, b = 223, c = 577.

Problem 51J / 41S. Helen stained two randomly chosen places on a one-meter bar. Thereupon Alex
came and shattered the bar, just as well randomly, into 2013 pieces. What is the probability of both
stains being now on the same piece?
1
Result. 1007

Solution outline. Imagine the bar is in one piece. Helen marked it randomly with two stains, whereas
Alex marked it randomly with 2012 breaks. The bar thus contains altogether 2014marks, two of which
are stains. The total number of possibilities of which marks can be stains is 2014
2 = 1007 · 2013. The
stains are on the same piece (or shard) if and only if they are neighbouring marks, which occurs in 2013
cases. The resultant probability is
2013 1
= .
1007 · 2013 1007

Problem 52J / 42S. How many ten-digit positive integers that are made up of 0, 1, . . . , 9 (i.e. each digit
is used exactly once) are multiples of 11111?
Note: The first digit of a positive integer cannot be 0.
Result. 3456 = 25 · 5! − 24 · 4!

14
Solution outline. Since 0 + 1 + · · · + 9 = 9 · 5, the numbers under investigation must be divisible by
nine, hence even by 99999. Let us denote A and B the numbers consisting of the former and the latter
five-tuple of the investigated number respectively. We have
99999 | 100000A + B, iff 99999 | A + B.
Given that A, B are five-digit positive integers less than 99999, we observe
0 < A + B < 2 · 99999, hence A + B = 99999, or equally B = 99999 − A.
Therefrom we obtain the necessary and sufficient condition on A, B for divisibility of the investigated
ten-digit number by 99999: For i = 1, . . . , 5, the i-th digit of B is a complement of i-th digit of A into
nine. We couple the available digits into five pairs
(0, 9), (1, 8), (2, 7), (3, 6), (4, 5).
We know that these pairs must be used in certain order (5! options) and at the same time, in each pair
we may choose, which digit will be put in A and which in B (25 options). However, we have to remove
the choices including a zero as the first digit in A – that is 4! options of redistributing the remaining
pairs and 24 options of redistributing their digits between A and B. The amount of desired numbers is
thus 5! · 25 − 4! · 24 .

Problem 53J / 43S. A polynomial P (x) of degree 2013 with real coefficients fulfills for n = 0, 1, . . . , 2013
the relation P (n) = 3n . Evaluate P (2014).
Result. 32014 − 22014
P2013 
Solution outline. Define the polynomial Q(x) = k=0 xk 2k . Its degree is 2013 and moreover, for any
x ∈ {0, . . . , 2013} the binomial theorem yields
2013
X x  x  
k
X x k
Q(x) = 2 = 2 = (1 + 2)x = P (x).
k k
k=0 k=0

The polynomial P (x) − Q(x) is of degree 2013 and it possesses 2014 roots, whereby it must be zero.
Hence P (x) = Q(x) and it only remains to compute
2013
X 2014 2014
X 2014  
k k 2014 2014
Q(2014) = 2 = 2 − 2 = (1 + 2)2014 − 22014 = 32014 − 22014 .
k k 2014
k=0 k=0

Problem 54J / 44S. Inside an isosceles triangle ABC fulfilling AB = AC and ∠BAC = 99.4◦ , a point
D is given such that AD = DB and ∠BAD = 19.7◦ . Compute ∠BDC.
Result. 149.1◦
Solution outline. Denote by E the image of B in reflection over AD.
A

60◦

D
30◦
B C

E
Then AE = AB = AC and ∠EAC = ∠BAC − 2 · ∠BAD = 60◦ , entailing that the triangle AEC is
equilateral and CE = CA. In addition, DE = DB = DA due to symmetry, and therefore CD is the
perpendicular bisector of AE and ∠ACD = 12 ∠ACE = 30◦ . Now it is simple to use the nonconvex
quadrilateral ABDC for computing
∠BDC = ∠DBA + ∠BAC + ∠ACD = 19.7◦ + 99.4◦ + 30◦ = 149.1◦ .

15
Problem 55J / 45S. Find the largest positive integer not ending with a zero such that removal of one
of its “inner” digits produces its divisor.
Note: An “inner” digit denotes any digit except for the first and the last one.
Result. 180625
Solution outline. Let X be the sought-after number. First we deduce the digit to be removed must be
the second one.
Proceeding by contradiction, assume the two initial digits have remained. Out of an n-digit number
X, the removal produces an (n − 1)-digit number X ′ . The number 10 · X ′ has then n digits again, the
two initial ones being the same as in X, yet X 6= X ′ because the former does not end with a zero. This
is a contradiction – subtracting two multiples of an (n − 1)-digit number cannot produce an (n − 2)-digit
number.
Now we write X = a · 10n+1 + b · 10n + c, where a and b are digits (a 6= 0) and c < 10n is a number with
a nonzero terminal digit. Removal of the second digit produces X ′ = a · 10n + c. Hence, for a suitable
k ∈ N, it follows that
a · 10n+1 + b · 10n + c = k · (a · 10n + c).
At this point we observe k < 20. Indeed, provided k ≥ 20, the number X would have a greater initial
digit than X ′ , which is impossible. Let us modify the equality into

10n (10a + b − k · a) = (k − 1)c.

Given that the left-hand side is divisible both by 2n and 5n , the right-hand side must share the same
property. The number c does not end with a zero, hence k − 1 has to be divisible by at least one of the
prime numbers 2, 5 in their full power. Seeing that k < 20, we conclude n ≤ 4 (by reason of 25 > 20
and even 52 > 20), resulting in X having at most 6 digits. On the other hand, for n = 4 it must be
k − 1 = 16, yielding
54 (b − 7a) = c.
In order for the right-hand side to be nonnegative, the only option is a = 1 (a and b are digits). As for
b, we may choose b = 8, b = 9, the latter of which we disregard for c would end with a zero. With b = 8,
we calculate c = 54 = 625 and finally verify that X = 1 · 105 + 8 · 104 + 625 = 180625 really is a solution
to the problem.

Problem 56J / 46S. Three mutually distinct real numbers a, b, c satisfy

a = (b − 2)c, b = (c − 2)a, c = (a − 2)b.

Compute the product abc.


Result. 3
Solution outline. If any of a, b, c is zero, then all the numbers are zero, which contradicts their mutual
distinctness. Similarly, the numbers a, b, c cannot be three.
We use the third equality to express c in the first and the second equation and then we modify the
second expression b = (ab − 2b − 2)a into b(a2 − 2a − 1) = 2a. Since the right-hand side is nonzero, the
same must hold also on the left; it is therefore legitimate to divide by a2 − 2a − 1 and hence to express b
by means of a. We insert the result into the first expression. After some modifications we obtain

a(a − 3)(a3 + 3a2 − 9a − 3) = 0,

hence a is a root of the polynomial P (x) = x3 + 3x2 − 9x − 3. By analogy, we derive that b and c are roots
of the same polynomial as well. Since a, b, c are mutually distinct, we infer P (x) = (x − a)(x − b)(x − c).
Comparing coefficients of the absolute term, we get abc = 3.

Problem 57J / 47S. In a scalene triangle ABC, there is an altitude of the same length as one median
and another altitude of the same length as another median. In what ratio are the lengths of the third
altitude and the third median?
2
Result. 7

16
Solution outline. Without loss of generality assume a > b > c. The corresponding altitudes and medians
then satisfy va < vb < vc and ta < tb < tc respectively. At the same time, va < ta , vb < tb and vc < tc ,
which implies vb = ta and vc = tb .
Let us denote by M the midpoint of the side BC and by M0 the projection of M to the side AC. In
the right triangle AM M0 we observe M M0 = 12 vb = 12 ta = 21 AM , hence ∠M AC = 30◦ . Denoting by N
the midpoint of the side AC, we obtain similarly ∠N BA = 30◦ .

A1

A
A
◦ M0
30 N 30◦

30 G B G N
B M C C

C1

Now let G be the centroid of triangle ABC. Consider the equilateral triangle A1 BC1 with BN
being one of its medians. The point A1 fulfills ∠N BA1 = 30◦ as well as ∠GA1 N = 30◦ and yet it
is distinct from the point A (the triangle ABC is scalene). “The real” point A must be therefore the
other intersection of the ray BA1 with the arc GA1 N , that is, the midpoint of the line segment BA1 .
Consequently ∠BAC = 120◦ and AC : AB = 2.

In a triangle with an angle √ α = 120 and side √ lengths AB p = 1, AC = 2 1we √ employ the Law
of Cosines to calculate a = 12 + 1 · 2 + 22 = 7 and tc = 1/4 + 1 + 4 = 2 21, then the area
√ √
S = 12 · 1 · 2 · sin 120◦ = 21 3 and finally va = 2S/a = 3/ 21. Putting everything together we thus obtain

√3
va 2
= 1
√21 = .
tc 2 21
7

17
Problem 1J. Peter has a gold bar of size 2 × 3 × 4. Being an amateur blacksmith, he melted down the
bar and created three identical cubes from the liquid gold. What is the common sidelength of Peter’s
cubes?
Result. 2
Solution outline. The volume of the original blockstone was 2 · 3 · 4 = 24. Since all three cubes have the
same volume, it equals 243 = 8. As the volume of√ a cube is the third power of its sidelength, we conclude
that the sidelength of the three golden cubes is 3 8 = 2.

Problem 2J. A new Year’s Eve celebration was attended by 43 people. The bar was selling juice, beer,
and champagne. During the night 25 people drank beer, 19 people drank champagne, and 12 people
drank both beer and champagne. The others were drivers, so they drank only juice. How many people
drank only juice?
Result. 11
Solution outline. The total number of people drinking alcohol was 25 + 19 − 12 = 32 (we added the
numbers of people drinking beer and people drinking champagne and subtracted the number of those
drinking both beverages, since we counted them twice). All the others drank juice only, so there were
43 − 32 = 11 such people.

Problem 3J. By drinking a cup of black tea, one gets enough caffeine for one hour. By drinking a cup
of coffee, one gets caffeine for 4 hours. In what ratio should we mix black tea and coffee in order to have
a full cup containing caffeine for two hours?
Result. 2 : 1
Solution outline. Denote by k the amount of caffeine for one hour. Then black tea contains k, coffee
contains 4k and our goal is to mix 2k into one cup. If we fill a fraction x of a cup with tea and the remainder
1 − x with coffee into one cup, the amount of caffeine in the mixture satisfies x · k + (1 − x) · 4k = 2k.
This yields x = 2/3 and hence the desired ratio is
2
x
= 3 2 = 2 : 1.
1−x 1− 3

Problem 4J. The mirrors v and h in the picture are perpendicular to each other. The angle between
the mirrors h and s is 75◦ . A light ray was incident on the mirror v, from which it reflected to h, then
to s, and finally back to v again. The initial angle of incidence on v was 50◦ . What was the angle of
incidence on v after reflecting from s?
Note: The angle of reflection equals the angle of incidence.

v s
50◦

75◦
h

Result. 80◦
Solution outline. We sketch the trajectory of the light ray into a picture and compute angles in the
resulting triangles. The angle of the first reflection from v is 50◦ from the last assumption. We know that
the sum of angles in a triangle is 180◦ and v is perpendicular to h. Therefore the angle of incidence on h
was 40◦ , as was the angle of reflection. By the same reasoning, we infer that the angle of reflection from s
was 65◦ . Next we recall that the sum of angles in a quadrilateral is 360◦ and hence compute the desired
value, i.e. 360◦ − 90◦ − 75◦ − (180◦ − 65◦ ) = 80◦ .

1
80◦

80◦ 65◦
v 50◦ s
50 ◦ 65◦

50◦
40◦ 40◦ 75◦
h

Problem 5J. A shop sold 235 intergalactic spaceships during the year 2013. In each month, 20, 16, or
25 spaceships were sold. Determine in how many months the shop sold exactly 20, 16, and 25 spaceships,
respectively.
Result. 4, 5, 3
Solution outline. Let a, b and c be the number of months in which 20, 16 and 25 spacecrafts were sold,
respectively. Then 20a + 16b + 25c = 235. The right hand side is divisible by 5, so the left hand side has
to be divisible by 5, too. Clearly, 20a + 25c is a multiple of 5, hence 16b has to be a multiple of 5 as well.
Since 5 - 16, necessarily 5 | b. There are only three possibilities:
• b = 0. Then c = 12 − a and we get 20a + 25(12 − a) = 235 or 4a + 60 − 5a = 47. Thus a = 13,
which is not possible as there are only twelve months altogether.
• b = 5. Then a = 7 − c and substitution gives 80 + 5c = 95. The solution is a = 4, c = 3.
• b = 10. Then c = 2 − a and 210 − 5a = 235, which is not possible.

Problem 6J. Two circles have radii 5 and 26. The centre of the larger circle lies on the circumference of
the smaller one. Consider the longest and the shortest chord of the larger circle which are tangent to the
smaller circle. What is the difference between the lengths of these extremal chords?
Result. 4
Solution outline. The longest chord is clearly the diameter of the larger circle, so its length is 52. The
shortest is the one whose distance from the centre of the larger circle is the greatest. This distance can
be at most 10, because the chord has √ to be tangent to the smaller circle. By the Pythagorean theorem,
the length of the shortest chord is 2 · 262 − 102 = 48, so the desired difference is 52 − 48 = 4.

26

10

Problem 7J. Collin Farrel was born in the last century in Sleepy Hollow. We know that his age in 1999
was the same as the sum of the digits of the year when he was born. What is Collin’s year of birth?
Result. 1976
Solution outline. Let 10x + y be his age in 1999, where x, y are digits. Since he was born in the 20th
century, the sum of the digits of his year of birth is 1 + 9 + (9 − x) + (9 − y), therefore 28 − x − y = 10x + y or
28 = 11x + 2y. Since 0 ≤ x, y ≤ 9, the only solution is x = 2, y = 3, hence he was born in 1999 − 23 = 1976.

2
Problem 8J. Four circles of radius 1 are pairwise tangent (except for the top and the bottom one) as in
the figure below. A tight band is wrapped around them, as shown in the figure. What is the length of
the band?

Result. 8 + 2π
Solution outline. We can split the band into straight and curved parts. The curved parts together form a
full turn, so their total length is 2π. The length of each straight part is the same as the distance of the
centers of the corresponding circles, which is 2. Thus the overall length is 8 + 2π.

Problem 9J. A certain sports teacher always orders his class of bugs by the number of their legs. In
the last sports session, only Buggy and four other bugs, who have 6, 3, 10, and 9 legs, were present. We
know that when they stood in the correct order, the number of legs of the middle bug was the arithmetic
mean of the numbers of legs of all five bugs. Find all possible numbers of Buggy’s legs.
Result. 2, 7, 17
Solution outline. Let m be the number of legs of the middle bug, and let b be number of Buggy’s legs.
Then we have
6 + 3 + 10 + 9 + b 3+b
m= =5+
5 5
or b = 5m − 28. As m is also the median (i.e. the “middle value”), it has to be one of 6, 9, or b, so (by
plugging into the last equation) we find that b is one of 2, 7, or 17.

Problem 10J. For non-zero digits X and Y , denote XY the number whose decimal representation is
XY . Assuming non-zero digits A, B, C satisfy AA + BB + CC = ABC, determine ABC.
Result. 198
Solution outline. Comparison of the units digit in the equality yields A + B + C = C + f or A + B = f ,
where f is 0 or 10; as A, B are non-zero digits, only A + B = 10 is possible. Similarly, the tens digit
gives A + B + C + 1 = B + 10 (the one on the left hand side is carried from units) or A + C = 9. Finally,
since we again carry only 1 to the hundreds, we obtain A = 1, thus B = 9 and C = 8.

Problem 11J / 1S. The famous celestial object collector Buggo was forced to sell one third of his
collection due to the financial crisis. Immediately afterwards he gave three of his Solar System planets to
his daughter. Later, he sold one third of the rest of his collection, and, furthermore, he also gave two
moons of Jupiter and two moons of Saturn to his wife. Finally he sold one third of the rest of his objects
followed by Mars with both its moons, and after that there were only 9 planets in the Alpha Centauri
system left in his collection. How many objects did Buggo have at the beginning?
Result. 54
Solution outline. Let x be the number of celestial objects at the beginning. The number of objects
changes during the process in the following way:

x, 32 x, 23 x − 3, . . . , 23 23 32 x − 3 − 4 − 3 = 9.
 

After a bit of manipulation we obtain


3 3 3
 
x= 2 2 2 (9 + 3) + 4 + 3 = 54.

3
Problem 12J / 2S. Find the least positive integer greater than 2014 that cannot be written as a sum of
two palindromes.
Note: A palindrome is a number whose digits (in decimal representation) are the same read backward.
Result. 2019
Solution outline. If a number greater than 2014 can be written as a sum of two palindromes, then
one of these palindromes needs to be at least 4 digits long. However, there are not many of these –
namely 1001, 1111, . . . , 1991, 2002 (and the rest is too large). We can write the numbers up to 2018 as
2015 = 1551 + 464, 2016 = 1441 + 575, 2017 = 1331 + 686 and 2018 = 1221 + 797.
Assume that 2019 can be decomposed in such a fashion; then one of the summands must have four
digits. If the second summand had an even number of digits, their sum would be a multiple of 11, whereas
2019 is not. Thus we have 2019 = 1AA1 + BCB, where A, B and C are digits. From the units digit
we see that B = 8, so there is no carrying and either A + C = 1 or A + C = 11. The former would
imply 1AA1 + 8C8 < 2000, the latter A + 8 + 1 = 10 (hundreds digit), so A = 1 and C = 10 which is a
contradiction. Therefore the number we are looking for is 2019.

Problem 13J / 3S. Vodka gave Ondro a number puzzle. He chose a digit X and said: “I am thinking
of a three digit number that is divisible by 11. The hundreds digit is X and the tens digit is 3. Find the
units digit.” Ondro soon realised that he had been cheated and there is no such number. Which digit X
did Vodka choose?
Result. 4
Solution outline. Taking into account the well-known rule for divisibility by 11 the problem simplifies to
determining for which digit X there is no digit Y (the units digit) such that (X + Y ) − 3 is divisible by
11. This further reduces to seeking X such that 0 < X + Y − 3 < 11 for all Y , which is satisfied only for
X = 4.

Problem 14J / 4S. The real numbers a, b, c, d satisfy a < b < c < d. If we pick all six possible pairs of
these numbers and compute their sums, we obtain six distinct numbers, the smallest four being 1, 2, 3,
and 4. Determine all possible values of d.
Result. 3.5, 4
Solution outline. The two smallest sums clearly are a + b and a + c, hence a + b = 1 and a + c = 2.
Similarly, the two largest sums are c + d and b + d, so there are two cases:
• a + d = 3, b + c = 4: Then we have 2b = (a + b) − (a + c) + (b + c) = 1 − 2 + 4, so b = 1.5, resulting
in a = −0.5 and d = 3.5.
• b + c = 3, a + d = 4: The same computation yields b = 1, thus a = 0 and d = 4.

Problem 15J / 5S. Jack is twice as old as Jill was when Jack was as old as Jill is today. When Jill is as
old as Jack is now, the sum of their ages will be 90. How old is Jack now?
Result. 40
Solution outline. Let x and y be Jack’s and Jill’s age, respectively. The first sentence says x = 2(y−(x−y)),
whence 3x = 4y. The second sentence then states ((x − y) + y) + ((x − y) + x) = 90, implying 3x − y = 90.
Therefore x = 40 and y = 30.

Problem 16J / 6S. Positive integers a1 , a2 , a3 , . . . form an arithmetic progression. If a1 = 10 and


aa2 = 100, find aaa3 .
Result. 820
Solution outline. It is well known that the k-th term of an arithmetic progression can be written as
ak = a1 + (k − 1)d = 10 + (k − 1)d. It follows that a2 = 10 + d and thus
aa2 = a10+d = 10 + (9 + d)d = 10 + 9d + d2 .
From the statement we know that aa2 = 100 and hence 100 = 10 + 9d + d2 . The only positive solution of
this quadratic equation is d = 6. We infer that ak = 4 + 6k and the rest is a straightforward computation:
a3 = 4 + 18 = 22, aa3 = a22 = 4 + 132 = 136, and finally aaa3 = a136 = 4 + 816 = 820.

4
Problem 17J / 7S. Natali’s favorite number has the following properties:
• it has 8 digits;
• its digits are pairwise distinct and decreasing when read from left to right; and
• it is divisible by 180.
What is Natali’s favorite number?
Result. 97654320
Solution outline. Natali’s number is divisible by 20, so its units digit is 0 and its tens digit is even.
However, it cannot be 4, 6 or 8, because then there would not be enough digits to complete an 8 digit
number (in the light of the second condition), so it is 2. Further, we decide which of the digits 3, 4, . . . , 9
will be missing to ensure the divisibility by 9. Since 0 + 2 + 3 + 4 + · · · + 9 = 44, we omit the number 8.
Finally, using the second condition, we conclude that Natali’s favourite number is 97654320.

Problem 18J / 8S. Kika likes to build models of three-dimensional objects from square ruled paper.
Last time she scissored out a shape as shown in the figure below. Then she glued it together in such a
way that no two squares were overlapping, there were no holes in the surface of the resultant object and
it had nonzero volume. How many vertices did this object have? Note that by a vertex we mean a vertex
of the three-dimensional object, not a lattice point on the paper.

Result. 12
Solution outline. If we look at the resulting object from the front, we see the same number of squares as
if we look from behind, similarly for other sides. Denote by a, b, and c the number of squares seen from
the front, the left, and above, respectively.
Assume that there are no unseen squares, then we get 2(a+b+c) = 14 (there are 14 squares altogether).
W.l.o.g. we may assume a ≤ b ≤ c. This leaves us with four possible triples (a, b, c), namely (1, 1, 5),
(1, 2, 4), (1, 3, 3), and (2, 2, 3). As the inequality c ≤ ab must hold, only two possibilities remain: a block
with dimensions 1 × 3 × 3 and an “L”-shape, of which only the latter can be constructed from the given
shape.
If there was an unseen square, there would have to be at least five squares visible from one side, which
we have already ruled out.

Problem 19J / 9S. Find all pairs of positive integers a, b such that ab = gcd(a, b) + lcm(a, b), where
gcd(a, b) and lcm(a, b) are the greatest common divisor and the least common multiple of a and b,
respectively.
Result. (2, 2)
Solution outline. Let d = gcd(a, b). By considering the prime factorizations of a and b, we can easily
establish the (general) relation lcm(a, b) = ab/d, hence ab = d + ab/d or (d − 1)ab = d2 . As a, b ≥ d and
d > 0, we infer that a = b = d and d − 1 = 1, thus (a, b) = (2, 2).

5
Problem 20J / 10S. There are 29 unit squares in the diagram below. A frog starts in one of the five
(unit) squares in the bottom row. Each second, it jumps either to the square directly above its current
position (if such a square exists), or to the square that is one above and one to the right from its current
square (if such a square exists). The frog jumps every second until it reaches the top. How many distinct
paths from the bottom row to the top row can the frog take?

Result. 256
Solution outline. Every path of the frog has to go through the square in the middle. On its way from this
square the frog decides four times whether to jump directly up or diagonally, thus the number of ways
from the middle up is 24 = 16. The number of ways from the bottom row to the middle is the same as
the number of ways from the middle downwards using the opposite moves (because of the symmetry of
the diagram), so it is 16 as well. Any two such subpaths may be joined in the middle square to form a
full path, so the total number is 162 = 256.

Problem 21J / 11S. Palo is walking along the diagonals of a regular octagon. His walk begins in a
fixed vertex of the octagon, and should continue along all the diagonals in such a way that he walks along
each diagonal exactly once. How many different walks can Palo take?
Result. 0
Solution outline. Whenever Palo enters a vertex other than the first and the last one of his walk, he has
to leave it along a previously unused diagonal. However, there is an odd number of diagonals incident at
every vertex (namely five), therefore he cannot use all the diagonals incident to some vertex. We conclude
that there is no such walk.

Problem 22J / 12S. Consider a 2014 × 2014 grid of unit squares with the lower left corner in the point
(0, 0) and upper right corner in (2014, 2014). A line p passes through the points (0, 0) and (2014, 2019).
How many squares of the grid does p cross?
Note: A line crosses a square if they have at least two points in common. For example, the diagonal
crosses 2014 squares of the grid.
Result. 4023
Solution outline. As p passes through the points (0, 0) and (2014, 2019), each of its points (x, y) satisfies
x/y = 2014/2019. Since 2014 and 2019 are coprime, the fraction 2014/2019 is in its lowest terms and p
does not pass through any vertex of a square in the grid other than (0, 0). Thus, p crosses a new square
with every intersected inner grid segment. “On its way” from (0, 0) to (2014, 2019), p intersects 2013 such
horizontal and b20142 /2019c = 2009 vertical segments. If we add the corner square in which p “starts”,
we get 1 + 2013 + 2009 = 4023 crossed squares in total.

Problem 23J / 13S. A convex n-gon has one interior angle of an arbitrary measure and all the remaining
n − 1 angles have a measure of 150◦ . What are the possible values of n? List all possibilities.
Result. 8, 9, 10, 11, 12
Solution outline. Every n-gon can be cut into n − 2 triangles, hence the sum of interior angles is
(n − 2) · 180◦ . Thus we have (n − 2) · 180◦ = (n − 1) · 150◦ + x or n = x/30◦ + 7, where 0◦ < x < 180◦ is
an arbitrary convex angle. We conclude that 8 ≤ n ≤ 12.

6
Problem 24J / 14S. If the sides a, b, c of a triangle satisfy
3 1 1
= + ,
a+b+c a+b a+c
what is the angle between the sides b and c?
Result. 60◦
Solution outline. After multiplying the whole expression appropriately to get rid of fractions we obtain

3(a2 + ab + ac + bc) = 2a2 + b2 + c2 + 3(ab + ac) + 2bc,


1
so a2 = b2 + c2 − bc. Comparison with the law of cosines (a2 = b2 + c2 − 2bc cos α) yields cos α = 2 and
α = 60◦ .

Problem 25J / 15S. Find all integers between 1 and 200, whose distinct prime divisors sum up to 16.
(For example, the sum of the distinct prime divisors of 12 is 2 + 3 = 5.)
Result. 66, 132, 198, 55, 39, 117
Solution outline. The number 16 can be written as a sum of distinct primes in the following ways:

16 = 2 + 3 + 11 = 3 + 13 = 5 + 11

The first decomposition corresponds to the numbers of the form 2a 3b 11c with a, b, c positive integers;
only 2 · 3 · 11 = 66, 22 · 3 · 11 = 132 and 2 · 32 · 11 = 198 are less or equal 200. The second case yields
3 · 13 = 39, 32 · 13 = 117 and from the last one we get 5 · 11 = 55.

Problem 26J / 16S. The lengths of line segments in the figure satisfy DA = AB = BE, GA = AC = CF
and IC = CB = BH. Moreover, EF = DI = 5 and GH = 6. What is the area of triangle ABC?
I F

D A B E

G H

Result. 3
Solution outline. The line segment AC joins the midpoints of DB and BI. Therefore its length must be
AC = DI/2 = 2.5. Similarly, CB = EF/2 = 2.5 and AB = GH/2 = 3. We see that ABC is an isosceles
triangle and its area can thus be computed by the Pythagorean theorem as
p √
AC 2 − (AB/2)2 · AB/2 = 6.25 − 2.25 · 1.5 = 3.

Problem 27J / 17S. We call a prime p strong if one of the following conditions holds:
• p is a one-digit prime; or
• if we remove its first digit, we obtain another strong prime, and the same holds for the last digit.
For example, 37 is a strong prime, since by removing its first digit we get 7 and by removing its last digit
we get 3 and both 3 and 7 are strong primes. Find all strong primes.
Result. 2, 3, 5, 7, 23, 37, 53, 73, 373

7
Solution outline. One-digit strong primes (1SP) are 2, 3, 5 and 7.
Two-digit strong primes (2SP) are prime numbers obtained by joining two 1SP together, namely 23,
37, 53, and 73.
Let us now find the three-digit strong primes (3SP): By removing the first digit we get a 2SP and
similarly by removing the last digit. Therefore, we seek two 2SP such that the second digit of one is the
first digit of the other. Out of the possible candidates 237, 537, 737 and 373, only 373 is a prime.
Finally, we focus on the four-digit strong primes (4SP). After removing both the first and the last
digit we need to obtain 3SP, i.e. 373, which is clearly impossible. Hence 4SP do not exist and neither do
strong primes with more than 4 digits. To sum up, the only strong primes are 2, 3, 5, 7, 23, 37, 53, 73,
and 373.

Problem 28J / 18S. Let k and l be two circles of radius 16 such that each of them passes through the
centre of the other. A circle m is internally tangent to both k and l and also tangent to the line p passing
through their centres. Find the radius of the circle m.

k l

Result. 6
Solution outline. Let Sk , Sl and Sm be the centres of the circles k, l, and m respectively. Denote by r
the radius of the circle m. We know Sk , Sm and the point where m touches k lie on one line, whence
Sk Sm = 16 − r. Let A be the point where the circle m touches the line p. By symmetry we have
Sk A = Sl A = 16/2 = 8. Clearly, Sm A = r and the line Sm A is perpendicular to p, so we can use the
Pythagorean theorem in 4Sk ASm , yielding the equation 82 + r2 = (16 − r)2 with the unique solution
r = 6.

Sm
m
Sk A Sl p

k l

Problem 29J / 19S. How many six-digit positive integers satisfy that each of their digits occurs the
same number of times as is the value of the digit? An example of such an integer is 133232.
Result. 82
Solution outline. Firstly observe that the sum of distinct digits of any such number is 6. As 0 cannot
occur among these numbers, we are looking for partitions of 6 into distinct positive integers; these are 6
alone, 1 + 5, 1 + 2 + 3 and 2 + 4. There are 6 · 52 = 60 numbers with digits 1, 2, and 3, since there are


six possible positions for digit 1 and then we choose two positions for digit 2 out of 5 remaining. Using
similar elementary combinatorics, we compute that the partitions 6, 1 + 5 and 2 + 4 provide 1, 6, and
6

2 = 15 valid numbers. Therefore, there are 60 + 1 + 6 + 15 = 82 numbers satisfying the given condition.

8
Problem 30J / 20S. True to his reputation as a cool guy, E.T. glued together 4 balls of radius 1 so
that they were pairwise tangent. What is the radius of the smallest sphere containing E.T.’s 4 balls?


6
Result. 1 + 2

Solution outline. When we decrease the radius of the containing sphere by 1, we obtain the circumscribed
sphere C of the regular tetrahedron T formed by the centres of the given balls. Note that edges of T
are of the
√ length √2. Using the Pythagorean theorem, we discover that faces of T have altitudes of the
length 4 − 1 = 3. Now we can calculate the length of the altitudes of T , once again by means of
q √ √
the Pythagorean theorem, obtaining 4 − ( 23 3)2 = 23 6. It is well known (or we may employ the
Pythagorean theorem for the √
third time) that the centre√of C divides the altitude with the ratio 1 : 3,
yielding the radius of C as 2 and the desired value 1 + 26 .
6

Problem 31J / 21S. Determine the number of pairs of positive integers (x, y) with y < x ≤ 100 such
that the numbers x2 − y 2 and x3 − y 3 are coprime.
Result. 99
Solution outline. Recall x2 − y 2 = (x − y)(x + y) and x3 − y 3 = (x − y)(x2 + xy + y 2 ). In order to achieve
gcd((x − y)(x + y), (x − y)(x2 + xy + y 2 )) = 1, we need x − y = 1, i.e. x = y + 1. Substituting for y, we
search for x such that gcd(2x − 1, 3x2 − 3x + 1) = 1. After subtracting (2x − 1)(x − 1) from 3x2 − 3x + 1,
we get an equivalent condition gcd(2x − 1, x2 ) = 1. Suppose there exists a common prime divisor of
2x − 1 and x2 and denote it p. Then p | x and p | 2x − 1, whence p | x − 1. This contradicts p > 1, so the
assumption is false and gcd(2x − 1, 3x2 − 3x + 1) = 1 for any x ≥ 1. Given that we have shown x = y + 1,
the desired pairs are (2, 1), (3, 2), . . . , (100, 99). All in all there are 99 of them.

Problem 32J / 22S. Consider a number that starts with 122333444455555 . . . and continues in such a
way that we write each positive integer as many times as its value indicates. We stop after writing 2014
digits. What is the last digit of this number?
Result. 4
Solution outline. One-digit numbers require 1 + 2 + · · · + 9 = 45 digits, two-digit numbers 2 · (10 + 11 +
· · · + 99) = 2 · ( 99·100
2 − 9·10
2 ) = 9810. Hence we reach 2014 digits while writing a two-digit number.
Observe that we are asking for a digit in an even position, and considering one-digit numbers take up an
odd length, we only need to know the value of the digit on tens place for the number that occupies the
2014th position. The last written number is the least n such that 45 + 2 · (10 + 11 + · · · + n) > 2014,
which yields a rough estimate 40 < n < 50, and so the last digit is 4.

9
Problem 33J / 23S. What is the smallest positive integer N for which the equation (x2 − 1)(y 2 − 1) = N
has at least two distinct integer solutions (x, y) such that 0 < x ≤ y?
Result. 360
Solution outline. Let us list the first few values that can be obtained by (t2 − 1) for integer t: 0, 3, 8,
15, 24, 35, 48, 63, 80, 99, 120, etc. We want to find two pairs with the same minimal product. Divine
revelation yields 3 · 120 = 15 · 24 = 360, which happens to be the smallest possibility actually: Should one
of the factors be greater than 120, the product would exceed 360. Therefore we may confine ourselves to
the factors listed above. Since at most one factor may be 3, we may concentrate only on the products
without this factor smaller than 360, i.e. 8 · 8, 8 · 15, 8 · 24, 8 · 35, and 15 · 15. These products, however,
do not allow for an alternative factorization. Hence the smallest N really is 360.

Problem 34J / 24S. Let two circles of radii 1 and 3 be tangent at point A and tangent to a common
straight line (not passing through A) at points B and C. Find AB 2 + BC 2 + AC 2 .
Result. 24
Solution outline. Let S1 be the centre of the smaller circle, S2 the centre of the larger one and T the
point where the line BC intersects the common tangent passing through A. Note that T C = T A = T B
(tangent lines). Thales’ theorem then implies that the triangle ABC is right-angled and we obtain
AB 2 + AC 2 = BC 2 . Let us now translate the line segment BC so that B is moved onto S1 and denote
by P the translated point C. Noting CP = S1 B = 1, the Pythagorean theorem entails BC 2 = S1 P 2 =
(S1 A + S2 A)2 − (S2 C − CP )2 = 12, yielding the solution 24.

T P
B

S1 A S2

Problem 35J / 25S. Find the largest prime p such that pp | 2014!.
Result. 43
Solution outline. Let q be a prime. Then factors from 2014! divisible by q are q, 2q, . . . , b2014/qcq.
Denoting m = b2014/qc, we have q m | 2014!. If q > m then q 2 > mq and q m+1 - 2014!, implying q q - 2014!,
which is undesirable. Conversely, if q ≤ m, then clearly q q | 2014!. Hence p must be the largest prime
satisfying p ≤ b2014/pc, i.e. p2 ≤ 2014, which results in p = 43.

Problem 36J / 26S. Consider an array of 2 columns and 2014 rows. Using 3 different colours, we paint
each cell of the array with a colour so that if the cells share a wall, they are of different colours. How
many different colourings are there?
Result. 6 · 32013
Solution outline. First we colour the bottom row arbitrarily, yielding 3 · 2 = 6 possibilities. If a row is
coloured (a, b) then the next one can be (b, a), (b, c) or (c, a). In other words, independently of choosing
a colouring for a row, we have exactly three possibilities of how to colour the following one. That gives us
6 · 32013 different colourings.

10
Problem 37J / 27S. Find the smallest positive integer m such that 5m is the fifth power of an integer,
6m is the sixth power of an integer and 7m is the seventh power of an integer.
Result. 584 635 790
Solution outline. Let us write m = 5a 6b 7c d where d is divisible by neither 5, 6 nor 7. In order for 5m to
be a fifth power, we need 5 to divide a + 1, b and c. Likewise 6 | a, b + 1, c and 7 | a, b, c + 1. The number
d has to be a fifth, sixth and seventh power at the same time; we are looking for the smallest integer, so
we let d = 1. The number a is divisible by 42 and a + 1 by 5, therefore the smallest suitable a is 84. We
find b = 35 and c = 90 analogously. The smallest suitable m turns out to be 584 635 790 .

Problem 38J / 28S. We fold a rectangular piece of paper in such a way that two diagonally opposite
vertices are touching, thus creating a fold, i.e. a line segment across the paper. After folding, the length
of the fold line has the same length as the longer side of the rectangle. What is the ratio of the length of
a longer side to the length of a shorter side of the rectangle?
q √
1+ 5
Result. 2
Solution outline. Label important points as in the picture. Since similar rectangles yield the same result,
we may w.l.o.g. assume AB = 1. The question is now to find x = BC. First note that EF ⊥ AC, as
EF is the axis of symmetry of AC. We can see that ES = F S = x2 by symmetry. The Pythagorean
√ √
2 2
theorem implies CS = x2 +1 . Triangles ABC and ESC are similar, hence we infer x1 = xx+1 . This
4 2 2
relation
q yields x − x − 1 = 0 which, after substituting z = x , produces a unique non-negative solution

1+ 5
x= 2 .

D C


F x2 +1
2

S
x

x
2
E

A B
1

Problem 39J / 29S. We have 20 marbles, each of which is either yellow, blue, green, or red. Assuming
marbles of the same colour are indistinguishable, how many marbles can be blue at most if the number of
ways we can arrange the marbles into a straight line at most is 1140?
Result. 17
Solution outline. Let b, y, g, and r denote the number of blue, yellow, green, and red marbles, respectively.
We have 20

r different ways of placing the red marbles. There remain 20 − r positions for the green
20−r

marbles, hence g different ways of placing them, having placed the red ones. Similarly, the yellow
marbles can be placed in 20−r−g

y different ways after fixing the red and green marbles. The remaining
blue marbles just fill in the void. So we seek the smallest y + g + r such that
   
20 20 − r 20 − r − g
= 1140.
r g y
We can manipulate the expression to obtain
20(20 − 1) · · · (b + 1)
1140 = .
r!g!y!
If b ≥ 18, the numerator is at most 20 · 19 = 380, so the number of ways is less than 1140. For b = 17 the
number 1140 can be attained via setting y = 3, g = 0, r = 0.

11
Problem 40J / 30S. Find all integers n ≥ 3 such that the regular n-gon can be divided into at least
two regular polygons.
Result. 3, 4, 6, 12
Solution outline. Firstly observe that an equilateral triangle can be divided into four equilateral triangles
by its midlines, a square can be divided into four squares and a regular hexagon into six triangles.
Consider a regular n-gon with n 6= 3, 4, 6. The internal angles of regular polygons with 3, 4, 5, and 6
vertices are 60◦ , 90◦ , 108◦ , and 120◦ , respectively. We have to “fill” the internal angle of the n-gon either
using a smaller n-gon or two polygons with at most five vertices. However, the former option leaves us
with an acute angle different from 60◦ , which cannot be filled.
Therefore, the internal angles of the n-gon have to be filled either with a triangle and a square or
with a triangle and a pentagon. The corresponding interior angles are 150◦ and 168◦ , i.e. n is 12 or 30.
It is indeed possible to divide the regular dodecagon: We alternately put squares and triangles to the
sides of the polygon and a hexagon to the middle.

On the other hand, such a division of a triacontagon does not exist; it would need to have a pentagon
adjoined to every other side. Two such neighbouring pentagons would meet in a vertex with one angle
equal to 60◦ , thus the angle on the other side would be 84◦ , which cannot be filled.

60◦

108
84◦

Problem 41J / 31S. A rook is standing in the bottom left corner of a 5 × 5 chessboard. In how many
ways can the rook reach the top right corner of the board, provided that it is limited to move only up or
to the right? We consider two paths to be distinct if the sequences of the visited squares are different.
Result. 838
Solution outline. For each square we compute the number of ways to reach it. There is only one way
for the bottom left corner (empty sequence of moves). For every other square S we take the following
approach: Consider a sequence of moves ending in S and exclude its last move; before this move, the
rook was in a square T either below or to the left of S. On the other hand, whenever the rook gets to a
square T below or to the left of S, by adding one move we obtain a way to reach S. Therefore if we know
the numbers of ways to get to all such squares T , it suffices to sum up all these ways.
Thus we employ the following procedure: Firstly, we write 1 to the bottom left corner of the board
and then we repeatedly apply the rule “Find a square with all squares below and to the left filled in and
fill it with the sum of these values.” The sought number is in the top right corner.

8 28 94 289 838

4 12 37 106 289

2 5 14 37 94

1 2 5 12 28

1 1 2 4 8

12
Problem 42J / 32S. What is the digit in the place of hundreds in 112014 ?
Result. 2
Solution outline. The binomial theorem entails
        
2014 2014 2014 2014
112014 = (1 + 10)2014 = ·1+ · 10 + · 100 + 1000 · + ··· .
0 1 2 3

We need to compute this expression modulo 1000 and to take the first digit, so we may drop the terms
grouped in the last brackets and proceed as

112014 ≡ 1 + 2014 · 10 + 1007 · 2013 · 100 ≡ 1 + 140 + 7 · 3 · 100 ≡ 241 (mod 1000).

Problem 43J / 33S. A grasshopper is jumping on vertices of an equilateral triangle. Whenever it sits
on a vertex, it randomly chooses one of the other two vertices as the destination for its next jump. What
is the probability that it returns to the starting vertex with its tenth jump?
171
Result. 512

Solution outline. Denote by A the starting vertex and by ai the probability of the grasshopper being
in the vertex A after i jumps (so a0 = 1). We can find a recurrence relation for ai+1 provided that ai
is given: If the (i + 1)-th jump of the grasshopper ended in A, then the previous jump did not and the
grasshopper had to choose vertex A out of the two possible ones, so
1
ai+1 = (1 − ai )
2
or, equivalently,  
1 1 1
ai+1 − =− ai − .
3 2 3
As a0 = 1/3 + 2/3, we infer that
 i
1 2 1
ai = + · −
3 3 2
and thus
1 1 171
a10 = + = .
3 3 · 512 512

Problem 44J / 34S. Parsley is a lumberjack. His work is strictly divided into minutes, in which he may
choose one of the following actions:
• He cuts down n trees, where n is his current power. This wears him out, so his power decreases by
one.
• He takes a rest and restores one point of power.
What is the maximal number of trees he may cut down in 60 minutes, if his initial power is 100?
Result. 4293
Solution outline. If Parsley spent a minute resting and in the following minute he cut trees, he would cut
one tree less than if he firstly cut and then rested, but in neither case does his power change. Therefore,
in order to maximize the number of trees cut, he begins with r minutes of rest and proceeds with 60 − r
minutes of cutting for some r. The total number of trees cut is then

(r + 100 + 2r + 41) 3
(r+100)+(r+99)+· · ·+(r+100−(60−r−1)) = ·(60−r) = − ((r−6.5)2 −2862.25).
2 2
The maximum is attained if the term (r − 6.5)2 is lowest possible, i.e. for r = 6 or r = 7. In this case
Parsley cuts 32 · 2862 = 4293 trees.

13
√ √ √ √ √
Problem 45J / 35S. Denote by N the set { 1, 2, 3, 4, . . . }, i.e. √ the set of all √
square roots of
positive integers. Let S be the set of real numbers a for which both a ∈ N and 36/a ∈ N hold. What
is the product of the elements of the set S?
Result. 625
√ √
Solution outline. If a = na,1 and 36/a = na,2 for some na,1 , na,2 ∈ N, it follows na,1 na,2 = 362 . This
observation lets us compute the desired product as follows:
Y sY sY s Y Y s Y √
a= na,1 = n= 2k1 3k2 = 210 3k2 = 250 350 = 625 .
a∈S a∈S n|362 0≤k2 ≤4 0≤k1 ≤4 0≤k2 ≤4

Problem 46J / 36S. What is the probability that the product of 2014 randomly chosen digits is a
multiple of 10?
5 2014 8 2014 4 2014
  
Result. 1 − 10 − 10 + 10
Solution outline. We seek the probability that the product is divisible by both 2 and 5. Let us compute
the probability of the complementary event: the probability of the product not being divisible by 2 is
a = (5/10)2014 (all the factors have to be odd). Similarly, for the divisibility by 5 we have b = (8/10)2014
(there may be no 0 or 5), and the probability of not being divisible neither by 2 nor 5 is c = (4/10)2014 .
The sought probability is 1 − a − b + c, since we have subtracted the probability of non-divisibility by 2
and 5 twice and thus we have to add it back.

Problem 47J / 37S. Evaluate the expression


 √   √   √
3

2014 3 −2014 + 2014 3 −2013 + · · · + 2014 2014 .

Note: The symbol bxc denotes the integral part of x, e.g. the greatest integer not exceeding x.
Result. −2002
Solution outline. We rewrite the expression as
 √3
  √
3
  √   √3
  √ 
2014 0 + 2014 1 + 2014 3 −1 + · · · + 2014 2014 + 2014 3 −2014 =
 √
3
  √
3
  √
3
  √
3
  √
3
  √
3

= 2014 1 − 2014 1 + 2014 2 − 2014 2 + · · · + 2014 2014 − 2014 2014 ,

where dxe denotes the smallest integer greater or equal to x. Observe that if x is an integer, then bxc − dxe
is equal to zero; otherwise it is −1. As the cube root of an integer is either an integer or an irrational
number, the sum in question is minus the number of numbers between 1 and 2014 (inclusive) which are
not a cube. Since 123 = 1728 < 2014 and 133 = 2197 > 2014, there are exactly 12 cubes between 1 and
2014, so the desired value is −(2014 − 12) = −2002.

Problem 48J / 38S. Boris counted the sum 1 + 2 + · · · + 2012. However, he forgot to add some numbers
and got a number divisible by 2011. Ann counted the sum A = 1 + 2 + · · · + 2013. However, she missed
the same numbers as Boris and got a number N divisible by 2014. What is the ratio N/A?
Result. 2/3
Solution outline. Boris’s number is N − 2013, so

N ≡ 2013 ≡ 2 (mod 2011)

and N ≡ 0 (mod 2014). We may similarly proceed with the number 2A:

2A = 2013 · 2014 ≡ 0 (mod 2014)

and 2A ≡ 2 · 3 = 6 (mod 2011). Since 2A is divisible by 3 (as it is divisible by 2013), but neither of 2011
and 2014 is, we may divide the congruences for A by 3 and find that the number 2A/3 satisfies precisely
the conditions for N . On the other hand, no other such N is admissible, since the distance of the next
closest numbers satisfying the congruences for N is 2011 · 2014 > A. Thus N/A = 2/3.

14
Problem 49J / 39S. A game is played with 16 cards laid in a row. Each card has one side black and
the other side red. A move consists of taking a consecutive sequence of cards (possibly only containing 1
card) in which the leftmost card has its black side face-up and the rest of the cards have red side face-up,
and flipping all of these cards over. The game ends when a move can no longer be made. What is the
maximum possible number of moves that can be made before the game ends?
Result. 216 − 1
Solution outline. Imagine the row of 16 cards as a binary number where a black side face-up corresponds
to 0 and a red one to 1. A move in our game then corresponds to binary addition of some number to
thus interpreted row of cards so that the sum would always remain less than 216 . Therefore the game
cannot run forever and we have an upper bound for the number of moves 216 − 1. This number of moves
can actually be attained: consider the initial state consisting of all zeroes (i.e. all cards are black side
face-up) and perform moves corresponding to adding 1 in every move.

Problem 50J / 40S. We have a right-angled triangle with sides of integral lengths such that one of the
catheti is 201414 in length. How many such triangles exist up to congruence?
27·292 −1
Result. 2

Solution outline. Denote the length of the second cathetus b and the hypotenuse c. The Pythagorean
theorem implies (c − b)(c + b) = 201428 . Since all the lengths are integers, the question is in how many
ways we can write 201428 as a product of two distinct, even divisors: the numbers c − b and c + b have
the same parity, hence both must be even. The possibility c − b = c + b is ruled out for it implies b = 0.
We have 201428 = 228 · 1928 · 5328 , so there are 27 · 29 · 29 − 1 pairs of divisors satisfying the conditions.
2
Given that we have to take into account also c − b < c + b, we get 27·292 −1 possible triangles.

Problem 51J / 41S. What is the smallest positive integer that cannot be written as a sum of 11 or
fewer (not necessarily distinct) factorials?
Result. 359
PN
Solution outline. Observe that every n ∈ N can be uniquely decomposed as n = k=1 ak k! for some N
dependent on n and ak ∈ N such that ak ≤ k since (k + 1)k! = (k + 1)!. For instance 43 = 1 + 3 · 3! + 4!.
PN
The question asked can thus be reformulated as finding the smallest n for which k=1 ak = 12. This
question can now easily
P5 be answered as the corresponding ak will be a1 = 1, a2 = 2, a3 = 3, a4 = 4 and
a5 = 2 so that n = k=1 ak k! = 359.

Problem 52J / 42S. In a chess tournament, each participant plays against every other participant, and
there are no draws. Call a group of four chess players ordered if there is a clear winner and a clear loser,
that is, one person who beat the other three and one person who lost to the other three. Find the smallest
integer n for which any chess tournament with n people has a group of four chess players that is ordered.
Result. 8
Solution outline. Note first that for 8 participants, the average number of victories of a player is 3.5,
and hence there must be a player v who won (at least) 4 times, say against players a1 , a2 , a3 and a4 .
Similarly, one of these four players must have won against at least two other of them, so without loss of
generality assume that a1 defeated a2 and a3 . Finally, somebody won the match a2 against a3 , we can
without loss of generality assume it was a2 . Observe that the group {v, a1 , a2 , a3 } is ordered. Hence we
see n ≤ 8.
Actually n = 8, because a smaller number of participants permits a tournament without an ordered
group of 4 players. To prove this, assume without loss of generality there are 7 players a0 , . . . , a6 . For
every 0 ≤ i ≤ 6, consider the player ai won the match with aj where j = (i + 1) mod 7, (i + 2) mod 7,
and (i + 4) mod 7. The only possible candidates for an ordered group of size 4 are quadruples of a player
and the three beaten by him/her. However, such groups are not ordered.

15
Problem 53J / 43S. I chose two numbers from the set {1, 2, . . . , 9}. Then I told Peter their product
and Dominic their sum. The following conversation ensued:
• Peter: “I don’t know the numbers.”
• Dominic: “I don’t know the numbers.”
• Peter: “I don’t know the numbers.”
• Dominic: “I don’t know the numbers.”
• Peter: “I don’t know the numbers.”
• Dominic: “I don’t know the numbers.”
• Peter: “I don’t know the numbers.”
• Dominic: “I don’t know the numbers.”
• Peter: “Now I know the numbers.”
What numbers did I choose?
Result. 2, 8
Solution outline. Given that there must be a solution, both players can be expected to behave perfectly
rationally. They write sums and products of all pairs from {1, . . . , 9}. If Peter saw that the number he
had been told was there only once, he would have known the original numbers immediately. But he could
not infer a clear answer. Therefore Dominic might ignore all pairs of numbers that have a unique product.
Then if Dominic had a unique sum, he would have known the numbers, but he did not, therefore Peter
could cross out all pairs with a unique sum. And in this manner we carry on.

Problem 54J / 44S. Three identical cones are placed in the space in such a way that their bases are
pairwise incident and all three bases completely lie in a single plane. We place a sphere between the cones
so that the top of the sphere is in the same height as the vertices of the cones. What is the radius of the
ball if each cone has base of radius 50 cm and height 120 cm?


200 3
Result. 9

Solution outline. Denote by r the radius of the sphere and O the center of the sphere. Select one of the
cones and denote the center of its base as S, its vertex as V and its tangent point with the sphere as T .
Let b be the plane in which bases of the cones lie. Let P be the orthogonal projection of O to b, X be on
the line OP so that T X is parallel with b, and Y be the intersection of the line V T and the boundary of
the selected cone. Finally, denote Z the orthogonal projection of T to b.
The Pythagorean theorem implies V Y = 130. As the sphere touches the cone, V Y is perpendicular to
5
T O. Hence 4T XO is similar to 4V SY and therefore OX = 13 r and T X = 12 18
13 r. Thus T Z = 120 − 13 r.
18 5 15
From the similarity of 4V Y S and 4T Y Z, we get Y Z = (120 − 13 r) · 12 = 50 − 26 r and consequently
SZ = 15 12
26 r. On the other hand, P Z = T X = 13 r. Observe that the base centers of the cones constitute

an equilateral triangle, with the side length 100, for which P is the center of mass. So SP = 1003 3 and
√ √ √
SZ = 1003 3 − 1312
r. With two expressions for SZ, we equate 1003 3 − 1312
r = 15
26 r, yielding r =
200 3
9 .

16
V

T X

S Z Y P

Problem 55J / 45S. Imagine a rabbit hutch formed by 7 × 7 cell grid. In how many ways can we
accommodate 8 indistinguishable grumpy rabbits in such a fashion that every two rabbits are at least
either 3 columns or 3 rows apart?
Result. 51
Solution outline. Let the columns of the hutch be called A–G and the rows 1–7. We will hold a discussion
according to the position of the rabbit closest to the center. If the central cell (D4) is taken, the remaining
rabbits can live only along the perimeter. By means of analysing every allowable pattern of such an
accommodation and utilizing the rotational symmetry, we obtain there are 36 possibilities then.
Consider now a rabbit dwells in D3. Then we have 2 alternatives, depending on whether D6 or D7 is
used. By symmetry we obtain that occupation of D3, D5, C4 or E4 yields 8 possibilities overall. If D2 is
taken, we have 2 alternatives not covered by the previous case. Since one of them coincides with taking
D6, we observe that occupation of D2, D6, B4 or F4 yields 6 possibilities overall. Next, if C3, B2 or C2
(or symmetries thereof) is occupied, it can be easily verified there is no way how to place the 7 remaining
rabbits. Lastly, there is only one way how to place all the rabbits along the perimeter. All in all, we have
36 + 8 + 6 + 1 = 51 possibilities.
A B C D E F G A B C D E F G
1 1
2 2
3 3
4 4
5 5
6 6
7 7

A B C D E F G A B C D E F G
1 1
2 2
3 3
4 4
5 5
6 6
7 7

17
Problem 56J / 46S. Given noncollinear points A, B, C, the segment AB is trisected by points D and
E. Furthermore, F is the midpoint of the segment AC, and EF and BF intersect CD at G and H,
respectively. Compute [F GH] provided that [DEG] = 18. Note that by [XY Z] we denote the area of
4XY Z.
C

F
H
G

A D E B

9
Result. 5

Solution outline. We will solve this problem using mass point geometry. Physical intuition states that
masses on opposite sides of a fulcrum balance if and only if the products of the masses and their distances
from the fulcrum are equal (in physics-speak, the net torque is zero). If a mass of weight 1 is placed at
vertex B and masses of weight 2 are placed at vertices A and C, then 4ABC balances on the line BF
and also on the line CD. Thus it balances on the point H where these two lines intersect. Replacing the
masses at A and C with a single mass of weight 4 at their center of mass F , the triangle still balances at
H. Thus BH HF = 4.
Next, consider 4BDF . Placing masses of weight 1 at the vertices B and D and a mass of weight 4
at F , the triangle balances at G. A similar argument shows that EG DG
GF = 2 and that GH = 5. Because
[DEG] EG DG
4DEG and 4F HG have congruent (vertical) angles at G, it follows that [F HG] = F G · HG = 2 · 5 = 10.
[DEG] 18
Thus [F GH] = 10 = 10 = 95 .

Problem 57J / 47S. The surface of a solid consists of two equilateral triangles with sides of length 1
and of six isosceles triangles with legs of length x and the base of length 1, as shown below. What is the
value of x given that the volume of the object is 6?


5 39
Result. 3

Solution outline. First consider a regular octahedron of side length 1, that is, the case x = 1. To compute
its
√ volume, divide it into two square-based pyramids with edges of length 1. Such a pyramid has height
2/2, so its volume is √ √
1 2 2 2
·1 · = .
3 2 6

We conclude that the volume of a regular octahedron is 2/3.
Note that the given solid can be obtained from the regular octahedron by stretching it appropriately
in the direction perpendicular to a pair of faces of the solid; let k be the ratio of this dilation. Since
octahedron dilates√in one dimension only, its volume increases k times as well. As the volume of the solid
is 6, we get k = 9 2.

18
Denote the vertices of the non-stretched triangles A, B, C and D, E, F in such a way that if we
project D, E, F to the plane ABC and denote the projections D√0 , E 0 , F 0 , respectively, then AD0 BE 0 CF 0
is a regular hexagon. From AB = 1 we easily establish AD0 = 3/3 and
r
p 1
AD = h2 + AD02 = h2 + ,
3
where h is the height of the solid (based on ABC).
F

D E

A C
0
D
B
p
Applying this formula in the case of a regular octahedron√(AD = 1) yields hreg = 2/3. The height of
the stretched octahedron is k times longer, i.e. hstr = 18/ 3. Substituting to the formula above yields
r r √ √
2 1 325 5 13 5 39
AD = hstr + = = √ = .
3 3 3 3

19
Problem 1J. Little Peter is a cool guy, so he wears only pairs of socks consisting of two socks of different colours.
There are 30 red, 40 green, and 40 blue socks in his wardrobe in an unlighted cellar room. Peter takes one sock after
the other out of the wardrobe without being able to recognize its colour. What is the minimum number of socks he has
to take out if he needs to get eight two-coloured pairs of socks? Note that one single sock must not be counted in two
different pairs.
Result. 48
Solution. If Peter pulls out all the green socks and seven red socks, he cannot form eight pairs as desired, so 47 is not
sufficient. But if he takes out 48 socks, there are at least 48
3 = 16 socks of one colour and at least 48 − 40 = 8 ones not
having this colour among them, which enables him to form eight two-coloured pairs of socks.

Problem 2J. Assume that x and y are positive integers satisfying x2 + 2y 2 = 2468. Find x if you know that there is
only one such pair (x, y) and 1234 = 282 + 2 · 152 .
Result. 30
Solution. Using the given equality 1234 = 282 + 2 · 152 , we get
2468 = 2(282 + 2 · 152 ) = (2 · 15)2 + 2 · 282 .
Since we know that there is only one such pair, we obtain x = 30.

Problem 3J. A digital watch displays time in hours and minutes using the twenty-four hour time format. How many
minutes per day can one see the digit 5 on the display?
Result. 450
Solution. There are two hours during which the digit five is displayed all the time: 5 and 15. That makes 120 minutes.
In the rest of the day, five can be seen during the last ten minutes of each hour (22 · 10 = 220 minutes) and then five
times during the remaining fifty minutes (22 · 5 = 110 minutes). To sum up, five is shown exactly 450 minutes.

Problem 4J. A big rectangle with perimeter 136 cm is divided into seven congruent rectangles as in the picture.

What is the area of the big rectangle in cm2 ?


Result. 1120
Solution. Since the side lengths of the small rectangles are in ratio 2 : 5, let us denote them by 2x and 5x. The side
lengths of the big rectangle are 10x and 7x, hence 2 · (10x + 7x) = 34x is the perimeter of the big rectangle. This
yields x = 4 cm, thus the area is 10 · 7 · 42 cm2 = 1120 cm2 .

Problem 5J. A box of chocolates has the shape of an equilateral triangle of side length s cm. There are 2n chocolates
in the shape of equilateral triangles, which tightly fill the box: n of side length 1 cm and n of side length 2 cm. What
is the smallest possible value of s?
Result. 10
Solution. Let a be the area of a small chocolate, i.e. the one with side length 1 cm. Then the area of a big chocolate
is 4a, the total area of all chocolates is na + 4na = 5na, and the area of the box is s2 a as the shape of the box is just
the shape of a small chocolate stretched by factor s in all directions. It follows that 5n = s2 , so s is a multiple of 5.
It can be proven that it is impossible to fit five big chocolates in a box of side length 5 cm, so s 6= 5. However, one
can easily find a composition of 20 small and 20 big chocolates inside a box of side length 10 cm.

1
Problem 6J. Little Peter has grown up, so now he wears pairs of socks consisting of two socks of the same colour.
He has also got many new socks, so there are currently 20 brown, 30 red, 40 green, 40 blue, 30 black, and 20 white
socks in his wardrobe. However, the wardrobe is still situated in an unlighted cellar room. What is the minimum
number of socks Peter has to take in order to get eight pairs of socks, if he cannot recognize their colours during the
procedure? Note that one single sock must not be counted in two different pairs.
Result. 21
Solution. On one hand, every number of socks chosen is the sum of an even number of socks, which are used to build
pairs of socks, and a certain number of unpaired socks, which can be at most six due to the number of colours. If Peter
takes 21 socks out of his wardrobe, there cannot be six unpaired socks among them because 21 − 6 = 15 is not even.
Thus there are at most five single socks and the remaining socks form at least 21−52 = 8 pairs. On the other hand, it
does not suffice to take 20 socks because Peter could have taken seven pairs of white socks and six single socks, one of
each colour. It follows that Peter has to take at least 21 socks out of his wardrobe.

Problem 7J. A square and a regular pentagon are inscribed in the same circle and they share a vertex. What is the
largest interior angle of the polygon which is the intersection of the two polygons?
Result. 153◦
Solution. Denote the vertices as in the picture. Then the polygon AB1 B2 C1 C2 D1 D2 is the intersection of the two
given polygons.
A

W Z

B B1 D2
D
B2 D1

C1 C2

X Y
C
Since the figure is symmetric with respect to the line AC, it suffices to check the interior angles at A, B1 , B2 , and C1 .
Clearly, the first one is 90◦ and the last one is 135◦ . As B2 D1 is parallel to XY , we have ∠D1 B2 B1 = ∠Y XW = 108◦ ,
and since ∠C1 B2 D1 = ∠CBD = 45◦ (B2 D1 k BD), the angle at B2 is 153◦ . Finally, triangle B1 BB2 is right-angled,
from which we easily obtain that the angle at B1 is 117◦ . Thus the largest angle is 153◦ .

Problem 8J. The circle 1 has diameter 48 mm. What diameter should the circle 2 have to make the whole mechanism
work?

2 1

Result. 20 mm
Solution. Having counted the numbers of teeth, one can observe that one full rotation of wheel 1 forces 20 4
15 = 3 of a
full circle of the double-wheel. Similarly, when the double-wheel performs one full rotation, wheel 2 covers 10 = 95 of a
18

full circle. Hence whenever wheel 1 rotates one full circle, wheel 2 rotates 43 · 95 = 12
5 of the full circle. The circumference
5
of circle 2 must therefore be 12 times the circumference of circle 1. The ratio of circumferences equals the ratio of
5
diameters, which allows us to compute the diameter of circle 2 as 12 · 48 mm = 20 mm.

2
Problem 9J. For his sixty-two-day summer holiday in July and August, Robert had prepared a precise plan on
which days he would lie and on which he would tell the truth. On the k-th day of the holiday (for each k from 1 to 62),
he reported that he had planned to lie on at least k days. How many of these statements were lies?
Result. 31
Solution. Observe that if Robert was telling the truth on one day, then he must have been doing so on all the previous
days. If he was telling the truth only for k < 31 days, it would contradict the fact that he was lying for 62 − k > 31
days. Similarly, telling the truth for more than 31 days would result in lying too little. We infer that Robert lied on
exactly 31 days.

Problem 10J. In a game of battleship, our opponent has hidden an aircraft carrier, represented by a 5 × 1 or 1 × 5
block, somewhere inside a 9 × 9 cell grid. What is the smallest number of shots, i.e. choices of a cell in the grid, that is
needed to ensure that we have hit the carrier at least once?
Result. 16
Solution. The decomposition of the 9 × 9 cell grid in figure 1 shows that 16 shots are necessary since every 5 × 1
rectangle must be hit by at least one shot.

But 16 shots are sufficient as well, as we see from figure 2.

Therefore 16 is the correct answer.

Problem 11J / 1S. What is the maximum possible value of a common divisor of distinct positive integers a, b, c
satisfying a + b + c = 2015?
Result. 155
Solution. Since 5 · 13 · 31 = 2015 = a + b + c = gcd(a, b, c) · (a0 + b0 + c0 ) for some distinct positive integers a0 , b0 , c0 ,
we obtain a0 + b0 + c0 ≥ 6. Hence gcd(a, b, c) can be at most 5 · 31 = 155. We can reach the maximum by taking any
distinct positive integers that sum up to 13. For instance, a0 = 1, b0 = 5, c0 = 7 yield a = 1 · 155, b = 5 · 155, c = 7 · 155.

Problem 12J / 2S. A train supplying an ironworks consists of a locomotive (which is always in the front) and six
carriages, each carrying either coal or iron ore. Adam wanted to take a picture of the train, but he failed to capture
the whole train, so only an iron ore carriage directly followed by two coal carriages was visible in the photo. The
carriages were not completely symmetric, so it was clear that the ore carriage was the first one of these three. How
many different trains can be photographed so that one obtains the same picture as Adam did?
Result. 31

3
Solution. There are four possibilities where the photographed I-C-C carriage sequence can be, and for each of them
there are 23 ways to complete the train. However, this results in counting the train I-C-C-I-C-C twice. Thus the
number of different trains is 4 · 8 − 1 = 31.

Problem 13J / 3S. An object built from several identical cubes looks like ‘1’ from behind, and like ‘3’ from above
(see the picture). How many cubes can be seen from the right if we know that the object contains maximum possible
number of cubes?

Note: The figure below shows a cube, its back view and its top view, respectively.

Result. 17
Solution. The structure can obviously fit in a box that is two cubes wide, five cubes high and five cubes long. Let us
divide it in half and analyse its two 1 × 5 × 5 parts separately. If we look at the structure from the front, we can see
mirror ‘1’. Hence in the right part, there is one square visible from the front and five squares visible from above. The
only way to achieve that is to put five cubes in one row. Similarly for the left part, there are five squares visible from
the front and three squares visible from above, so we get the maximum number of cubes if we place five cubes in each
of the three columns.
The resulting structure looks like the one in the following picture. If we look at it from the right, we can see 17
cubes.

Problem 14J / 4S. We say that a positive integer n is delicious if the sum of its digits is divisible by 17 and the
same holds for n + 10. What is the smallest delicious number?
Result. 7999
Solution. Denote by Q(r) the sum of digits of r. If the tens digit of n differs from 9, then we have Q(n + 10) = Q(n) + 1.
Hence the tens digit of n has to be 9. If the hundreds digit differs from 9, we have Q(n + 10) = Q(n) − 8, but if the
hundreds digit is 9 and the thousands digit is not, we get Q(n + 10) = Q(n) − 17, so we can make both Q(n) and
Q(n + 10) divisible by 17. To keep n as small as possible, suppose that this is the case and Q(n) = 2 · 17 = 34. The
sum of all digits but the hundreds and tens one is therefore 34 − 2 · 9 = 16 < 2 · 9, which means that two more digits
are enough. Now it is easy to see that n = 7999 is the desired result.

Problem 15J / 5S. A bus company runs a line between towns A and D with stops in towns B and C (in this order).
The ticket price is directly proportional to the distance travelled by the bus. For example, the ticket from A to C costs
the same as the tickets from A to B and from B to C together. Moreover, the company does not offer return tickets,
only one way ones. Lisa, a keen bus ticket collector, wants to gather tickets with all possible prices regardless of the
direction of the journey. So far she has got the tickets costing 10, 40, 50, 60, and 70. What are the possible prices of
the missing ticket?
Result. 20, 110

4
Solution. Assume first that Lisa has the most expensive ticket (i.e. the one from A to D) in her collection; thus its
price is 70. As this price is the sum of the tickets for the sections AB, BC, and CD, at least two of which are already
in Lisa’s possession, we see that the only possibility for the prices of these three tickets is 10, 20, and 40, so the missing
price has to be 20. It is easy to see that these prices can be assigned to the sections so that they agree with the rest of
the tickets.
If the most expensive ticket is missing, then the ticket costing 70 has to be for a journey with one stop; the only way
to decompose this price to a sum of two which Lisa already has is 10 + 60. We infer that the ticket for the remaining
section costs 40 and the longest journey costs 10 + 40 + 60 = 110. It is again easy to check that these values are
satisfactory.

Problem 16J / 6S. In a clock and watch store, Helen admires a watch which is packed in a transparent rectangular
box such that the center of the box and the center of the watch (the point where the hands meet) coincide. The shorter
side of the box is 3 cm long. She notices that at noon, the hour hand points to the middle of the shorter side of the
box, and at one o’clock, it points to the corner of the box. How far apart are the two points on the boundary of the
box where the hour hand points to one o’clock and to two o’clock, respectively?

Result. 3 cm
Solution. Denote by Px the point on the boundary of the box where the hour hand points at x o’clock, and let C be
the center of the box. Since ∠P2 CP1 = ∠P3 CP2 = 30◦ , we see that the point P2 is the incenter and centroid of the
equilateral triangle CC 0 P1 , where C 0 is C reflected through P3 . Now, the distance√P1 P2 , which
√ we are looking for, is
2/3 of the height of an equilateral triangle of side length 3 cm, i.e. P1 P2 = 32 · 12 · 3 · 3 cm = 3 cm.

P1

P2
C P3 C0

Problem 17J / 7S. Find an arrangement of 1, 2, . . . , 9 as a nine-digit number such that any two consecutive digits
form a number which is a product k · l of digits k, l ∈ {1, 2, . . . , 9}.
Result. 728163549
Solution. Let x, y ∈ {1, 2, . . . , 9} be distinct digits. A pair xy will be called valid whenever there exist k, l ∈ {1, 2, . . . , 9}
such that 10x + y = kl holds. Since the only valid pair involving 9 is 49, the block 49 must appear at the end of the
desired nine-digit number z. There are two valid pairs involving 7, namely 27 and 72. They cannot occur simultaneously,
and the end of z is already occupied. Therefore 72 must be put at the beginning of z. Since valid pairs involving 8 are
18, 28, 48 and 81, and 4 has been already used to form 49, the only option is to form a block 281. Hence z = 7281 . . . 49.
Now we have to handle the remaining digits 3, 5 and 6. Since neither 13 nor 34 is a valid pair, and the only valid pair
xy such that x ∈ {5, 6} and y = 3 is 63, we find z = 728163549. Clearly, z satisfies all imposed conditions.

Problem 18J / 8S. Find the largest prime p less than 210 such that the number 210 − p is composite.
Note: Recall that number 1 is neither prime nor composite.
Result. 89
Solution. Instead of looking for the largest prime p, let us look for the smallest composite number n such that 210 − n
is a prime. We have 210 = 2 · 3 · 5 · 7. Consequently, if n was divisible by 2, 3, 5, or 7, so would be 210 − n, i.e. 210 − n
would not be prime. The next smallest composite candidate for n is 112 which gives p = 210 − 121 = 89, which is a
prime.

Problem 19J / 9S. The management of an elementary school decided to buy a certain number of pencils and to
distribute them among the first grade pupils, who were divided into classes A, B, and C. If they gave the same number
of pencils to every pupil, each of them would get nine pencils. If they handed them only to class A, each pupil in this
class would get 30 pencils, and if they chose class B instead, each would get 36 pencils. How many pencils would a
single pupil from class C receive if only this class was given pencils?
Result. 20

5
Solution. Denote by T the total number of pencils and by a, b, c the numbers of pupils in the corresponding classes.
From the statement we know that T = 9(a + b + c), T = 30a, and T = 36b. We are looking for T /c. Substituting
a = T /30 and b = T /36 in the first equation yields
3
T = 10 T + 14 T + 9c,
9
20 T = 9c,
T
c = 20.

Problem 20J / 10S. Find all four-digit square numbers which have the property that both the first two digits and
the last two digits are non-zero squares, the latter one possibly starting with zero.
Result. 1681
Solution. Since for k ≥ 50 the inequality (k + 1)2 − k 2 > 100 holds, 502 = 2500 is the only square starting with 25
(similarly for 3600, 4900, 6400, and 8100). Hence the first two digits have to be 16. The only square greater than 1600
and less than 1700 is 412 = 1681, which clearly satisfies the problem conditions.

Problem 21J / 11S. A driver was driving on a highway between two cities at a constant speed. Unfortunately, some
parts of the highway were under repair, so he had to reduce his speed by one fourth while he was driving through these
sections. Consequently, by the time he would normally have reached his destination, he had travelled only six sevenths
of the entire distance. At that point, what fraction of the total time he had been driving did he spend in the sections
under repair?
Result. 4/7
Solution. Let x be the fraction of time spent in the sections under repair. Then 1 − x is the fraction of time spent on
the rest of the highway. Thus we have
6 3
= x + 1 − x,
7 4
which yields x = 4/7.

Problem 22J / 12S. A rectangle with integral side lengths is decomposed into twelve squares with the following
side lengths: 2, 2, 3, 3, 5, 5, 7, 7, 8, 8, 9, 9. What is the perimeter of the rectangle?
Result. 90
Solution. By summing the areas of the squares, the area of the rectangle is 464 = 24 · 29. The lengths of both sides of
the rectangle have to be at least 9 because of the square with side length 9. Hence the only possible factorization is
16 · 29, which yields the perimeter 90.
Note: Such a decomposition exists, as one can see in the figure:

5
8 7
9
3 2

2 3

8 9
7
5

Problem 23J / 13S. A square sheet of paper is folded so that one of its vertices is precisely on one of the sides. As
in the picture, there is a small triangle, which overlaps the original square. The length of its outer side that is adjacent
to the line of the folding is 8 cm, and the length of the other outer side is 6 cm.

What is the side length of the paper?

6
Result. 36 cm
Solution. By checking angles, we see that all triangles in the statement picture are right-angled and similar to each
other. If we denote the side lengths of the triangle in the lower right corner by 6x, 8x, and due to the Pythagorean
theorem 10x, then by “unfolding” we see that the side length of the square is 18x. This implies that the lower left
triangle has one side of length 18x − 6x = 12x, as can be seen in the following picture. The other sides of this triangle
are of length 9x and 15x due to the scaling factor 3/2 of similarity. Now we can see that the side length of the given
square is also 15x + 6 cm, which yields x = 2 cm. Therefore the side length of the paper is 36 cm.

8 10x
10
6

15x
10x
9x 8x

12x 6x

Problem 24J / 14S. An old steamship is moving along a canal at a constant speed. Simon wants to find out the
length of the ship. While the ship is slowly advancing, he walks at the waterside at a constant speed from the rear of
the ship to its nose counting 240 steps. Then he immediately turns around and walks backwards up to the rear of the
steamship counting 60 steps. What is the length of the steamship in steps?
Result. 96
Solution. When Simon returns to the back of the ship, he has made 300 steps and the ship has moved on 240 − 60 = 180
steps. Hence during the time in which Simon makes 60 steps, the ship moves 180 : 5 = 36 steps forward. However,
Simon reaches the back of the ship after 60 steps, so the length of the ship must be 60 + 36 = 96 steps.

Problem 25J / 15S. The number 137641 = 3712 is the smallest six-digit number such that it is possible to cross out
three pairwise different digits to obtain its square root: 137641. Find the largest six-digit number having this property.
Result. 992016 = 9962
Solution. Let (1000 − n)2 be the desired number (n ≥ 1). We can calculate the squares for n = 1, 2, 3, 4, . . . by using
the formula (1000 − n)2 = 1000 · (1000 − 2n) + n2 to get 9992 = 998001, 9982 = 996004, 9972 = 994009, 9962 = 992016,
. . . It remains to see that the digits 2, 0, 1 are pairwise different and that 992016 = 996 is the square root of 992016.
Therefore 992016 is the desired number.

Problem 26J / 16S. Linda typed something on her calculator, and so a three-digit number appeared on the display.
Patrick, who was sitting opposite to her, noticed that from his point of view (upside-down) it looked exactly as a
three-digit number that is greater by 369 than the one typed. What was the number that Linda typed?
Note: The calculator has a seven-segment display, therefore the digits look like this:

Result. 596
Solution. Notice that if we turn a digit upside-down, then either it does not change (0, 2, 5, 8), or it changes (6 ↔ 9),
or we do not get a digit at all (1, 3, 4, 7). Let us denote by x the Linda’s number and by x0 the same number
upside-down. Since the number 369 ends with 9, there are three candidates for the units digit of x: 0, 6, and 9 (all
other admissible numbers lead to a non-admissible units digit of x0 ).
If x ended with 0, then x0 would begin with 0, which is impossible. Furthermore, x ending with 9 would result in x0
ending with 8, i.e. x beginning with 8 and x0 beginning with 6, which is a contradiction since x > x0 . Finally, if 6 is the
units digit of x, we have x = 5a6 and x0 = 9a0 5 for some admissible digit a and its upside-down version a0 . However,
a = a0 is not possible in the light of x0 − x = 369, so a has to be either 6 or 9. Direct computation shows that x = 596
is the sought value.

7
Problem 27J / 17S. There are three families living on the island of Na-boi, each having two sons and two daughters.
In how many ways can these twelve people form six married couples if the marriages of siblings are forbidden?
Result. 80
Solution. Denote the families A, B, and C. If the sons from A marry two sisters from some other family (w.l.o.g.
B), then the daughters from A have to marry the sons from C (otherwise at least one son from C would marry his
sister). It follows that the sons from B marry the daughters from C. This yields 2 · 23 = 16 ways—there are two ways
to match the families, and then each pair of daughters has two options of marrying the sons from the matched family.
If, on the other hand, the sons from A decide to have wives from different families, their sisters must do the same in
order to prevent inter-family marriages. The two sons together have eight choices of wives and the same holds for
the daughters. After this matching is done, in each family B and C there is exactly one son and one daughter left
unmarried, which gives the one and only way to complete the marriage scheme. Therefore this scenario gives 8 · 8 = 64
ways.
We conclude that there are 16 + 64 = 80 ways to marry the people.

Problem 28J / 18S. Hansel and Gretel have baked a very big pizza consisting of 50 slices of equal circular sector
shape. They have distributed olives on the slices in such a way that there are consecutively 1, 2, 3, . . . , 50 olives placed
clockwise on the slices. Now they want to divide the pizza into two equal halves by making a straight cut between
the slices so that Hansel gets twice as many olives as Gretel. Determine the total number of olives on the four slices
adjacent to the cutting line.
Result. 68, 136
Solution. Let us number the slices by the number of olives put on them. Note that the cutting line cannot separate
the slices 1 and 50 (or the slices 25 and 26) because clearly
2 · (1 + 2 + · · · + 25) < 26 + 27 + · · · + 50.
Hence we can assume that n, n + 1, n + 25, n + 26 are the numbers of the slices adjacent to the cutting line, where
1 ≤ n ≤ 24. The sum of these numbers is 4n + 52. Now we compute (n + 1) + (n + 2) + · · · + (n + 25) = 25n + 12 · 25 · 26 =
25(n + 13) and 1 + 2 + · · · + 50 = 12 · 50 · 51 = 25 · 51. Then we have two possibilities:
1 2
25(n + 13) = · 25 · 51 = 25 · 17 or 25(n + 13) = · 25 · 51 = 25 · 34.
3 3
The first possibility yields n = 4 and the desired sum is 4n + 52 = 68. From the second possibility we obtain n = 21
and 4n + 52 = 136. In total, there are two solutions, namely 68 and 136.

Problem 29J / 19S. Find all primes p with the property that 19p + 1 is the cube of an integer.
Result. 421
Solution. If p is a prime fulfilling the given condition, then there exists an integer k > 2 with k 3 = 19p + 1. Therefore
we get the equation
19p = k 3 − 1 = (k − 1)(k 2 + k + 1).
Due to k > 2, both factors on the right-hand side are proper divisors of 19p. Since 19p is a product of two primes,
we have either k − 1 = 19 or k 2 + k + 1 = 19. The first case leads to k = 20 and p = 400 + 20 + 1 = 421, which is a
prime. In the second case, the quadratic equation k 2 + k − 18 = 0 has no integer solution. Therefore p = 421 is the
only solution.

Problem 30J / 20S. In a parallelogram ABCD, the point E lies on side AD with 2 · AE = ED and the point F
lies on side AB with 2 · AF = F B. Lines CF and CE intersect the diagonal BD in G and H, respectively. Which
fraction of the area of parallelogram ABCD is covered by the area of the pentagon AF GHE?
7
Result. 30
Solution. In the following, we shall denote the area by square brackets. Since the triangles EHD and CHB are
similar, the equation
BH BC AD 3
= = 2 =
HD ED 3 · AD
2
follows. Due to the similarity of the triangles F BG and CDG, we get DG
GB = 32 . Therefore DH = BG = 2
5 · DB and
HG = 15 · DB. Since
2 2 1
[ECD] = [ACD] = · [ABCD] = [F BC],
3 3 2
we get  
1 1 1 7
[AF GHE] = [AF CE] − [GCH] = − · [ABCD] = [ABCD].
3 5 2 30

8
Problem 31J / 21S. Find the largest five-digit number with non-zero digits and the following properties:
• The first three digits form a number which is 9 times the number formed by the last two digits.
• The last three digits form a number which is 7 times the number formed by the first two digits.

Result. 85595
Solution. Let abcde be a five-digit number such that abc = 9 · de and cde = 7 · ab. Then we have

63 · de = 7 · abc = 70 · ab + 7c = 10 · cde + 7c = 1007c + 10 · de,

so de = 1007c
53 = 19c. Analogously we get ab = 17c. If c ≥ 6, then the numbers 17c and 19c are greater than 100. It
follows that the maximum possible value of c is 5, i.e. the maximum possible value of 17119c is 85595.

Problem 32J / 22S. Twelve smart men, each of whom was randomly dealt one of twelve cards—nine blank cards
and three special cards labelled J, Q, and K—were sitting in a circle. Everybody looked at his card, and then passed
it to his right-hand neighbour. Continuing in this way, after every look at a card, they were asked to raise their hands
simultaneously if they knew who was holding which special card at that very moment. Nobody raised his hand after
having seen four cards. Exactly one man raised his hand after having seen his fifth card. Next, x men raised their
hand after having seen six cards and y men after seven cards. Determine xy.
Result. 42
Solution. The first man to raise his hand was the first one to see all three special cards. This man must have received
one special card as his fifth card because otherwise he could have raised his hand earlier. Moreover, he must have
received one special card at the very beginning because otherwise his left-hand neighbour would have seen all special
cards after the previous round. Denote C1 the first card, C3 the fifth card, and C2 the remaining special card that the
man got in the meantime. After the subsequent looks at their cards, exactly the persons who encountered C2 and at
least one other card know or can deduce the positions of all three special cards—the positions of C1 and C3 are known
to everyone, but their labels are not. It is easy to see that there are six such men after having seen six cards and seven
after having seen seven cards, so the answer is 42.

Problem 33J / 23S. Within an isosceles right-angled triangle with the length of the base 1, seven circles are
constructed as in the picture:

What is the total area of√the circles?


√ 2
Result. π 3−24 2
= π (1−4 2) = π 4(1+1√2)2 = π 4(3+2
1√
2)
Solution. If an isosceles
√ right-angled triangle is cut into two equal triangles, then they are similar to the original one
with the ratio 1 : 2, and so are the radii of the incircles. Thus each new incircle has half the area of the original
one. In other words, the total area of the incircles is not changed when cutting the big triangle. Hence it is sufficient
to determine the area of the incircle of the big

triangle, radius of which we compute using equal tangents. Since the
2
length of the legs of the big triangle equals 2 by Pythagorean theorem and the perpendiculars from the incenter to

the legs form a square, we get that the radius is 22 − 21 , so the area equals

3−2 2
π .
4

Problem 34J / 24S. Find all primes p such that p + 11 divides p(p + 1)(p + 2).
Result. 7, 11, 19, 79
Solution. As p is a prime, it is either equal to 11 (which clearly satisfies the given condition) or coprime to p + 11. In
the latter case, the product in question is divisible by p + 11 if and only if (p + 1)(p + 2) is. This reduced product
modulo p + 11 equals (−10) · (−9), thus p + 11 | 90. This is satisfied for p ∈ {7, 19, 79}.

9
Problem 35J / 25S. Wood lice (Porcellio scaber) have fourteen legs. Mother wood louse has large supplies of
identical socks and shoes and prepares her children for the upcoming cold season. She explains to little wood louse Jim
that he may put on his socks and shoes in arbitrary order, but he has to bear in mind that on each individual leg he
has to put a sock prior to a shoe. In how many ways can little Jim proceed to dress his feet?
28!
Result. 214
Solution. A way to dress Jim’s legs can be represented by a 28-tuple consisting of 14 socks and 14 shoes in which the
sock belonging to a specific leg is in a position previous to that of the shoe belonging to the same leg. For the first
pair of a sock and a shoe, there are 28 For the second pair, there are 28 − 2 = 26 places, hence 26

2 possibilities. 2
2

possibilities. Continuing this way, there remains only 2 = 1 possibility for the last pair. Therefore the result is
       
28 26 24 2 28!
· · ··· = 14 .
2 2 2 2 2

Problem 36J / 26S. Let x be a real number such that x3 + 4x = 8. Determine the value of x7 + 64x2 .
Result. 128
Solution. It suffices to plug in x3 = 8 − 4x into the given expression as follows:

x7 + 64x2 = x · (x3 )2 + 64x2 = x(8 − 4x)2 + 64x2 = 64x + 16x3 = 16(x3 + 4x) = 128.

Problem 37J / 27S. In an isosceles triangle ABC with base AB, let D be the intersection of angle bisector of
∠ACB with AB and E the intersection of angle bisector of ∠BAC with BC. Given that AE = 2 · CD, find ∠BAC.
Result. 36◦
Solution. Let F be the point on BC such that AE k DF . Now DF = 12 AE = CD, so the triangle F CD is isosceles
with base F C. Denote ϕ = ∠BAC. Then

∠AEC = ∠DF C = ∠F CD = ∠DCA = 90◦ − ϕ.

Finally, since ∠CAE = 12 ϕ, we have (using 4AEC)

1
ϕ + 3(90◦ − ϕ) = 180◦ ,
2
so ϕ = 36◦ .
C

E
F

A D B

Problem 38J / 28S. Given a sequence of real numbers (an ) such that a1 = 2015 and a1 + a2 + · · · + an = n2 · an for
each n ≥ 1, find a2015 .
1
Result. 1008
Solution. By subtracting the recursive formulas for n and n − 1, we get an = n2 · an − (n − 1)2 · an−1 , which can be
simplified to an = n−1
n+1 an−1 . Thus

n−1 n−2 n−3 2 1 2a1


an = · · · · · · · a1 = ,
n+1 n n−1 4 3 n(n + 1)
2·2015 1
and therefore a2015 = 2015·2016 = 1008 .

10
Problem 39J / 29S. Kate and Harriet invented the following game: They colour the faces of two twelve-sided fair
dice with cyan, magenta, and yellow so that each colour is present on at least one face of each die, and there are exactly
four yellow faces on the first die. If they throw the dice and both of these show the same colour, Kate wins, otherwise
Harriet wins. Suppose that the colours are distributed so that the girls have equal chances of winning. How many
magenta faces must be there on the second die?
Result. 1, 9
Solution. Denote by c1 , m1 , y1 , c2 , m2 , y2 the numbers of faces of respective colours on the dice. We know that
c1 + m1 + y1 = 12, c2 + m2 + y2 = 12, and y1 = 4. Moreover, from the total of 122 = 144 possible outcomes of throwing
the dice, exactly one half results in same colours, so
c1 c2 + m1 m2 + y1 y2 = 72.
Expressing the left-hand side in terms of c1 , c2 , and m2 using the relations above, we obtain
c1 c2 − c1 m2 − 4c2 + 4m2 + 48 = 72,
or
(c1 − 4)(c2 − m2 ) = 24.
Using −3 ≤ c1 − 4 ≤ 3 and −9 ≤ c2 − m2 ≤ 9, we deduce that c2 − m2 is either 8 or −8, which together with
0 < y2 = 12 − c2 − m2 yields that m2 is either 1 or 9. A straightforward check shows that both these values are possible.

Problem 40J / 30S. There are n > 24 women sitting around a great round table, each of whom either always lies or
always tells the truth. Each woman claims the following:
• She is truthful.
• The person sitting twenty four seats to her right is a liar.
Find the smallest n for which this is possible.
Result. 32
Solution. Let us pick a woman, then the one 24 seats to her right, then the next one 24 seats to the right etc. After
a certain number of such steps, say s, we get back to the original woman. This s is readily seen to be the smallest
positive integer such that 24s is a multiple of n. Hence s = n/d, where d is the greatest common divisor of n and 24.
Observe that the truthfulness of the women has to alternate, i.e. every woman is truthful if and only if the one 24
seats to her right is a liar. If s were odd, this would lead to a contradiction. Therefore n has to be divisible by a higher
power of 2 than 24 is. The smallest candidate for n is 32, and it is easy to check that this number satisfies the given
conditions.

Problem 41J / 31S. Two squares have a common center and the vertices of the smaller one lie on the sides of the
bigger one. If the small square is removed from the big one, it decomposes into four congruent triangles, the area of
each being one twelfth of the area of the big square. What is the size in degrees of the smallest internal angle of the
triangles?

Result. 15◦
Solution. Denote by a, b the legs and by c the hypotenuse of the remaining triangles, and assume a ≤ b. A
straightforward computation shows that the area of the small square equals two thirds of the area of the big square.
Hence the area of one of the triangles is one eighth of the area of the small square, that is
1 1
ab = c2 .
2 8
Let α be the smallest internal angle in the triangle. Then a = c sin α and b = c cos α, so
c2 = 4ab = 4c2 sin α cos α = 2c2 sin 2α,
or
1
sin 2α = .
2
Since α ≤ 45◦ , it follows that 2α = 30◦ , and finally, α = 15◦ .

11
Problem 42J / 32S. A circle ω3 of radius 3 is internally tangent to circles ω1 , ω2 of radii 1 and 2, respectively.
Moreover, ω1 and ω2 are externally tangent. Points A, B on ω3 are chosen so that the segment AB is a common
external tangent of ω1 and ω2 . Find the length of AB.

Result. 34 14
Solution. Denote O1 , O2 , O3 the centers of ω1 , ω2 , ω3 , respectively, and let T1 , T2 , T3 be the feet of perpendiculars from
O1 , O2 , O3 to the segment AB (hence T1 and T2 are the points of tangency of AB to ω1 , ω2 ). Since O1 T1 k O2 T2 k O3 T3 ,
O1 T1 = 1, O2 T2 = 2, and O1 O3 = 2O2 O3 , where O1 , O2 , O3 are collinear, we easily get O3 T3 = 53 (consider similar
triangles). Applying the Pythagorean theorem to the triangle AO3 T3 , we conclude that
s  2
5 4√
AB = 2AT3 = 2 32 − = 14.
3 3

O1 O3 O2

A
T1
T3
T2
B

Problem 43J / 33S. One day, Oedipus, an intrepid hero, met the Sphinx, who gave him the following riddle: She
chose a positive two-digit integer S. Oedipus could choose three one-digit integers a < b < c and ask if S was divisible
by them. For each of these numbers, he got an answer (either ‘yes’ or ‘no’). Oedipus fell into despair as there were
exactly two numbers satisfying the divisibility conditions. But then the Sphinx told him that she had been wrong
about the divisibility by b, which fortunately allowed Oedipus to find S with certainty. What was the value of S?
Result. 84
Solution. There are clearly three two-digit numbers satisfying both given answers about divisibility by a and c (two
corresponding to the Sphinx’ first answer and one after the correction). Moreover, both these answers have to be
positive since a negative answer would result in more than three eligible two-digit numbers. Hence these numbers are
multiples of lcm(a, c) = m. It follows that 25 ≤ m ≤ 33, but only two numbers within this range are the least common
multiples of two digits, namely 28 = lcm(4, 7) and 30 = lcm(5, 6). The latter is impossible because of c − a ≥ 2, so
we have a = 4 and c = 7. Now if b were 5, then there would be no two-digit number being divisible by a, b, and c
simultaneously. In the case b = 6, the negative answer about b corresponds to the numbers 28 and 56, while the positive
answer yields S = 84.

Problem 44J / 34S. Four people were moving along a road, each of them at some constant speed. The first one
was driving a car, the second one was riding a motorcycle, the third one was riding a Vespa scooter, and the fourth
one a bicycle. The car driver met the Vespa at 12 noon, the bicyclist at 2 p.m., and the motorcyclist at 4 p.m. The
motorcyclist met the Vespa at 5 p.m., and the bicyclist at 6 p.m. At what time did the bicyclist meet the Vespa
scooter?
Result. 3:20 p.m.
Solution. Since the time in question does not depend on the chosen reference frame, we may assume that the car is
not moving at all. Under this assumption, the motorcycle needed one hour from where it met the car to its meeting
point with the Vespa, whereas the Vespa needed five hours for the same distance, thus the motorcycle was five times
faster. Similarly, one may deduce that the motorcyclist was twice as fast as the bicyclist, hence the ratio of the speeds
of the Vespa and the bicycle was 2 : 5.
If the Vespa needed t hours to get from the car to its meeting point with the bicyclist, then the bicyclist needed
t − 2 hours. The ratio of these required times equals the inverse of the ratio of the speeds, so
t−2 2
= ,
t 5
or t = 10/3. Finally, using the fact that the Vespa met the car at 12 noon, we infer that it met the bicyclist at 3:20 p.m.

12
Problem 45J / 35S. The floor of a hall is covered with a square carpet of side length twenty-two meters. A robotic
vacuum cleaner (robovac) is given the task to clean the carpet. For its convenience, the carpet is divided into 484 unit
squares. The robovac cleans one square after another in accordance with the following rules:
• Once it has vacuumed a square, it must not move onto it again.
• It keeps moving in one direction unless it is forced to change it by approaching the edge of the carpet or a
previously cleaned square.
• If it has to change the direction and there are two options, it may choose whichever it prefers.
At the beginning, the robovac is placed on one square and it may choose any admissible direction. For how many
starting squares is the robovac able to clean the whole carpet if it does not need to finish its work on the edge?
Result. 20
Solution. If the robovac does not start in one of the corner 3 × 3 ‘multisquares’, there always remains a non-cleaned
part of the carpet—when the robovac leaves the edge of the carpet (i.e. no later than in its seventh move), it divides
the remaining non-cleaned squares into two separate regions. We can routinely verify that the squares with coordinates
(1, 2), (2, 1), (2, 3), (3, 2) and their symmetric counterparts in the other corners do not satisfy the given conditions for
similar reasons. However, those with coordinates (1, 1), (2, 2), (3, 3), (3, 1), (1, 3) do. Thus there are 4 · 5 = 20 possible
starting squares in total.

Problem 46J / 36S. Let the points A, B, C, D, E, F lie clockwise in this order on a circle ω. Assume further
that AD is a diameter of ω, BF intersects AD and CE in G and H, respectively, ∠F EH = 56◦ , ∠DGB = 124◦ , and
∠DEC = 34◦ . Find ∠CEB.
Result. 22◦
Solution. The inscribed angle theorem implies that ∠CEB = ∠CDB, so let us compute ∠CDB. Since the quadrilateral
BCEF is cyclic, we have

∠F BC = 180◦ − ∠F EC = 180◦ − (180◦ − ∠F EH) = 56◦ .

Moreover, ∠DGF = 180◦ − ∠DGB = 56◦ , so AD and BC are parallel. This implies ∠ADB = ∠DBC and using
inscribed angles we further infer ∠DBC = ∠DEC = 34◦ . Since ∠ABD = 90◦ (Thales’ theorem) and trapezoid ABCD
is cyclic, we obtain
∠ADC = 180◦ − ∠ABC = 180◦ − (∠ABD + ∠DBC) = 56◦ ,
and finally
∠CEB = ∠CDB = ∠ADC − ∠ADB = 56◦ − 34◦ = 22◦ .

13
A

H
E
C

Problem 47J / 37S. Ten people—five women and their husbands—took part in E events. We know that no married
couple took part in the same event, every pair of non-married people (including same-sex pairs) took part in exactly
one event together, and one person attended only two events. What is the smallest E for which this is possible?
Result. 14
Solution. Denote the couples (a1 , a2 ), (b1 , b2 ), (c1 , c2 ), (d1 , d2 ), (e1 , e2 ). We may w.l.o.g. assume that a1 is the person
who attended only two events and that the first of these events was also attended by b1 , c1 , d1 , and e1 , whereas
the second one by their counterparts. Every other event could have been attended by at most two people from
b1 , b2 , . . . , e1 , e2 simultaneously, so there had to be at least 12 other events. If further a2 takes part in any disjoint four
of these events, we obtain the situation described in the statement, thus the smallest value of E is 14.

Problem 48J / 38S. Students were given a three-digit number abc with 0 < a < b < c. The task was to multiply it
by 6 and then swap the tens digit with the hundreds digit. Alex made a mistake and performed the swap before the
multiplication, but the result turned out to be correct! Find abc.
Result. 678
Solution. Since 0 < a < b, we have b ≥ 2 and 6 · bac > 1200, so the result obtained by Alex must be a four-digit
number, say 6 · bac = def g. On the other hand, we know that 6 · abc = df eg. Subtracting these two equalities, we get
6(bac − abc) = def g − df eg.
Thus 540(b − a) = 90(e − f ), or 6(b − a) = e − f . As e, f are digits, we have e − f ≤ 9. It follows that e − f = 6 and
b − a = 1 (b − a > 0 because b > a). Substituting b = a + 1 in 6 · bac = def g gives
def g = 6(100(a + 1) + 10a + c) = 660(a + 1) − 6(10 − c).
This means that def g must differ from some multiple of 660 by a non-zero multiple of 6 not exceeding 42 (c ≥ 3).
Remembering that e − f = 6, and considering successively a = 1, 2, . . . , 7, we determine all candidates for the number
def g: 1938, 2604, 3930, 3936, 4602, 4608. Only for def g = 4608 the number bac = def g/6 = 768 turns out to have
distinct non-zero digits. Now we can directly check that indeed 6 · abc = 6 · 678 = 4068 = df eg.

Problem 49J / 39S. Consider a 5 × 3 grid. A mouse sitting in the upper left corner wants to reach a piece of cheese
in the lower right corner, whereas a crab sitting in the lower left corner wants to reach algae in the upper right corner.
They move simultaneously. Every second, the mouse moves one square right or down, and the crab moves one square
right or up. In how many ways can the animals reach their food so that they do not meet at all?

14
Result. 70
Solution. Observe that the animals may meet only in the middle row. Furthermore, the path of each animal is
completely determined by the visited squares in the middle row. It is easy to see that the animals do not meet if these
middle parts of their paths do not intersect, so we are looking for the number of pairs of disjoint segments of the middle
row.
Let us first handle the case when there is at least one unused square between the segments. Then the number of
pairs may be computed as 2 · 64 , since one chooses four out of six edges of squares in the middle row and then chooses
the animal which uses the segment between the first two of the chosen edges (the second animal uses the segment
between the second two edges). If there is no gap between the segments, we obtain 2 · 63 pairs since the segments are


described by three edges. 


In total, there are 2 · 64 + 2 · 63 = 70 possible ways.



Problem 50J / 40S. Find the number of positive integers n not exceeding 1000 such that the number b 3 nc is a
divisor of n.
Note: The symbol bxc denotes the integral part of x, i.e. the greatest integer not exceeding x.
Result. 172

Solution. Observe that b 3 nc = k if and only if k 3 ≤ n ≤ (k + 1)3 − 1. Out of the 3k 2 + 3k + 1 numbers in this range,
every k-th is divisible by k starting with k 3 , thus there are 3k + 4 such numbers. It remains to sum this expression for
all numbers k such that (k + 1)3 − 1 ≤ 1000 (i.e. k ≤ 9) and add one for the number 1000, which satisfies the given
condition as well. Hence the desired result is
9
X 9 · 10
1+ (3k + 4) = 1 + 9 · 4 + 3 · = 172.
2
k=1

Problem 51J / 41S. Find all real m such that the roots of the equation
√ √
x3 − 15 2x2 + mx − 195 2 = 0

are side lengths of a right-angled triangle.


Result. 281/2
Solution. Let a, b, c be the roots of the given equation, which are also side lengths of a right-angled triangle. Assume
w.l.o.g. that 0 < a, b < c, hence by Pythagoras’ theorem, the equation a2 + b2 = c2 holds. By Vieta’s formulas (or
working out (x − a)(x − b)(x − c) and then comparing coefficients), we get
√ √
15 2 = a + b + c, m = ab + ac + bc, 195 2 = abc.
√ √ √
Squaring 15 2 − c = a + b leads to 450 − 30 2c = 2ab. After multiplying by c and substituting abc = 195 2, we get
the quadratic equation √ 2 √
2c − 15c + 13 2 = 0,
√ √ √ √
having roots c1 = 2 and c2 = 13 2/2. Since the constraints 0 < a, b < c and abc = 195 2 allow only c = 13 2/2,
the desired number m can be calculated via
1 1
m = ab + ac + bc = · ((a + b + c)2 − 2c2 ) = · 450 − c2 = 281/2.
2 2

Problem 52J / 42S. There are green and red apples in a basket, at least one red and two green ones. The probability
that a randomly chosen apple is red is 42 times higher than the probability that two randomly chosen apples (without
replacement) are both green. How many green apples and how many red apples are there in the basket?
Result. 4 green and 21 red
Solution. Let g be the number of green apples and r the number of red apples. The statement may be rewritten as
r g · (g − 1)
= 42 · ,
g+r (g + r) · (g + r − 1)
equivalently
r2 + (g − 1)r − 42g · (g − 1) = 0.
The latter can be viewed as a quadratic equation with variable r and parameter g . The discriminant

(g − 1)2 + 168g · (g − 1) = 169g 2 − 170g + 1

15
must be a square of an integer, otherwise the roots would be irrational. Due to g ≥ 2, we obtain the inequalities

(13g − 6)2 = 169g 2 − 156g + 36 > 169g 2 − 170g + 1 > 169g 2 − 208g + 64 = (13g − 8)2 .

Therefore 169g 2 − 170g + 1 has to be equal to (13g − 7)2 , which implies 12g = 48, or g = 4. Then the roots of the
quadratic equation are −24 and 21, but since r > 0, we get r = 21 as the only solution. Hence there are 21 red and 4
green apples in the basket.

Problem 53J / 43S. Gilbert Bates, a very rich man, wants to have a new swimming pool in his garden. Since he
likes symmetry, he tells the gardener to build an elliptical-shaped pool within a 10 m × 10 m square ABCD. This
elliptical pool should touch all four sides of the square, particularly the side AB at point P which is 2.5 m away from
A. The gardener, who knows very well how to construct an ellipse if the foci and a point on the ellipse are given,
reflects that due to symmetry he only needs the distance of the foci in this case. Can you help him by computing the
distance of the foci in meters?
D C

P
A B

Result. 10
Solution. We solve the problem in a more general way. Let ABCD be a square of side length 1 with corner A in the
origin of a coordinate system. Point P is on AB and has coordinates (b, 0) with 0 < b < 12 . If focus F1 has coordinates
(f, f ), then focus F2 has coordinates (1 − f, 1 − f ) (both foci have to lie on the diagonal AC symmetrically with respect
to the other diagonal).
D C

F2

g2

g1

F1
αα

A P B

Now line g1 through P and F1 is given by


f
y = (x − b) · ,
f −b
and line g2 through P and F2 by
f −1
y = (x − b) · .
b+f −1
Since the line through P perpendicular to the tangent AB bisects ∠F2 P F1 , the gradient of g2 has to be the negative of
the gradient of g1 . Therefore we get
f f −1
= (−1) · ,
f −b b+f −1
which can be simplified to
b
f 2 − f + = 0.
2
1
√ 
This quadratic equation has two solutions f1,2 = 2 1 ± 1 − 2b , each one corresponding to one focus of the ellipse.

The distance of the foci can now be computed by Pythagoras’ theorem, yielding 2 − 4b. Setting b = 14 and scaling up
by 10 gives the desired result F1 F2 = 10.

16
Alternative solution. Let us shrink both the square and the ellipse along AC so that the ellipse becomes a circle and
denote the new points with a prime. Further, let S be the midpoint of AC and T the midpoint of A0 B 0 .
C

C0

D = D0 S B = B0

P0 T

A0
P

A
1 0 0
Since A0 P 0 = 4A B , we have A0 P 0 = P 0 T , so SA0 = ST . It follows that A0 C 0 = B 0 C 0 and that the triangle A0 B 0 C 0 is
equilateral. Hence the factor of shrinking is √
A0 C 0 3
= .
AC 3
It is easy to compute that the radius of the circle (which coincides with the length b of the minor semi-axis of the
ellipse) equals 14 AC. The length a of the major semi-axis is obtained by dividing the minor semi-axis by the factor of

shrinking. Knowing the lengths a and b, it remains to compute the eccentricity using the formula e = a2 − b2 and
multiply it by two to get the distance of the foci.

Problem 54J / 44S. A sequence (an ) is given by a1 = 1, and an = b a1 + a2 + · · · + an−1 c for n > 1. Determine
a1000 .
Note: The symbol bxc denotes the integral part of x, i.e. the greatest integer not exceeding x.
Result. 495
Solution. By writing down the first few members (1, 1, 1, 1, 2, 2, 2, 3, 3, 4, 4, 4, 5, 5, 6, 6, 7, 7, 8, 8, 8, 9, 9, . . . ), we discover
that number 1 repeats four times, while numbers greater than 1 seem to appear only twice or thrice. Let us prove by
induction that those appearing three times are exactly natural powers of 2.
Suppose we have written down the beginning of the sequence up to the first occurrence of n (n > 1) and assume it
behaves as described above. Let k be the largest integer such that 2k < n. Then the sum of all written members equals
s1 = (1 + 2 + · · · + n) + (1 + 2 + · · · + n − 1) + (1 + 2 + 22 + · · · + 2k ) + 1 = n2 + 2k+1 .

Since 2k+1 = 2 · 2k < 2n < 2n + 1 = (n + 1)2 − n2 , we have s1 < (n + 1)2 , and hence the following member is b s1 c = n.
Let us determine the next number. Now the sum is s2 = s1 + n = n2 + n + 2k+1 . If 2k+1 < n + 1, then again
s2 < (n + 1)2 and the following member is n. However, k is the largest integer satisfying 2k < n, so it holds 2k+1 ≥ n.
Therefore this case occurs exactly when 2k+1 = n. If n is not a natural power of 2, then the next member is n + 1
because n + 1 ≤ 2k+1 < 2n < 3n + 4, which is equivalent to (n + 1)2 ≤ n2 + n + 2k+1 < (n + 2)2 .
It remains to show that if n = 2k+1 , then there comes n + 1 after three occurrences of n. This is easy because if we
compute the next sum s3 = s2 + n = n2 + 2n + 2k+1 = n2 + 3n, we immediately get (n + 1)2 < s3 < (n + 2)2 . Thus
the induction step is complete and we get a1000 = 495 (since 500 = a1010 = a1009 ).

Problem 55J / 45S. Determine the number of 4 × 4 tables with non-negative integer entries such that
• every row and every column contains at most two non-zero entries,
• for each row and column, the sum of its entries is 3.
Result. 576
Solution. Observe that every such table uniquely decomposes into an entrywise sum of two tables, the first one
having exactly one 1 in every row and every column, and the second one exactly one 2 in every row and every column.
Conversely, having such a pair of tables, adding them entrywise results in a table satisfying the given criteria. Therefore
(4!)2 = 576 is the desired result.

17
Problem 1. Given is a cubical-shaped boulder with an original volume of 216 m3 . What size is the surface of the
boulder in m2 after knocking out a cuboidal block of dimensions 1 m × 1 m × 2 m as shown in the picture below?

Result. 216
Solution. Since 63 = 216, the side length of the cube is 6 m. The missing block does not change the surface of the
cube, hence the area of the surface is 6 · 62 = 216 m2 .

Problem 2. The two friends Christoph and Jonas hit the jackpot and bought a nice rectangular property of
dimensions 35 m by 25 m. They are planning to build a twin house and share garden G of size 300 m2 . The building
floor plan can be seen in the picture:

G
b

(The distance between two neighboring grid lines is 5 m.) How far must wall b intrude from one section of the twin
house into the other one so that the base areas of the friends’ parts are equal?
Result. 8.75 m
Solution. The base area of one house is half of 35 m · 25 m − 300 m2 = 575 m2 , that is 287.5 m2 . Since one side length
of the rectangular house is 10 m, the other side length must be 28.75 m. Therefore, we get b = 8.75 m.

Problem 3. Little Marcus wants to go to the beach. He owns the following distinguishable beach-outfits: 5 swimming
trunks, 3 straw hats, 4 sunglasses and 5 T-shirts. To comply with the beach rules, he has to wear swimming trunks.
Wearing sunglasses, hats and T-shirts is not obligatory at all, but if he puts on some outfit, he always takes at most
one of each category. How many different ways are there for Marcus to appear in an appropriate outfit?
Result. 600
Solution. Observe that the option of wearing nothing can be viewed as an additional outfit. Considering the hats,
Marcus has to choose between not wearing a hat at all, wearing the first hat, the second one, or the third one, which
gives 4 possibilities in total for the hats. Similarly, there are 5 possibilities to wear the sunglasses and 6 possibilities
of wearing a T-shirt. Since Marcus has to put on one of the 5 swimming trunks, in total he has 5 · 4 · 5 · 6 = 600
possibilities to appear in an appropriate outfit.

Problem 4. Laura spent her vacation in a rain forest. Each day it either rained in the morning, or it rained in
the afternoon, or it rained the whole day. Laura enjoyed altogether 13 days, when it did not rain all the time, but
experienced exactly 11 morning rains and 12 afternoon rains. How long was Laura’s vacation?
Result. 18 days
Solution. Let v be the number of days of Laura’s vacation. Then v − 11 is the number of days when it did not rain in
the morning, and similarly v − 12 is the number of days when it did not rain in the afternoon. Since there was no day
without any rain, we see that
(v − 11) + (v − 12) = 13
or v = 18.

Problem 5. Find the smallest non-negative integer solution of the equation n − 2 · Q(n) = 2016, where Q(n) is the
sum of the digits of n.
Result. 2034
Solution. The number n − Q(n) is always divisible by 9. Since 2016 is divisible by 9, also Q(n) and consequently n
have to be divisible by 9. Clearly n < 3000, so Q(n) ≤ 2 + 9 + 9 + 9, thus n = 2016 + 2 Q(n) ≤ 2074. Now the only
solution 2034 can easily be found.

1
Problem 6. How many positive integers have the property that their first (i.e. leftmost) digit is equal to their number
of digits?
Result. 111 111 111
Solution. If n is a non-zero digit, then there are exactly 10n−1 numbers starting with n and fulfilling the property
from the statement, for these are precisely the integers between n0 . . . 0 and n9 . . . 9. We conclude that there are

1 + 10 + · · · + 100 000 000 = 111 111 111

such numbers in total.

Problem 7. A paving consists of many pavers, one of which has the shape of a regular n-gon, completely surrounded
by other pavers. When this paver is rotated by 48◦ about its center, it fits again in its former position. What is the
minimal n for which this is possible?
Result. 15
Solution. A regular n-gon is preserved by a rotation precisely if it is by a multiple of the angle between the segments
connecting two consecutive vertices with the center. This angle is 360◦ /n, so we seek the smallest positive n such that
48 2
360 = n
n
15

is an integer. The result is n = 15.

Problem 8. A day is called happy if its date written in the format DD.MM.YYYY consists of eight distinct
digits—here DD fills in for the day, MM for the month and YYYY for the year, and if the day and the month is less
than 10, a leading zero is prepended. For example, 26.04.1785 was a happy day. When is the next happy day (from
now) going to be?
Result. 17.06.2345
Solution. The month of a happy day either contains a zero or is 12, so either the year does not contain a zero or
exceeds 3000. Let us pursue the former case. Since the leading digit of the day is one of 0, 1, 2, 3, we see that the year
has to be at least 2145. However, this implies that the day is 30, which collides with the month. The second smallest
possible year is 2345. We shall show that there is a happy day in that year. The leading digit of the day has to be 1, so
the first possible month is 06. Finally, setting the day to be 17 is enough to complete the date of the happy day.

Problem 9. How many different planes contain exactly four vertices of a given cuboid?
Result. 12
Solution. There are six planes containing the faces of the cuboid, and further for each pair of opposite faces, there are
two planes perpendicular to these faces and containing a face diagonal. In total there are 12 planes.

Problem 10. Little Sandra wants to draw a beautiful crescent using ruler and compass. First of all, she draws a
circle with center M1 and radius r1 = 3 cm. Then she sets the compass at a point M2 of this circle and draws a second
circle with radius r2 which meets the first circle in antipodal points of a diameter through M1 , as shown in the picture
below.

r2
r1
A M2
M1

What is the area of the crescent A in cm2 ?


Result. 9
Solution. To get the area of A, we have to subtract the area of the circular segment (center M2 , radius r2 ) from the
area of the semicircle (center M1 , radius r1 ). For the area of the segment we calculate the area of the quadrant with
radius r2 and subtract the area of the isosceles right-angled triangle with leg r2 . Using the fact that r22 = 2r12 (the
Pythagorean theorem), we infer that the sought area is

πr12
 2 
πr2
− − r12 = r12 = 9 cm2 .
2 4

2
Problem 11. All servants of King Octopus have six, seven, or eight legs. The ones having seven legs always lie,
whereas the ones having six or eight legs always tell the truth. One day, King Octopus assembled four of his servants
and asked them how many legs the four of them had altogether. The first servant reported that the total number of
legs was 25, the next one claimed 26, the third one said 27 and the last one 28. How many legs do the the king’s
truth-telling servants (among these four) have in total?
Result. 6
Solution. Only one of the answers may be correct, so there are either three or four liars among the servants. However,
if there were four of them, they would have 28 legs in total, implying that the last servant did not lie–a contradiction.
So, the lying servants have 21 legs altogether. If the sole truth-telling servant had eight legs, the total number would
be 29, which is not among the answers. We deduce that the truth-teller had six legs (and it was the third one to report
the number of legs).

Problem 12. A shop sells bars of milk, white, and dark chocolate for the same price. One day, the shop earned 270
for the sold milk chocolate, 189 for the white chocolate, and 216 for the dark chocolate. What is the smallest total
number of chocolate bars the shop could have sold on that day?
Result. 25
Solution. The price of a single chocolate bar is a common divisor of the amounts in the statement. Should the
number of bars be minimal, the price has to be the greatest possible, i.e. the greatest common divisor. Since
GCD(270, 189, 216) = 27, we infer that the total number of sold bars is
270 189 216
+ + = 25.
27 27 27

Problem 13. A father of five children wants to have pastries for his family for tea time. Based on painful experience
he knows that he has to distribute either the same type or five different types of pastry to his children, or else all kinds
of heavy dispute will arise among the kids. One day, after a long discussion without any consent on the type of pastries,
he exasperatedly instructed his youngest daughter Anna: “You’ll go to the pastry shop and ask the salesgirl to give
you x pieces of pastries randomly! After you return home, each of the children shall get exactly one piece of pastry and
the remaining pieces will be for mom and dad!” Assuming that the shop sells more than five types of pastry and it
is always well stocked with every type, what number x did the father choose in order to keep the peace among his
children in any case and to keep the costs as low as possible at the same time?
Result. 17
Solution. If Anna ordered 16 or less pieces of pastry to be taken randomly, trouble among the kids could arise: For
example in case of receiving 4 brownies, 4 blueberry muffins, 4 honey scones, and 4 danish, or less pieces of any of
these, there are neither five pieces of the same type nor five pieces different from each other. On the other hand, if
she asks for 17 random pieces, there might be five or more different types of pastry and the children would be happy.
Otherwise there are at most four different types; however, in such a case, if there were less than five pieces of each type,
there would be at most 4 · 4 = 16 pieces in total—a contradiction. We conclude that the father suggested Anna to ask
the salesgirl for 17 randomly chosen pieces of pastry.

Problem 14. What is the ratio of the area of a circle to the area of a square, perimeters of which are equal?
Result. 4 : π
Solution. Let r be the radius of the circle and a be the side length of the square. Since 2πr = 4a, we compute the
ratio of areas as
πr2 2r · πr 2r · 2a 4
= = = .
a2 a · 2a a · πr π
Problem 15. In February, Paul decided to visit the Cocos Islands with his private jet. He took off from his mansion
in Europe at 10:00 Central European Time (CET) and landed on the Islands the next day at 5:30 local time (Cocos
Islands Time, CCT). When returning home, he started at 8:30 CCT and landed at 17:00 CET the same day. Assuming
that the duration of the flight was the same in both cases, what was the time on the Cocos Islands when Paul returned
home?
Result. 22:30
Solution. Let d be the duration of the flight and s the difference between the time in Europe and on the Cocos Islands
(in hours). The statement may be rewritten as the system of equations
d + s = 19.5,
d − s = 8.5
with the solution d = 14, s = 5.5. We deduce that Paul returned home at 22:30 of the Cocos Islands Time.
Note: The Cocos Islands indeed use the time zone GMT+6:30.

3
Problem 16. The numbers 14, 20, and n fulfill the following condition: Whenever we multiply any two of them, the
result is divisible by the third one. Find all positive integers n for which this property holds.
Result. 70, 140, 280
Solution. Since n divides 14 · 20 = 23 · 5 · 7, only the primes 2, 5, and 7 may occur in the factorization of n, with 5
and 7 occurring at most once and 2 at most three times. Further, from 14 | 20n wee see that n is a multiple of 7, and
similarly, 20 | 14n implies 10 | n, so 70 | n. It remains to conclude that all of the possible numbers 70, 140, 280 fulfill
the conditions from the statement.

Problem 17. A rectangle is divided into two trapezoids along the line segment x as in the picture below. The
distance P A is 10 cm and AQ is 8 cm. The area of the trapezoid T1 is 90 cm2 and the area of T2 is 180 cm2 .

P A Q

T1
x T2

What is the length of the segment x in cm?


Result. 17
Solution. Denote by R, S the other two vertices of the rectangle, B the other endpoint of x, and M the point on SR
such that SM = P A = 10.
P A Q

T1
x T2

S B M R
As P Q = 18 and the area of rectangle P QRS is 180 + 90 = 270, it follows that P S = QR = 270/18 = 15. The formula
for the area of trapezoid T2 states that
1
180 = (BR + AQ) · QR
2
or BR = 16. Now BM = BR − M R = 8 and using the Pythagorean theorem,
p √
x = AM 2 + BM 2 = 289 = 17.

Problem 18. Elisabeth has harvested strawberries in her garden. She wants to distribute them to her four sons
in such a way that each son gets at least three strawberries and Valentin receives more strawberries than Benedikt,
Benedikt more than Ferdinand, and Ferdinand more than Michael. Each son knows his number of strawberries, the
total number of strawberries distributed, and the above-mentioned conditions. How should Elisabeth distribute the
strawberries in order to hand as few as possible of them and none of her sons is able to determine the whole distribution?
Result. (M, F, B, V ) = (3, 5, 6, 8)
Solution. Let us denote by V the number of Valentin’s strawberries; obviously, V ≥ 6. By analysing cases, we will
show that Elisabeth cannot distribute less than 22 strawberries. If V = 6, there is only one possible distribution,
namely (3, 4, 5, 6). In the case V = 7, each of the possible distributions (3, 4, 5, 7), (3, 4, 6, 7), (3, 5, 6, 7), and (4, 5, 6, 7)
uses a different number of strawberries, therefore Valentin, knowing the total number, can determine the distribution.
Similarly, for V = 8 or V = 9, only the distributions (3, 4, 5, 8), (3, 4, 6, 8), and (3, 4, 5, 9) exist (with less than 22
strawberries), each one being computable by Valentin.
On the other hand, the distribution (3, 5, 6, 8) satisfies all the conditions: Valentin and Michael cannot distinguish
it from (3, 4, 7, 8), whereas Ferdinand and Benedikt can think that the distribution is (4, 5, 6, 7). It remains to show
that no other distribution of 22 strawberries complies with the requirements: From (3, 4, 5, 10), (3, 4, 6, 9), (3, 4, 7, 8),
and (4, 5, 6, 7), the third one can be deduced by Benedikt and the remaining three by Valentin.

4
Problem 19. We write all the integers from 1 to 1000 consecutively clockwise along the circumference of a circle.
We now mark some of the numbers: Starting with 1, go clockwise and mark every 15th number (i.e. 16, 31 etc.). We
continue this way, until we are forced to mark a number which we have already marked. How many numbers stay
unmarked at the end of the procedure?
Result. 800
Solution. In the first pass, all the numbers of the form 15k + 1 (for some integer k) are marked, starting with 1 and
ending with 991; the following pass starts with 6 and ends with 996, marking all the numbers of the form 15k + 6.
Finally, in the third pass one begins with the number 11 and ends with 986 (the marked numbers having the form
15k + 11), which is the last number to be marked. Observe that we have marked exactly all the numbers of the form
5k + 1, which comprise precisely one fifth of all the numbers on the circle. We conclude that 4/5 · 1000 = 800 numbers
remain unmarked.

Problem 20. Find the sum of the seven marked interior angles of this 7-pointed star (in degrees)!

Result. 540◦
Solution. Denote the tips of the star by A, B, . . . , G as in the picture; further, let X, Y be the intersection of DE
with AB, AG, respectively.
G C

D
B
Y
X
A E
Let S be the sum in question. Since the sum of the internal angles in both quadrilaterals XBCD and Y EF G is 360◦ ,
we see that
S + ∠BXY + ∠XY G − ∠XAY = 2 · 360◦ .
However, ∠BXY = 180◦ − ∠AXY and ∠XY G = 180◦ − ∠XY A, so
∠BXY + ∠XY G − ∠XAY = 360◦ − (∠AXY + ∠XY A + ∠XAY ) = 180◦ .
It follows that S = 540◦ .

Problem 21. Pupils were given the following exercise: They should compute the arithmetic mean of the numbers 1,
3, 6, 7, 8, and 10. However, Lucy chose a wrong approach: First she picked two of the numbers and computed their
arithmetic mean. Then she computed the arithmetic mean of the result and some other number and repeated this step
until she had used all the numbers. What is the largest absolute value of the error (i.e. the difference with the correct
result) Lucy could have achieved?
Result. 17/6
Solution. One can easily see that Lucy’s procedure is in fact the following: She picks some ordering of the given
numbers, say (a1 , a2 , a3 , a4 , a5 , a6 ), and computes
a1 a2 a3 a4 a5 a6
S = 5 + 5 + 4 + 3 + 2 + 1.
2 2 2 2 2 2
Among all these orderings, the highest value of S is achieved for the ascending ordering, since the largest number is
divided by the smallest, the second largest by the second smallest etc. Similarly, the smallest value of S is achieved for
the descending ordering. Obviously, the largest error will occur for one of these extremal orderings. The arithmetic
mean of the given numbers is 35/6. If the ordering is chosen ascending, we get S = 67/8, yielding the error 61/24. If
the ordering is descending, we obtain S = 3 with the error 17/6, which is the greater of the two and thus the sought
result.

5
Problem 22. Along one side of a straight road there are five street lights L1 , L2 , L3 , L4 , and L5 lined up equally
spaced 12 m apart. On the other side of the road there is an ice cream shop. If Julien is standing at the entrance E of
the shop, the angle subtended at this point by L1 and L2 is α = 27◦ . If he is standing at L5 , the angle at that point
subtended by L1 and E is 27◦ , too.
L5 L4 L3 L2 L1
α

E
What is the distance from L1 to E?
Result. 24 m
Solution. The triangles EL1 L2 and EL1 L5 are similar, since α and ∠L5 L1 E occur as interior angles in both of them.
Therefore, we get
EL1 L5 L1
= or EL21 = L2 L1 · L5 L1 = 12 · 48 = 576,
L2 L1 EL1
yielding the desired distance EL1 = 24 m.

Problem 23. Clara chose two distinct integers from 1 to 17, inclusive, and multiplied them. Surprisingly, the product
turned out to be equal to the sum of the remaining fifteen numbers. Find Clara’s two numbers.
Result. 10 and 13
Solution. Denote by a and b the numbers fulfilling the given condition. As the sum of the first 17 numbers is 153,
we have to solve the equation 153 − (a + b) = ab. Rearranging and adding 1 on both sides of the equation leads to
154 = ab + a + b + 1 = (a + 1)(b + 1). Since 154 = 2 · 7 · 11 and 2 ≤ (a + 1), (b + 1) ≤ 18, the only possible factorization
is 154 = 11 · 14. Therefore, the sought-after numbers are 10 and 13.

Problem 24. How many 6-tuples (a, b, c, d, e, f ) of positive integers satisfy a > b > c > d > e > f and a + f =
b + e = c + d = 30 simultaneously?
Result. 14

3 = 364
Solution. Let us express the 6-tuple as
(a, b, c, d, e, f ) = (15 + x, 15 + y, 15 + z, 15 − z, 15 − y, 15 − x),
where 0 ≤ x, y, z < 15. The condition a > b > c > d > e > f is equivalent to x > y > z > 0, so the 6-tuple is uniquely
determined by a choice of three positive integers less than 15. It follows that there are 14
3 = 364 such 6-tuples.

Problem 25. A timed bomb is equipped with a display showing the time before the explosion in minutes and seconds.
It starts counting down with the value 50:00 on the display. A light bulb blinks whenever the displayed number of
remaining minutes is equal to the displayed number of remaining seconds (e.g. 15:15) or when the four digits on the
display read the same when reversed (e.g. 15:51). We can disable the bomb when the light blinks for the 70th time.
What will be the time on the display then?
Result. 03:03
Solution. The number of minutes is equal to the number of seconds once in each minute, so in 50 minutes this
happens 50 times. The event when the number on the display reads the same occurs once in each minute with the
units digit not exceeding 5, so this happens 30 times. There are five cases when both these events happen at once:
00:00, 11:11, . . . , 44:44. Therefore the light bulb blinks 50 + 30 − 5 = 75 times (including at 00:00) before the bomb
explodes; we can disable the bomb when there are only five blinks remaining (00:00, 01:01, 01:10, 02:02, 02:20), i.e.
when there is 03:03 shown on the display.

Problem 26. Five circles are tangent to each other as indicated in the figure. Find the radius of the smallest circle,
if the radius of the big circle is 2 and the two other circles with marked centers are of radius 1.

6
Result. 13
Solution. Denote by M1 , M2 , and M3 the centers of the circles as in the figure and by r3 the radius of the second
smallest circle.
A
M3
P C
B

M2 M1

2
Due to the symmetry of the whole figure, M1 M2 ⊥ M1 M3 and the Pythagorean theorem helps to find r3 = 3 from the
equation
M1 M22 + M1 M32 = M2 M32 or 1 + (2 − r3 )2 = (1 + r3 )2 .
Let P be a point that completes the centers M1 , M2 , and M3 to a rectangle. Let A, B, and C be the intersection points
of rays M1 P , M2 P , and M3 P with the respective circles. Since M2 M1 M3 P is a rectangle, we get P B = 43 − 1 = 13 ,
P C = 1 − 23 = 13 and P A = 2 − M2 M3 = 2 − (1 + r3 ) = 31 . Therefore, P has distance 13 to all three points A, B, and
C. Due to this fact, these points lie on the circle with center P and radius 13 . Since M1 P , M2 P , and M3 P are straight
lines, the points A, B, and C are the tangent points of the corresponding circles and the circle with center P and
radius 31 is the small circle in the picture of the posed problem.

Problem 27. In a casino, some people sat around a large table playing roulette. When Erich left that table carrying
his assets of 16 000 euros away, the average balance of all players decreased by 1 000 euros. It diminished again by 1 000
euros, when the two gamblers Bettina and Elfi got into business at that table joining in with 2 000 euros each. How
many players sat around the table while Erich was still gambling?
Result. 9
Solution. By n denote the number of people at the beginning, when Erich still was playing, and by x denote the
average balance of one gambler at that table. From the given statements we obtain the following two equations:
nx − 16 000 nx − 16 000 + 2 · 2 000
= x − 1 000 and = x − 2 · 1 000.
n−1 n+1
Working out these equations yields
x = 17 000 − 1 000n and 2 000n − 10 000 = x,
and now we easily obtain n = 9. So, while Erich was gambling, there were nine people sitting at the table.

Problem 28. In a cube 7 × 7 × 7, each two neighboring unit cubes are separated by a partition. We want to remove
some of the partitions so that each unit cube will become connected with at least one of the outer unit cubes. What is
the minimum number of partitions to be removed?
Result. 125
Solution. At the beginning, there are 73 unit cubes. By removing one partition we connect two unit cubes and the
number of isolated spaces within the cube decreases by one. At the end, we want to have at most 73 − 53 isolated
spaces (that is the number of outer unit cubes). It follows that we need to remove at least 53 = 125 partitions. It is
easy to see that 125 is enough.

Problem 29. It is known that 20∗∗∗16 is a 7-digit square of an integer. What are the three missing digits?
Result. 909
Solution. Let a2 be a perfect square ending with . . . 16. This means that a2 − 16 = (a − 4)(a + 4) is divisible
by 100, so a = 2b and (b − 2)(b√+ 2) is divisible by 25. Thus, b = 25n ± 2 and consequently a = 50n ± 4. As
14042 < (1.414 · 1000)2 < (1000 2)2 = 2000000 and 14542 > 14502 = 2102500 > 2100000, the only possibility is
a = 1446, yielding a2 = 2090916.

7
Problem 30. Triangle ABC with AB = AC = 5 m and BC = 6 m is partially filled with water. When the triangle
lies on the side BC, the surface of the water is 3 m above the side. What is the height in meters of the area filled with
water when the triangle lies on the side AB?
A
C

3 ?

B C A B

Result. 18/5
Solution. Let D be the midpoint of BC; then 4ABD is a right-angled triangle, so from the Pythagorean theorem,
AD = 4. The part of the triangle not filled with water is therefore a triangle similar to 4ABC with ratio of similitude
1/4. Since the ratio of the areas (of the non-filled part and the whole triangle) stays the same after rotating the triangle,
an analogous similarity has to occur in the new situation, too. Therefore the surface of the water is always in 3/4 of
the height, so it suffices to compute the height from AB. Knowing that the area of 4ABC is 12 · AD · BC = 12, we
have hAB = 2 · 12/AB = 24/5. We conclude that the area filled with water is of height 3/4 · 24/5 = 18/5.

Problem 31. There are six boxes numbered 1 to 6 and 17 peaches somehow distributed in them. The only move we
are allowed to do is the following: If there are exactly n peaches in the n-th box, we eat one of them and add the
remaining n − 1 peaches to the boxes 1 to n − 1, one to each box. What is the distribution of the peaches provided
that we can eat all the peaches?
Result. 1, 1, 3, 2, 4, 6
Solution. Let us trace backwards the possible moves: The final state (0, 0, 0, 0, 0, 0), i.e. when all the peaches are
eaten, can be reached only from (1, 0, 0, 0, 0, 0), which in turn could have emerged only from (0, 2, 0, 0, 0, 0) etc.—this
way we construct a unique chain of distributions of peaches

. . . (0, 2, 0, 0, 0, 0), (1, 2, 0, 0, 0, 0), (0, 1, 3, 0, 0, 0), (1, 1, 3, 0, 0, 0), . . .

ending with (1, 1, 3, 2, 4, 6), which is the sought-after distribution of seventeen peaches.

Problem 32. A simple aerial lift with fixed two-person chairs operates on a mountain. 74 people are planning to
travel upwards, whereas 26 passengers are waiting at the upper station. Exactly at noon, the lift starts working and a
pair of people gets on the lift on both its stations; the rest of the passengers is then loaded continuously. At 12:16,
the leading chair going upwards meets the last occupied chair going downwards, and at 12:22, the leading chair going
downwards meets the last occupied chair going upwards. The distance between each two chairs on the rope is the same,
the lift maintains constant speed, and all the passengers travel in pairs. How long does it take from the lower station
to the upper (in minutes)?
Result. 26
Solution. Firstly, the distance between the first and the last up-going chair is thrice the distance of the first and the
last down-going chair. Thus, we see that the time between the two moments described in the statement is twice the
time from the moment the leading chairs meet till the moment the leading up-going chair meets the last down-going
one. We infer that the leading chairs met at 12:13 (exactly in the middle of the lift), hence the time needed to go
through the whole lift is 26 minutes.

Problem 33. Let ABCD be a rhombus and M , N points on the segments AB, BC different from A, B, C such
that DM N is an equilateral triangle and AD = M D. Find ∠ABC (in degrees).
Result. 100◦
Solution. Since CD = AD = M D = N D, the triangles AM D and N CD are isosceles with bases AM , N C, respectively.
Put θ = ∠DAB; then ∠ABC = ∠ADC = 180◦ − θ. On the other hand, since

∠DAM = ∠AM D = ∠DN C = ∠N CD = θ,

we have
∠ADM = ∠N DC = 180◦ − 2θ
and
∠ADC = ∠ADM + ∠M DN + ∠N DC = 420◦ − 4θ.

8
We conclude that
420◦ − 4θ = 180◦ − θ
or θ = 80◦ , therefore ∠ABC = 100◦ .
N
C B

D A

Problem 34. In how many ways is it possible to color the cells of a 2 × 7 table with green and yellow in such a way
that neither green nor yellow L-trimino appears in the table?
Note: L-trimino is the following (possibly rotated) shape:

Result. 130
Solution. If any column of the table is monochromatic, then the neighboring column(s) must have the other color, so
the next column(s) must have the same color as the first one etc., so there are two possibilities to color the table this
way, according to which color is used in the first column.
On the other hand, the previous paragraph implies that if there is a column colored with both colors, all the other
columns have to use both colors as well. It is easy to see that no matter how we distribute the colors between the
upper and lower cells in this case, the resulting coloring will always satisfy the condition from the statement, so there
are 27 = 128 such colorings.
In total there are 2 + 128 = 130 colorings of the table.

Problem 35. Michael is a keen diamond collector, but so far he owns less than 200 diamonds. He divided all his
diamonds into several (at least two) piles in such a way that

• each two piles consist of different number of diamonds,


• none of these piles consists of exactly two diamonds,
• for each of these piles it holds that whenever it is divided into two smaller piles, at least one of these new piles
has the same size as some previously existing one.

What is the greatest number of diamonds Michael can possess?


Note: A pile consists of a non-zero number of diamonds.
Result. 196
Solution. Assume that we have a division in accordance with the problem statement. Let m be the number of
diamonds in the smallest pile. If m ≥ 2, then the smallest pile can be divided into two piles of sizes 1 and m − 1
respectively, neither of which has the same size as some other pile; hence m = 1.
Next, we show that the second smallest pile consists of 3 diamonds. Since 2 is excluded, we have to rule out the
case when it consists of n ≥ 4 diamonds. However, this is clearly not possible because of the division n = 2 + (n − 2).
Finally, we prove that if the piles 1, 3, . . . , 2k − 1 (k > 1) are the k smallest piles in the division, then the (k + 1)-th
smallest pile (if it exists) consists of 2k + 1 diamonds. Let p be the size of the (k + 1)-th smallest pile. Clearly, p is odd,
for a pile of even size could be divided into two smaller ones of even size. If p ≥ 2k + 3, the division p = 2 + (p − 2)
yields a contradiction. We conclude that the only possibility is p = 2k + 1, which is readily seen to satisfy the conditions
from the statement.
It follows by induction that the number of Michael’s diamonds is of the form 1 + 3 + · · · + (2k − 1) = k 2 . The largest
square number less than 200 is 142 = 196, which is the largest possible number of diamonds in Michael’s possession.

9
Problem 36. Recall that in the game of rock-paper-scissors we have three shapes: R – rock, P – paper and S –
scissors such that S > P , P > R, R > S and R = R, P = P , S = S, where A > B means ’A beats B’ and A = B
means ’when A is played against B, the game ends in a tie’. A tournament in Two Handed Rock-Paper-Scissors
Without Repetition between players P1 and P2 consists of 9 games. In every game each player chooses a pair (`i , ri )
where `i and ri stand for shapes played by the left and right hand, respectively, of the player Pi . During the whole
tournament each player must choose every possible pair exactly once. In a single game we distribute 4 points in the
following way: the winner on each pair of playing hands (left/right) receives 2 points and the loser receives 0 points or
both player receive 1 point if there is a tie in a pair of hands. Suppose that players are choosing their moves at random.
What is the probability that each of the 9 games in the tournament ends in a tie (i.e. with score 2:2)?
Result. 3!3 /9! = 1/1680
Solution. Let us define three sets of three pairs each:
DR = {(R, R), (P, S), (S, P )}, DP = {(P, P ), (S, R), (R, S)}, DS = {(S, S), (R, P ), (P, R)}.
Note that a single game in the tournament ends in a tie if and only if two pairs from the same set out of DR , DP , DS
are played against each other.
Possible outcomes of the whole tournament are all pairs of permutations of the set DR ∪ DP ∪ DS . All the games end
in a tie if and only if elements of each set DR , DP , DS occupy the same three positions in P1 ’s and P2 ’s permutation.
Consider any permutation and let it denote the order of moves of the player P1 in subsequent games. Number of
arrangements of P2 ’s moves yielding a draw in each game equals 3!3 , no matter what the P1 ’s permutation was.
Therefore the desired probability is
3!3 1
= .
9! 1680

Problem 37. The net of a solid consists of eight regular triangles and six squares, as shown in the picture:

Assuming that the length of each edge is 1 km, what is the volume of the solid (in km3 )?

Result. 53 2
Solution. The described solid can be obtained from a cube in the following way: Each corner of the cube is cut
off,
√ the cut going through √ the centers of the edges adjacent to the removed vertex. The edge length of the cube is
2, so its volume is 2 2. The removed corners √ are eight (oblique) pyramids,
√ the base of each being an isosceles
right-angled √ triangle√with the√leg of length 2/2, and the height being 2/2,
√ too. √
Therefore √
the volume of one corner
is 13 · 12 · ( 2/2)2 · ( 2/2) = 2/24 and the volume of the given solid is 2 2 − 8 · 2/24 = 5 2/3.

Problem 38. Find the only three-digit prime factor of 999 999 995 904.
Result. 601
Solution. Observe that
999 999 995 904 = 1012 − 212 = 212 (512 − 1)
and
512 − 1 = (5 − 1)(5 + 1)(52 + 1)(52 − 5 + 1)(52 + 5 + 1)(54 − 52 + 1),
but only the last factor is greater than 100. Since we know that a three-digit prime factor exists and 54 − 52 + 1 = 601
is clearly divisible by neither of 2, 3, 5, it is a prime, and hence the sought number.

Problem 39. Thirteen bees: one little bee and twelve large bees are living on a 37-cell honeycomb. Each large bee
occupies 3 pairwise adjacent cells and the little bee occupies exactly 1 cell (see the picture). In how many ways can the
honeycomb be divided into 13 non-overlapping sectors so that all thirteen bees can be accommodated in accordance
with the given restrictions?

10
Result. 20
Solution. Let us consider 13 cells shaded in the picture below:

Each three-cell sector contains exactly one shaded cell, so the cell of the little bee must be one of the shaded.
If it is the central cell, then there are exactly two ways to divide the rest of the honeycomb into 12 large bee sectors
(the one shown in the picture and the one rotated by 60 degrees). For each of the 6 ‘middle’ shaded cells there is
exactly one way to accommodate large bees in the rest of the honeycomb. Finally, for each of the boundary shaded cells
there are exactly two ways to divide the remaining cells into three-cells sectors (the one shown and the symmetric one).

This gives a total of 2 + 6 · 1 + 6 · 2 = 20 ways to dissect the honeycomb into the sectors as requested.

Problem 40. Equilateral triangle ABC is inscribed in circle ω. Point X is on the (shorter) arc BC of ω and T is
the intersection of AB and CX. If AX = 5 and T X = 3, find BX.
Result. 15/8
Solution. Since ∠AXB = ∠ACB = 60◦ and ∠AXC = ∠ABC = 60◦ , ∠BXT = 180◦ − ∠AXB − ∠AXC = 60◦ . Let
U be a point on AX such that T U k BX.

T B A

X
U

Then T U X is an equilateral triangle and 4T U A ∼ 4BXA. Therefore we have


TU T X · AX 15
BX = · AX = = .
AU T X + AX 8

Problem 41. Let ABC be an equilateral triangle. An interior point P of ABC is said to be shining if we can find
exactly 27 rays emanating from P intersecting the sides of the triangle ABC such that the triangle is divided by these
rays into 27 smaller triangles of equal area. Determine the number of shining points in ABC.
Result. 26

2 = 325
Solution. We see that P A, P B, P C are among the 27 rays from P : If not, we would obtain a quadrilateral, leading
to a contradiction. Let us divide the perimeter of 4ABC into 27 segments such that each side is divided into segments
of equal length; there are altogether 26

2 = 325 such divisions, since if we fix A to be the first dividing point, we can
freely choose B and C from the remaining 26 points. Finally, observe that each such division corresponds to exactly

11
one shining point and vice versa: Clearly, (the rays from) each shining point gives rise to a division. On the other hand,
given the numbers a, b, c of the segments the respective sides are divided into, we let P be the unique point inside
4ABC such that the distances of P to the sides BC, CA, AB are in the ratio a : b : c. A straightforward computation
shows that P is indeed the shining point, the rays from which divide 4ABC in accordance with the given division.

Problem 42. How many positive divisors of 20162 less than 2016 are not divisors of 2016?
Result. 47
Solution. From the prime factorization 2016 = 25 · 32 · 7 we obtain 20162 = 210 · 34 · 72 . Therefore, 20162 has
11 · 5 · 3 = 165 positive divisors of which 12 · (165 − 1) = 82 are less than 2016—excluding 2016, the divisors may be
divided into pairs (x, y) such that x · y = 20162 and x < 2016 < y. Note that 2016 has 6 · 3 · 2 − 1 = 35 divisors less
than 2016 which naturally are divisors of 20162 , too. Hence, the desired number of divisors is 82 − 35 = 47.

Problem 43. Let √


4n + 4n2 − 1
Zn = √ √ .
2n − 1 + 2n + 1
Compute Z1 + Z2 + · · · + Z2016 .

Result. 12 (4033 4033 − 1)
Solution. Observe that for n ∈ N,
√ √ √ √ √ √ √
4n + 4n2 − 1 ( 2n + 1 − 2n − 1)(( 2n + 1)2 + ( 2n + 1)( 2n − 1) + ( 2n − 1)2 )
√ √ = √ √ √ √
2n − 1 + 2n + 1 ( 2n + 1 − 2n − 1)( 2n + 1 + 2n − 1)
1 √ √
= (( 2n + 1)3 − ( 2n − 1)3 ).
2
Therefore,
1 √ 3 √ √ √ √ √
( 3) − ( 1)3 + ( 5)3 − ( 3)3 + · · · + ( 4033)3 − ( 4031)3

Z1 + · · · + Z2016 =
2
1 √
= (4033 4033 − 1).
2

Problem 44. We construct a sequence of integers a0 , a1 , a2 . . . in the following way: If ai is divisible by three, let
ai+1 = ai /3; otherwise let ai+1 = ai + 1. For how many different positive integers a0 does the sequence reach the value
1 for the first time in exactly eleven steps (i.e. a11 = 1, but a0 , a1 , . . . , a10 6= 1)?
Result. 423
Solution. The number 1 can be reached only from 3, which in turn can be obtained from 2 or 9. Further, 9 can arise
from 8 or 27; 2 can be reached from 1 or 6, but only 6 is admissible in the light of the problem statement. Let us
construct further predecessors: Let Pn be the set of positive integers such that the sequence from the statement reaches
1 in exactly n steps if and only if a0 ∈ Pn . Clearly, to establish Pn+1 for n ≥ 3, we take 3x for each x ∈ Pn and also
x − 1 for each x ∈ Pn such that x − 1 is not a multiple of three.
Let pn be the number of elements of Pn , and denote further by fn , gn , hn the number of elements of Pn of the form
3k, 3k + 1, 3k + 2, respectively. Observe that for n ≥ 3, all the elements of Pn are greater than 3, and so
• fn+1 = pn , since for each x ∈ Pn there is 3x ∈ Pn+1 ,
• gn+1 = hn , since for each x ∈ Pn of the form 3k + 2 there is x − 1 = 3k + 1 ∈ Pn+1 , and
• hn+1 = fn for similar reasons.

Therefore we have
pn = fn + gn + hn = pn−1 + pn−2 + pn−3
for n ≥ 4. The initial calculations show that p1 = 1, p2 = 2, and p3 = 3, and the subsequent terms can be calculated in
a straightforward manner using the recurrence above. The sought result is p11 = 423.

12
Problem 45. Let ABCD, AEF G, and EDHI be rectangles with centers K, L, J, respectively. Assume further
that A, D, E are inner points of line segments HI, F G, BC, respectively, and ∠AED = 53◦ . Determine the size of
∠JKL (in degrees).
Result. 74◦
Solution. Since KJ is a median in triangle BID, we have KJ k BI, and similarly, KL k CF . Thus ∠JKL =
∠IBA + ∠DCF . Since ∠AIE = ∠ABE = 90◦ , the quadrilateral BIAE is cyclic. Consequently,
∠IBA = ∠IEA = 90◦ − ∠AED = 37◦ .
In the same way we deduce ∠DCF = 37◦ , therefore ∠JKL = 74◦ .
C E B

F
I
K

L
J

D A

G
H

Problem 46. James has picked several (not necessarily distinct) integers from the set {−1, 0, 1, 2} in such a way
that their sum equals 19 and the sum of their squares is 99. What is the greatest possible value of the sum of the
cubes of James’ numbers?
Result. 133
Solution. Assume that there are exactly a, b, c numbers equal to −1, 1, 2, respectively, among the James’ numbers
(those equal to 0 clearly play no role). The conditions from the statement may then be rewritten as
−a + b + 2c = 19,
a + b + 4c = 99.
Our goal is to maximize −a + b + 8c = 19 + 6c. However, by adding the equalities, we find out that 6c = 118 − 2b, so
c ≤ 19. The value c = 19 can be obtained with the choice a = 21, b = 2, so the sought maximum is 19 + 6 · 19 = 133.

Problem 47. Find the largest 9-digit number with the following properties:
• all of its digits are different;
• for each k = 1, 2, . . . , 9, when the k-th digit is crossed out the resulting 8-digit number is divisible by k.

Result. 876 513 240


Solution. Denote by Ak the k-th digit of the sought number, so the number is A1 A2 A3 A4 A5 A6 A7 A8 A9 . There are 10
possible digits, so exactly one, say d, is not used in the decimal representation of the desired number. Let Nk be the
number with k-th digit crossed out.
We know that N2 is even, so 2 | A9 . Furthermore, the number N5 is divisible by 5, hence so is A9 . This means that
A9 = 0.
The number N9 is divisible by 9, so the digit sum of this number, namely 1 + 2 + 3 + 4 + 5 + 6 + 7 + 8 + 9 − d = 45 − d,
is divisible by 9, which leads to d = 9.
Numbers N8 and N4 are both divisible by 4 which means that both digits A7 , A8 are even. In addition, the first of
these numbers is divisible by 8, so the 2-digit number A6 A7 is divisible by 4. Also numbers N3 and N6 are divisible by
3 and so is the digit sum of the desired number. From this we get {A3 , A6 } = {3, 6}.
As we are looking for the largest possible number, suppose that A1 = 8, A3 = 6, A6 = 3. We then have
{A7 , A8 } = {2, 4} but since 4 | A6 A7 , A7 = 2 and A8 = 4.
It suffices to check that putting the remaining digits in a decreasing order in the gaps leads to the number 876513240
which satisfies the remaining condition, i.e. the number N7 = 87651340 is divisible by 7.

13
Problem 48. Point P lies inside a rectangle ABCD with AB = 12. Each of triangles ABP , BCP , DAP has its
perimeter equal to its area. What is the perimeter of triangle CDP ?
D C

A 12 B

Result. 25
Solution. Note that a triangle has equal area and perimeter if and only if the incircle has radius 2. Thus triangles
BCP and ADP are congruent; indeed, if P is closer to AD than BC, then the inradius of ADP is smaller than the
inradius of BCP . This means that P lies on one of the axes of symmetry of ABCD.
D C
y
P
Q

A M 6 B

Let Q be the perpendicular projection of P onto BC, M the midpoint of segment AB and put x = BQ,√y = CQ. The
area of triangle ABP is thus 6x, and using the Pythagorean theorem in triangle M BP , we find BP = x2 + 62 . The
equality of area and perimeter of 4ABP thus translates into the equation
p
6x = 12 + 2 x2 + 62

with the only solution x = 9/2. p


The value of y can be found similarly: We have BP = 15/2 and CP = y 2 + 62 , so the condition on triangle BCP
implies  
1 9 9 15 p 2
·6· y+ =y+ + + y + 62
2 2 2 2
with the only positive solution y = 5/2.
We conclude that CP = 13/2 and the perimeter of triangle CDP equals 25.

Problem 49. The pair of integers (0, 0) is written on a blackboard. In each step, we replace it in this way: If there is
a pair (a, b), we replace it with (a + b + c, b + c), where either c = 247 or c = −118 (we may choose the number c in each
step). Find the smallest (non-zero) number of steps after which the pair (0, b) for some b appears on the blackboard.
Result. 145
Solution. Denote ci the number c used in the i-th step. After n steps, the number a (i.e. the first coordinate of the
pair) will be a = nc1 + (n − 1)c2 + · · · + cn . Fix n and let s = nε1 + (n − 1)ε2 + · · · + εn , where εi = 1 if ci = 247,
and εi = 0 otherwise. Define t in a similar fashion with εi = 1 if and only if ci = −118. Clearly a = 247s − 118t, so
the condition a = 0 implies 247s = 118t, but as the numbers 247 and 118 are coprime, there is an integer k such that
s = 118k and t = 247k. It follows that
n(n + 1)
365k = s + t = 1 + 2 + · · · + n = ,
2
and since 365 = 5 · 73, we see that n is at least 2 · 73 − 1 = 145.
It remains to show that there are numbers ci such that 247s = 118t with n = 145. Indeed, let m be the smallest
positive integer such that
247
1 + 2 + ··· + m ≥ · (1 + 2 + · · · + n);
365

14
now put ci = −118 for i ∈ {1, . . . , m} \ {r} and ci = 247 otherwise, where
247
r = 1 + 2 + ··· + m − · (1 + 2 + · · · + n).
365
(In fact, m = 120 and r = 97.) This way we get precisely
118 · 247
247s = 118t = · (1 + 2 + · · · + n)
365
as desired.

Problem 50. A zigzag consists of two parallel rays of opposite directions with the initial points joined with a segment.
What is the maximum number of regions the plane can be divided into using ten zigzags?
Result. 416
Solution. Every two zigzags can intersect in at most nine points, and for any number of zigzags, we may easily achieve
the configuration when every two intersect in exactly nine points (and each point is the intersection of at most two
lines). Consider placing the zigzags one by one: The n-th added zigzag is divided by the 9(n − 1) intersection points
with the n − 1 already placed zigzags into 9(n − 1) + 1 segments, each dividing an existing region into two. It follows
the maximum number of regions definable using n zigzags, Zn , satisfies Z1 = 2 and Zn = Zn−1 + 9n − 8 for n ≥ 2.
The general result Zn = 92 n2 − 72 n + 1 then follows easily by induction and in particular, Z10 = 416.

Problem 51. Each face of a tetrahedron is a triangle with sides 1, 2, and c and the circumradius of the tetrahedron
is 5/6. Find c.

Result. 23/3
Solution. We will prove a more general result: If each side of a tetrahedron is a triangle with sides a, b, c and the
circumradius of the tetrahedron is %, then a2 + b2 + c2 = 8%2 . The result in our particular situation then follows directly
by plugging in.
Inscribe the tetrahedron in a cuboid with edges of lengths p, q, r so that the edges of the tetrahedron are the face
diagonals of the cuboid. By the Pythagorean theorem,

p2 + q 2 = a2 , p2 + r2 = b2 , and q 2 + r2 = c2 .

Furthermore, the circumsphere of the tetrahedron coincides with the one of the cuboid, the diameter of which is the
space diagonal. Therefore
1
(2%)2 = p2 + q 2 + r2 = (a2 + b2 + c2 ),
2
which after rearranging gives the claimed equality.

Problem 52. For a big welcome party butler James has lined up 2016 cocktail glasses in a row, each containing
delicious cherry cocktail. To finish things up, his task is to cover one of the glasses with a silver lid, to put a statue on
top of the lid and to distribute an odd number of cherries into the uncovered glasses, at most one cherry per glass.
How many possible arrangements of cherries and the lid are there if there have to be more cherries on the right-hand
side of the lid than on its left-hand side?
Result. 2016 · 22013
Solution. First, consider all possible arrangements of the lid and at most one cherry in each uncovered glass without
posing any further condition. There are 2016 possible locations for the lid and 22015 possibilities to put at most one
cherry in each of the remaining 2015 glasses, which gives a total of 2016 · 22015 arrangements. The binomial formula
expansion
2016
X 2016 1008
X 2016 1008 X  2016 
2016 i
0 = (−1 + 1) = (−1) = −
i=0
i i=0
2i i=1
2i − 1
shows that the number of all arrangements having an even number of cherries is equal to the number of all arrangements
having an odd number of cherries. Hence, the set M of all arrangements of the lid and an odd number of cherries
contains 12 · 2016 · 22015 = 2016 · 22014 elements. Now observe that every element (n1 , n2 ) of M representing the
arrangement with n1 cherries on the left-hand side of the lid and n2 cherries on its right-hand side has got exactly
one corresponding element (n2 , n1 ) in M . These arrangements are different from each other, because the sum n1 + n2
being an odd number implies that one of the numbers n1 or n2 is even, the other one is odd, and one number is bigger
than the other. Hence, exactly one of the two corresponding arrangements complies with the stated condition that
there should be more cherries on the right-hand side of the lid than on its left-hand side. Therefore, the answer is
1 2014
2 · 2016 · 2 = 2016 · 22013 .

15
Problem 53. We are given a wooden cube with its surface painted green. There are 33 different planes, each located
between some two opposite faces of the cube and parallel to them, which dissect the cube into small cuboidal blocks.
Given that the number of blocks with at least one green face equals the number of blocks with no green faces, determine
the total number of blocks into which the cube is dissected.
Result. 1260 or 1344
Solution. It is easy to see that there have to be at least four planes in each of three possible directions (if in one of
the directions there are less than five layers of blocks, then the number of at-least-one-green-side blocks is greater than
the number of inside blocks). Denote the numbers of planes in different directions by a + 3, b + 3, c + 3, where a, b, c
are positive integers. It follows that (a + 3) + (b + 3) + (c + 3) = 33, so a + b + c = 24.
The problem condition can be rewritten as

(a + 4)(b + 4)(c + 4) = 2(a + 2)(b + 2)(c + 2)

which yields abc = 240 = 24 · 3 · 5 after simplification. Since a + b + c is even, either (1) exactly one or (2) all three of
numbers a, b, c are even.
In the case (1), one of the numbers a, b, c (w.l.o.g. a) must be divisible by 16 and since a + b + c = 24 < 2 · 16, we
have a = 16. It follows that b + c = 8 and bc = 15, so {b, c} = {3, 5}. We can now calculate the total number of blocks:
(a + 4)(b + 4)(c + 4) = 20 · 7 · 9 = 1260.
In the case (2) we have w.l.o.g. a = 4x, b = 2y, c = 2z where xyz = 15 and 2x + y + z = 12. The only possibility is
x = 3, {y, z} = {1, 5}, which gives (a + 4)(b + 4)(c + 4) = 16 · 6 · 14 = 1344.

Problem 54. Given a positive integer n, let p(n) be the product of non-zero digits of n. Find the largest prime
divisor of the number p(1) + · · · + p(999).
Result. 103
Solution. Let S = p(1) + · · · + p(999). By expanding A = (0 + 1 + 2 + · · · + 9)(0 + 1 + 2 + · · · + 9)(0 + 1 + 2 + · · · + 9)
one can see that A would be the result if we multiplied by zero digits as well. Hence we have S = (1 + 1 + 2 + · · · +
9)(1 + 1 + 2 + · · · + 9)(1 + 1 + 2 + · · · + 9) − 1 because of the extra 1 which we do not want to count. So

S = 463 − 1 = (46 − 1)(462 + 46 + 1) = 33 · 5 · 7 · 103

and the conclusion follows.

Problem 55. Let (an )∞ n=1 be a strictly increasing sequence of positive integers such that 9 | a3k−2 , 14 | a3k−1 , and
19 | a3k for all positive integers k. Find the smallest possible value of a2016 .
Result. 14478
Solution. We may assume that for all n the value of an is the smallest integer greater then an−1 , which satisfies
the divisibility condition. Observe that given a3k there are only two options for a3k+3 : Either a3k+3 = a3k + 19, or
a3k+3 = a3k + 38. The latter occurs if and only if there are integers c, d, 5 ≤ d ≤ c ≤ 9, such that 9 | a3k + c and
14 | a3k + d, since this implies a3k+1 = a3k + c and a3k+2 = a3k + 14 + d ≥ a3k + 19.
There are exactly 62 = 15 pairs (c, d) satisfying 5 ≤ d ≤ c ≤ 9. Since the numbers 9, 14, and 19 are pairwise


coprime, the Chinese remainder theorem guarantees that for each such pair (c, d) there is exactly one non-negative
integer a3k less than 9 · 14 · 19 such that 19 | a3k , 9 | a3k + c and 14 | a3k + d. Therefore there are exactly 15 terms
a3k less than 9 · 14 · 19 for which a3k+3 = a3k + 38. It is easy to see that the difference of no two of these terms is
19 and that 9 · 14 · 19 − 19 is not such a number, which means that a3` = 9 · 14 · 19 for some `. From the fact that
a3k+3 = a3k + 38 happens exactly 15 times we infer that ` = 9 · 14 − 15 = 111.
For the terms an succeeding a333 , the remainders modulo 9, 14, and 19 is the same as for an−333 , so we obtain the
relation an+333 = an + 9 · 14 · 19. We may easily compute that a18 = 114, and so

a2016 = a6·333+18 = 6 · 9 · 14 · 19 + 114 = 14478.

Problem 56. Let P be a point inside triangle ABC. Points D, E, F lie on the segments BC, CA, AB, respectively,
such that the lines AD, BE, CF intersect in P . Given that P A = 6, P B = 9, P D = 6, P E = 3, and CF = 20, find
the area of triangle ABC.
Result. 108
Solution. Denote by [XY Z] the area of triangle XY Z. From AP = DP we obtain [ABP ] = [BDP ] and [AP C] =
[DCP ]. Further, 3EP = BP implies that 3[AP E] = [ABP ] and

3[CEP ] = [BCP ] = [BDP ] + [DCP ] = 3[AP E] + [AP E] + [CEP ],

and so [CEP ] = 2[AP E]. We conclude that [ABP ] = [BDP ] = [AP C] = [DCP ]; in particular, BD = CD.

16
Put k = F P : CP . Then from AP = DP and ∠AP F = ∠CP D we have [AF P ] = k[DCP ]; similarly, [F BP ] =
3k[CEP ]. Combining with the known ratios above we get k = 1/3, therefore F P = 5, CP = 15. If we complete triangle
CP B to a parallelogram CP BQ, we may note that BP 2 + P Q2 = BQ2 , and so ∠DP B = 90◦ .
B
Q

P F

C A
E

We conclude that
1
[ABC] = 4[BDP ] = 4 · · 6 · 9 = 108.
2

Problem 57. Find the last two digits before the decimal point of the number (7 + 44)2016 .
Result. 05
√ √
Solution. Firstly observe that
√ the number 7√− 44 is strictly between 0 and 1, so the same holds for (7 − 44)2016 .
Moreover, the number
√ (7 + 44)2016 + (7 − 44)2016 is readily seen to be an integer using the binomial formula (the
odd powers of 44 cancel out), so, in fact,
√ √ √
b(7 + 44)2016 c = (7 + 44)2016 + (7 − 44)2016 − 1.

Exploiting the fact that 122 ≡ 44 (mod 100), we obtain


√ √
(7 + 44)2016 + (7 − 44)2016 ≡ (7 + 12)2016 + (7 − 12)2016 (mod 100),

so it suffices to find the last two digits of 192016 and 52016 . The latter is just 25, as 53 ≡ 52 (mod 100). To handle the
former one, employ the binomial formula again to obtain
   
2016 2016 1 2015 2016
(20 − 1) ≡ · 20 · (−1) + (−1)2016 ≡ −19 (mod 100)
2015 2016

(all the terms up to the last two ones are divisible by 202 ). We conclude that the sought digits are −19 + 25 − 1 = 05.
√ √
Alternative solution.
√ As above,
√ we shall seek the last two digits of the number (7 + 44)2016 + (7 − 44)2016 . As
2
the numbers 7 + 44, √ 7 − 44 are roots of the√ quadratic equation x − 14x + 5 = 0, the sequences (αn )n≥0 , (βn )n≥0 ,
defined via αn = (7 + 44)n and βn = (7 − 44)n , are subject to the recurrence √ relation
√ αn+2 − 14αn+1 + 5αn = 0
and the same for βn . Moreover, the same holds for their sum, γn = (7 + 44)n + (7 − 44)n . Our goal is to compute
γ2016 mod 100.
Put γ̃n = γn mod 100. The sequence (γ̃n )n≥0 is completely determined by the recurrence relation γ̃n+2 =
(14γ̃n+1 − 5γ̃n ) mod 100 and the initial values γ̃0 = 2, γ̃1 = 14. Further, since γ̃n attains only finitely many values and
every term depends only on the previous two, the sequence has to be periodic. By computing several of its values,

2, 14, 86, 34, 46, 74, 6, 14, 66, 54, 26, 94, 86, 34, . . . ,

we see that from γ̃2 on, the sequence is periodic with period 10; thus γ̃2016 = γ̃6 = 6. Since the sought number is one
less, the last two digits are 05.

17
Problem 1. The windows on an old tram look like shown in the picture. All curves forming the round corners are
arcs of a quarter circle with radius 10 cm. A portion of the sliding window is opened 10 cm as you can see in the second
picture. The height of the open section is 13 cm. What is the area of the opening in cm2 ?

Result. 130
Solution. It is just the area of the sliding window overlapping the fixed part, which is a rectangle 10 cm × 13 cm.

Problem 2. A rectangle is subdivided into nine smaller rectangles as shown in the picture. The number written
inside a small rectangle denotes its perimeter. Find the perimeter of the large rectangle.

14 10 17

12

Result. 42
Solution. Looking at the picture we notice that the perimeter of the large rectangle equals the sum of the perimeters
of the four outer small rectangles with given perimeter minus the perimeter of the middle one. Therefore, the answer is
14 + 9 + 17 + 12 − 10 = 42.

Problem 3. Peter deleted one digit from a four-digit prime number and obtained 630. What was the prime number?
Result. 6301
Solution. Since the last digit of a (four-digit) prime number cannot be even, the prime number was of the form 630∗.
Moreover, the last digit cannot be 5, because the number would be divisible by 5. So the options 1, 3, 7 and 9 are left.
But since 630 is divisible by 3, the digits 3 and 9 are impossible. Similarly, 630 is divisible by 7 and so would be 6307.
Therefore, the number was 6301.

Problem 4. Star architect Pegi wants to build a very modern pentagonal mansion on a rectangular lot of land having
side lengths 35 m and 25 m. The floor area of the mansion fits into the lot as can be seen in the picture:

(The dots on the boundary mark 5 m distance.) What fraction of the area of the lot does the floor area of the mansion
cover?
41
Result. 70
Solution. Since we are only interested in the fraction of areas, we can use 5 m as a unit. By summing up the areas of
the three right-angled triangles, we obtain the result
 
1 5·3 6·1 4·2 41
1− + + = .
5·7 2 2 2 70

1
Problem 5. A square grid of 16 dots, as can be seen in the picture, contains the corners of nine 1 × 1-squares, four
2 × 2-squares, and one 3 × 3-square, for a total of 14 squares whose sides are parallel to the sides of the grid. What
is the smallest possible number of dots you can remove so that, after removing those dots, each of the 14 squares is
missing at least one corner?

Result. 4
Solution. It is necessary to remove four dots, because the four 1 × 1-squares in the corners of the given grid do not
share any dots in common. Removing two opposite corners of the grid and two centre dots along the other diagonal
provides an example to show that this number is sufficient.

Problem 6. Find the digit at the units place of the sum of squares

12 + 22 + 32 + · · · + 20172 .

Result. 5
Solution. The digits in the unit position of square numbers show up periodically having period 10. We have

12 + 22 + 32 + · · · + 102 = 1 + 4 + 9 + 16 + 25 + 36 + 49 + 64 + 81 + 100 = 385,

so the last digit is 5. Consequently, the last digit of 12 + 22 + · · · + 20102 is 5 due to 201 · 5 = 1005. Furthermore, the
last digit of the sum 20112 + 20122 + · · · + 20172 is 0. Altogether, the desired digit is 5.

Problem 7. Express the quotient


0.2
0.24
a
as a fraction in lowest terms with positive integers a and b.
b
Note: The overline denotes periodic decimal expansion, for example 0.123 = 0.123123 . . ..
11
Result. 12
Solution. The given quotient can be written in the following way:

0.2 0.22 22 · 0.01 11


= = = .
0.24 0.24 24 · 0.01 12

2
Problem 8. Passau has a railway station in the form of a triangle. Anna, Boris, and Cathy observe the railway
traffic in Linz, Regensburg, and Waldkirchen, respectively, on the rails coming from Passau. Anna counts 190, Boris
208 and Cathy 72 incoming and outgoing trains in total. How many trains went from Linz to Regensburg or vice versa
if no train starts, ends or reverses its direction in Passau?

Waldkirchen
Cathy

Anna
Passau Linz

Boris
Regensburg

Result. 163
Solution. We denote the number of trains between Linz and Waldkirchen by r, the one between Linz and Regensburg
by w, and the one between Waldkirchen and Regensburg by l. Anna counts all the trains between Linz and the other
two cities. Therefore, we obtain the equation r + w = 190. We get the equations l + w = 208 and l + r = 72 similarly.
Adding the first two equations and subtracting the third one gives 2w = 190 + 208 − 72 which leads to w = 12 · 326 = 163.

Problem 9. Find all positive integers x < 10 000 such that x is a fourth power of some even integer and one can
permute the digits of x in order to obtain a fourth power of some odd integer. The result of permuting may not begin
with zero.
Result. 256
Solution. Assume that x is equal to a4 for some even integer a and that it can be transformed to b4 for some odd
integer b.
Since 10 000 = 104 , both a and b are less than 10. By squaring even one-digit integers and then squaring their last
digits, we find out that a4 always ends with 6. Considering the possible values of b, we see that only 54 = 625 and
94 = 6561 contain the digit 6. However, any permutation of digits of 94 would be divisible by 3 (digit sum does not
change), so the only possible result would be 64 = 1296, which cannot be permuted to 94 . For b4 = 625 we easily find
x = a4 = 256 as the only result.
1
Problem 10. In parallelogram ABCD a line through point C meets side AB in point E such that EB = 5 AE. The
line segment CE intersects the diagonal BD in point F . Find the ratio BF : BD.
D C

A E B

Result. 1 : 7
Solution. The triangles EBF and CDF are similar with a scale factor EB : DC = 1 : 6. As a consequence
BF : F D = 1 : 6 as well, hence BF : BD = 1 : 7.

Problem 11. A big house consists of 100 numbered flats. In every flat, there lives one person or there live two or
three persons. The total number of people living in the flats No. 1 to No. 52 is 56 and the total number of people living
in the flats No. 51 to No. 100 is 150. How many people live in this house?
Result. 200
Solution. Since the maximum number of people in a flat is three, exactly three people live in each flat from No. 51 to
No. 100. Therefore, there are 56 − 2 · 3 = 50 people living in the first up to the 50th flat in total. This leads to a total
number of 50 + 150 = 200 inhabitants for the entire house.

3
Problem 12. In the first stage, Nicholas wrote the number 3 with a red pencil and 2 with a green pencil on a sheet
of paper. In the following stages, he used the red pencil for writing the sum of the two numbers from the previous
stage and the green pencil for their (positive) difference. What number did he write in red in the 2017th stage?
Result. 3 · 21008
Solution. It is easy to see that in each stage, the red number is greater than the green one. Assuming that in
the n-th stage the numbers Rn , Gn are written with red and green, respectively, then in the stage n + 1, we have
Rn+1 = Rn + Gn and Gn+1 = Rn − Gn and in the stage n + 2,

Rn+2 = Rn+1 + Gn+1 = 2Rn ,


Gn+2 = Rn+1 − Gn+1 = 2Gn .

Therefore, both the numbers are doubled each two stages. Between the 1st and the 2017th stage this happens 1008
times, therefore the result is 3 · 21008 .

Problem 13. Little Red Riding Hood finds herself at the entrance of the “Rectangular Forest”. Starting at point A,
she has to reach point B as fast as possible. One possibility is to walk along the edges of the woods which will be 140 m
in total. Of course, she knows that according to the triangle inequality a direct path would be shorter. Unfortunately,
there is only a path which has the form of a zigzag with two right-angled turns as can be seen in the picture. If she
knew that this way was shorter than 140 m, she would dare to take the shortcut. Find the length (in meters) of the
zigzag through the forest!
B

60 m

A
80 m

Result. 124

Solution. By the Pythagorean theorem, the length of the diagonal of the rectangle is 602 + 802 = 100. For example
by the cathetus theorem, one can determine the shorter segment cut by the altitude to the diagonal as 602 /100 = 36.
Now we obtain
√ 100 − 36 = 64 for the longer segment of the diagonal. Using the altitude theorem, the length of the
altitude is 36 · 64 = 48. Altogether, the zigzag path is of length 48 + (64 − 36) + 48 = 124.

Problem 14. Eight-digit palindromes are numbers of the form abcddcba where a, b, c, and d are not necessarily
distinct digits. How many eight-digit palindromes have the property that we can delete some digits in such a way that
the resulting number would be 2017?
Result. 8
Solution. Since all digits of 2017 are different, all digits a, b, c, d are distinct and we should erase each letter exactly
once. Observe that after erasing, either the first or the last digit of 2017 will be equal to a. In the former case we have
a = 2 and arrive at an analogous problem for a six-digit palindrome with digits 0, 1, 7. In the latter case we have a = 7
and arrive at an analogous problem for a six-digit palindrome with digits 2, 0, 1. Again, in both cases we have two
choices for the first digit, so we obtain four problems for a four-digit palindrome. Again, each of these problems splits
into two problems for a two-digit palindrome, which has a unique solution. Hence, overall we have 23 = 8 options.

Problem 15. For a positive integer n the sum of its digits is denoted by S(n), and the product of its digits by P(n).
How many positive integers n are there having the property n = S(n) + P(n)?
Result. 9
Solution. For a positive integer n consisting of only one digit, we always have S(n) + P(n) = 2n > n. Now we consider
positive integers n having more than one digit. Let m ≥ 1 and n = am 10m + · · · + a0 be an integer with 0 ≤ ak ≤ 9 for
0 ≤ k ≤ m and am 6= 0. Then we have

n − S(n) − P(n) = am 10m + · · · + a0 − (am + · · · + a0 ) − am · · · a0


= (10m − 1 − am−1 · · · a0 )am + (10m−1 − 1)am−1 + · · · + 9a1
≥ (10m − 1 − 9m )am
≥0

4
and equality can hold only for m = 1. Therefore, for n to satisfy the given condition we must have

n = 10a1 + a0 = a1 + a0 + a1 a0

which is equivalent to a1 (9 − a0 ) = 0, that is to a0 = 9. We conclude that exactly the nine numbers 19, 29, 39, 49, 59,
69, 79, 89, and 99 have the desired property.

Problem 16. A factory owner hires 100 employees. Each work team leader earns 5 000 e per month, each labourer
1 000 e per month and each part-time worker earns 50 e per month. In total, the factory owner pays 100 000 e monthly
to his employees and there is at least one employee of each type. How many work team leaders are there in his factory?
Result. 19
Solution. Denote by x, y, z the number of work team leaders, labourers, and part-time workers, respectively. The
conditions from the statement can be rewritten into a system of equations

x + y + z = 100 (1)
5 000x + 1 000y + 50z = 100 000 (2)

in positive integers. Expressing z from (2) yields z = 2000 − 20y − 100x with the right-hand side divisible by 20, so z
can be written as z = 20k for some positive integer k. Therefore, (2) can be simplified to 5x + y + k = 100. Subtracting
(1), wherein z = 20k is used as well, leads to 4x = 19k. As 4 and 19 are coprime, it follows that x is a multiple of 19.
Since x, y, and z are positive, (2) implies x < 20. Consequently, x = 19 (with y = 1 and z = 80) is the only solution.

Problem 17. The mosaic below is composed of regular polygons. The hexagon and the dark grey triangle are
inscribed in the same circle. If the area of a striped triangle is 17, determine the area of the dark grey triangle.

Result. 51
Solution. Firstly, consider the dark grey triangle being rotated by 30◦ about the center of the circle such that its
vertices coincide with vertices of the hexagon. Then it is clear that it covers half of the hexagon.

Secondly, observe that a striped triangle is equilateral having the same side length as the hexagon. Therefore, the area
of a striped triangle is 16 of the area of the hexagon. Altogether, the area of the dark grey triangle is 3 · 17 = 51.

Problem 18. In the course of renovating the train station of Passau, special paving for visually handicapped people
is being built in. The shape of the paving is shown in the picture below. Unfortunately, only tiles of the size 1 × 2 are
available. In how many ways can the paving be filled with tiles? The tiles are indistinguishable and two pavings are

5
considered to be different if at some spot tiles are in a different position.

Result. 15
Solution. By starting to place the tiles along the narrow strips, it is obvious that the way of tiling is determined up to
the 3 × 5-area:

There are three options for the next tile: Either it can be put in the same direction as the last one (case (1) below),
which produces two separated 3 × 2-areas, or it can be put perpendicularly (cases (2) and (3)). In these cases we may
carry on tiling in a unique way until all but one 3 × 2-area remains.

(1) (2) (3)

Each 3 × 2-area can be tiled in three ways as in the following picture:

Therefore, there are 3 · 3 = 9 ways to finish the tiling in the case (1) and 3 ways in both cases (2) and (3). Altogether,
we get 9 + 3 + 3 = 15 possibilities.

Problem 19. Michael picked a positive integer n. Then he chose a (positive) divisor of n, multiplied it by 4, and
subtracted this result from n, getting 2017. Find all numbers that Michael could have picked.
Result. 2021, 10085
Solution. For the chosen divisor d of n, we have n = kd for some integer k. Now the equation reads

kd − 4d = (k − 4)d = 2017.

Since 2017 is a prime number, we obtain either d = 1 or d = 2017. In the first case, we get n = k = 2017 + 4 = 2021.
In the second case, we obtain k = 5 and hence n = 2017 · 5 = 10085.

6
Problem 20. Grisha and Vanechka are very good friends, so whenever they sit next to each other they begin chatting.
Five students (including Grisha and Vanechka) want to have a constructive discussion, so they want to take five chairs
around a round table in such a way that Grisha and Vanechka are not neighbours. In how many ways can they do it?
Arrangements that differ by rotations are different.
Result. 60
Solution. Once Grisha has taken one of the five places, there are only 2 positions left to seat Vanechka in order not to
be next to Grisha. This yields 5 · 2 possibilities. For each of them, the three remaining students can be seated in 3!
ways, which leads to 5 · 2 · 3! = 60 ways in total.

Problem 21. In the figure, AB is a diameter of a circle with center M . The two points D and C are on the circle in
such a way that AC ⊥ DM and ∠M AC = 56◦ . Find the size of the acute angle between the lines AC and BD in
degrees.
C

A M B

Result. 73◦
Solution. Denote the point of intersection of the lines AC and DM by S and the point of intersection of the lines AC
and BD by T .
C

D T
S

A M B
From ∠M AC = 56◦ and the given right angle at S, we get ∠SM A = 34◦ using the sum of angles of triangle AM S.
Further, ∠BM D = 180◦ − ∠SM A = 146◦ . As both M D and M B are radii of the circle, 4BDM is isosceles
(∠M BD = ∠M DB) and using the sum of angles again results in ∠M DB = 17◦ . Our goal is to find ∠CT B, which
equals ∠ST D; however, this can be computed using the sum of angles of the right triangle DST with the knowledge of
∠SDT = ∠M DB. The result is ∠CT B = 73◦ .

Problem 22. An object is built from unit squares by successively composing its copies as indicated in the figure
below. What is the length of the thick boundary of the object in the stage 6?

1 2 3

Result. 488
Solution. Let fn denote the length of the thick boundary in the n-th stage. Note that in each stage three congruent
figures from the previous stage are glued together along two unit segments which are no longer parts of the boundary.
This observation yields fn+1 = 3 · fn − 2 · 2 for all n ≥ 1 which combined with f1 = 4 gives f6 = 488 after direct
computation.

7
Problem 23. All the 2017 seats around a very large round table are occupied by superheroes and villains. The
superheroes always tell the truth, whereas the villains always lie. Each person sitting at the table reported that he or
she was sitting between a superhero and a villain. For unknown reasons, exactly one superhero made a mistake. How
many superheroes are there?
Result. 1345
Solution. Firstly, observe that there are no villains sitting next to each other: if that was the case, another villain
would have to sit on either side, next to this one another one etc., implying that no superheroes are present, which is
not true because the statement says that there is at least one. Assuming first that all superheros tell the truth, every
superhero sits between another superhero and a villain, so the whole party at the table can be divided into segments
of the form superhero-superhero-villain. The lying superhero can now be either seated between two superheroes, or
inserted together with one more villain between a superhero and a villain. The remainder of the division of the total
number of people by three would be 1 in the former case and 2 in the latter case. Since 2017 gives remainder 1 when
divided by 3, the former case applies and we conclude that there are 23 · 2016 + 1 = 1345 superheroes at the table.

Problem 24. Find all positive real numbers x such that

x2017x = (2017x)x .

Result. 2016 2017
Solution. Since x is positive,
√ we may raise both sides to the power 1/x and obtain x2017 = 2017x or x2016 = 2017.
2016
The solution is x = 2017.

Problem 25. Leo wants to colour the edges of a regular dodecahedron in a special way: He chooses a felt pen of
one colour, starts at one vertex of the dodecahedron and moves along connected edges without lifting his pen and
without colouring any edge twice until he wants to or is forced to stop. Then he takes another felt pen and begins
colouring some connected uncoloured edges. He continues this procedure using one colour at a time until every edge of
the dodecahedron is coloured exactly once. What is the minimum number of colours which can be used?
Note: Regular dodecahedron is a regular solid with twelve pentagonal faces as shown in the picture:

Result. 10
Solution. A regular dodecahedron has 20 vertices and at every vertex 3 edges meet. Now consider only the graph
model consisting of nodes (vertices) and edges of the dodecahedron. After each step of colouring a path of connected
edges with one colour, remove those coloured edges from the model. If a closed path is taken out, at each node either 0
or 2 edges are removed. If the beginning node b and the ending node e of the removed path are different, exactly one
edge is taken out at the nodes b and e whereas at the remaining nodes 0 or 2 edges are eliminated. Therefore, each
node has to be a starting/ending node of at least one path and so there must be at least 10 paths (colours).
The following picture shows that a colouring using 10 colours exists:

8
Problem 26. Landscape gardener Joe designed a new gravel path around a lake. The triangle ABC marks the
middle of the path, its side lengths are a = 80 m, b = 100 m and c = 120 m. The borders of the path have distance 1 m
from this triangle, as can be seen in the following picture. How many m3 of fine loose gravel must Joe order if the
covering with gravel should be of height 4 cm in average?
C

A B

Result. 24
Solution. The area of the path can be subdivided into three trapezoids having height 2 and midline a, b, and c,
respectively.

Since the area of a trapezoid can also be computed by multiplying its height by its midline, we get
2 · (80 + 100 + 120) = 2 · 300 = 600
as the result for the area of the path in m2 . The amount of gravel to be ordered can now be computed as 600 m2 ·0.04 m =
24 m3 . So the answer is 24.

Problem 27. Find all four-digit squares of integers such that the first two digits are equal and the last two digits are
also equal.
Result. 7744
Solution. Let N be the number in question and denote by x and y its first and last digit, respectively. Then we have
N = 1000x + 100x + 10y + y = 11(100x + y),
so N is divisible by 11. But N being a square number, it also has to be divisible by 112 . Thus we get N = 112 k 2 for
some positive integer k and 100x + y = 11k 2 . Since the left-hand side of this equality is a three-digit integer x0y with
the tens digit being zero, k 2 has to be a two-digit square number whose digits sum up to 10. The only number of this
kind is 82 = 64. Therefore, we have 100x + y = 11 · 82 and obtain N = 112 · 82 = 882 = 7744.

Problem 28. At a fun fair there is a lottery drawing with the following rules: a participant has to choose one of four
indistinguishable boxes and afterwards he draws one ball out of the chosen box. If this ball is white, the participant is
a winner, if it is black, he loses. For example, if the distribution of the balls in the four boxes is
(6, 6), (5, 3), (4, 0), (3, 5),
where each pair (w, b) represents a box with w white and b black balls inside, then a participant wins with a probability
of 58 . Every 1000th participant receives a super-joker: he or she is allowed to redistribute all the balls among the boxes
arbitrarily, putting at least one ball in each box. Then the boxes are mixed again and the participant can choose a box
and draw one ball. Johanna is a lucky girl and has won the super-joker. What is the largest probability for a win she
can achieve by using an appropriate distribution of the balls given in the example?
51
Result. 58
Solution. If Johanna puts exactly one white ball each in three of the four boxes, she wins in any case if she chooses one of
these boxes. Given the distribution (1, 0), (1, 0), (1, 0), (15, 14) for (w, b), she wins with probability 14 (1+1+1+ 15 51
29 ) = 58 .
It is easy to see that every other distribution leads to a smaller probability for a win: the probability of a loss is
the sum of probabilities of choosing each single black ball and for a single ball, the probability to be chosen is clearly
minimal if it is in a box with as many balls as possible.

9
Problem 29. A bus, a truck, and a motorcycle move at constant speeds and pass a stationary observer in this order
at equal time intervals. They pass another observer farther down the road at the same equal time intervals, but in
different order: this time the order is bus, motorcycle, truck. Find the speed of the bus in km/h, if the speed of the
truck is 60 km/h, and the speed of the motorcycle is 120 km/h.
Result. 80
Solution. Let t be the common time interval between the moments the vehicles pass the observers. The motorcycle
reaches the first observer t hours after the truck and reaches the second observer t hours before the truck. Therefore,
the motorcycle covers the distance between the two observers 2t hours faster than the truck does. The motorcycle is
travelling twice as fast as the truck, so it covers the distance between the two observers in half the time. Consequently,
the truck must take 4t hours to cover the distance between the observers while the motorcycle takes 2t hours.
Now the bus passes the first observer t hours ahead of the truck and it passes the second observer 2t hours ahead of
the truck. The truck takes 4t hours to go between the observers, so the bus takes 3t hours to go the same distance.
Consequently, the bus is going 43 the speed of the truck and this is 80 km/h.

Problem 30. Find all ways to fill all gaps in the statement below with positive integers in order to make the
statement true: “In this statement, % of digits is greater than 4, % of digits is less than 5, and % of digits
is equal to either 4 or 5”.
Result. 50, 50, 60
Solution. Clearly, there cannot be more than ten digits in the statement. Therefore, using the digits we already know,
we observe that the first two gaps should be filled with at least 20 and the third one with at least 40. Thus there
are altogether ten digits and all numbers in the gaps should end with 0, so we can denote them by a0, b0, and c0,
respectively, with digits a, b, and c such that a + b = 10.
Because there are at least two digits greater than 4 and at least five digits less or equal 4, we can conclude that
5 ≥ a ≥ 2. Moreover, at least one of the digits a and b is greater than 4, so we get 5 ≥ a ≥ 3.
Having already four digits equal to either 4 or to 5 means c ≥ 4. Since the definition of c0 % excludes the case c = 4,
we must have c ≥ 5 which yields 5 ≥ a ≥ 4. In case a = 4, we get b = 6, but then for every c ≥ 5, it is not possible to
obtain a true statement as requested. Now let a = 5. This leads to b = 5 and the statement is true for c = 6.

Problem 31. Paul’s cattle grazes on a triangular meadow ABC. Since his spotted cows and those without spots did
not go well together, Paul built a twenty-meter long fence perpendicular to the side AC, starting at point P on this side
and ending in B, dividing the meadow into two right-angled triangles. However, this was soon met with protests from
the spotted cows (grazing in the part with point A), which pointed out that AP : P C = 2 : 7 and demanded a fairer
division. Consequently, Paul replaced the fence with a new one, parallel to the old one and with ends on the borders of
the meadow, but dividing the meadow into two parts of equal area. What was the length of the new fence in meters?
q
Result. 30 27
Solution. Since triangle P BC has greater area than ABP , the endpoint of the new fence on side AC (denoted by X)
lies between C and P , and the other endpoint (denoted by Y ) lies on side BC. Triangles P BC and XY C are similar,
so XY : XC = P B : P C.
B
Y

C A
X P
Further, as the area of triangle XY C is one half of the area of triangle ABC,
1 1 1
· XY · XC = · · P B · AC
2 2 2
or
1 XY 1 AP + P C
XY 2 = · · P B · AC = · P B 2 · ,
2 XC 2 PC
therefore s  r

1 AP 2
XY = P B · 1+ = 30 .
2 PC 7

10
Problem 32. There were five (not necessarily distinct) real numbers written on a blackboard. For every pair of these
numbers, Wendy calculated their sum and wrote the ten results

1, 2, 3, 5, 5, 6, 7, 8, 9, 10

on the blackboard, erasing the initial numbers. Determine all possible values of the product of the erased numbers.
Result. −144
Solution. Denote the initial numbers by a ≤ b ≤ c ≤ d ≤ e. Among the ten calculated sums, the smallest is a + b = 1,
the second smallest is a + c = 2, the largest is d + e = 10, and the second largest is c + e = 9. Since all ten sums add
up to
4(a + b + c + d + e) = 1 + 2 + 3 + 5 + 5 + 6 + 7 + 8 + 9 + 10 = 56,
we get a + b + c + d + e = 14 and c = 14 − 1 − 10 = 3. It follows that a = 2 − c = −1, b = 1 − a = 2, e = 9 − c = 6, and
d = 10 − e = 4. For these values we can easily check that the remaining six sums exactly fit into the list of numbers on
the blackboard. Therefore, the desired product is −1 · 2 · 3 · 4 · 6 = −144.

Problem 33. Write 333 as the sum of squares of (arbitrarily many) distinct positive odd integers.
Result. 3 + 5 + 72 + 92 + 132
2 2

Solution. Since 172 = 289 < 333 < 361 = 192 , only the numbers 12 , 32 , . . . , 172 (altogether nine distinct numbers)
may appear as summands. Further, whenever an odd integer is squared, the result gives remainder 1 when divided by
8—since 333 has remainder 5 after dividing by 8, the number of summands in the desired sum has to be five.
Since 12 + 32 + 52 + 72 + 172 > 333, 172 cannot appear in the sum. Let us take a look at the leftover summands
modulo 5: Two of them are divisible by 5 (52 , 152 ), three of them give remainder 1 (12 , 92 , 112 ) and three give
remainder −1 (32 , 72 , 132 ). Since 333 gives remainder 3 (or −2), there are two cases to consider. Firstly, we may sum
up all the numbers with remainder 0 or 1; however, this turns out to exceed 333. Secondly, to achieve −2, we have
to sum all the numbers with remainder −1, one with remainder 0 and one with 1. It is easy to see that the results
containing 112 or 152 are too large, and out of the two remaining possibilities, only 32 + 52 + 72 + 92 + 132 is equal to
333.

Problem 34. Ellen picked three real numbers a, b, c and defined the operation as x y = ax + by + cxy. As an
exercise, she computed that 1 2 = 3 and 2 3 = 4. After further investigation, she noticed that there was a non-zero
real number u such that z u = z for every real z. What was the value of u?
Result. 4
Solution. From 0 = 0 u = bu we get b = 0 (u is non-zero). The equations in the statement now may be rewritten as

a + 2c = 3,
2a + 6c = 4

with solution a = 5, c = −1. Finally 1 = 1 u = 5 − u yields u = 4.

Problem 35. Three circles of radius r and one circle of radius 1 touch each other and a straight line as shown in the
picture. Find r.


1+ 5
Result. 2
Solution. Denote certain points of contact by A, B, and T and centers of circles by X, Y , and Z as shown in the

11
following picture:

Z
r
r
T r
Y
1
r
X 1 r
1

A 2 r B

Applying the Pythagorean theorem, we get

AB 2 = XY 2 − (BY − AX)2 = (r + 1)2 − (r − 1)2 = 4r.

As BY k T Z and BY = T Z = r, the quadrilateral BY ZT is a parallelogram and BT = Y Z = 2r. Since the


Pythagorean theorem in triangle ABT yields AB 2 + AT 2 = BT 2 , we now obtain 4r + 22 = (2r)2 , which reduces to

r2 − r − 1 = 0.

1+ 5
The only positive solution of this quadratic equation is r = 2 .

Problem 36. On an old concrete wall, someone had sprayed five (not necessarily distinct) real numbers, the sum of
which was 20. For each pair of these numbers, Harry computed their sum and rounded it down (i.e. took the greatest
non-exceeding nearest integer), thus obtaining ten integers. Finally, he added together all these new numbers. What
was the smallest possible value of the sum that Harry could get?
Result. 72
Solution. Let a1 , . . . , a5 be the numbers on the wall. We are seeking the smallest possible value of
X
W = bai + aj c,
1≤i<j≤5

where bxc denotes the greatest integer not exceeding x. We can rewrite this expression as
X X X X
bai + aj c = (ai + aj ) − {ai + aj } = 80 − {ai + aj },
1≤i<j≤5 1≤i<j≤5 1≤i<j≤5 1≤i<j≤5

where {x} stands for the fractional part of x (i.e. {x} = x − bxc). Therefore our goal is to maximize the sum
X
S= {ai + aj }.
1≤i<j≤5

This can be further split into two sums


5
X 5
X
{ai + ai+1 } + {ai + ai+2 }
i=1 i=1

(letting a6 = a1 and a7 = a2 ). Now the equalities


5
X 5
X
S1 = {ai + ai+1 } = 40 − bai + ai+1 c,
i=1 i=1
5
X 5
X
S2 = {ai + ai+2 } = 40 − bai + ai+2 c
i=1 i=1

imply that both S1 , S2 are integers. Further, they are both sums of five numbers less than 1, and so both are at most
4; consequently, S ≤ 8. For the original sum we thus obtain W ≥ 72 and the equality is achievable by letting all the
fractional parts of a1 , . . . , a5 be 0.4, for example a1 = 2.4, a2 , . . . , a5 = 4.4.

12
Problem 37. For a composite positive integer n, let ξ(n) denote the sum of the least three divisors of n, and ϑ(n)
the sum of the greatest two divisors of n. Find all composite numbers n such that ϑ(n) = (ξ(n))4 .
Note: By a divisor we mean a positive, not necessarily proper divisor.
Result. 864
Solution. The smallest divisor of n is 1. Let the second and third smallest divisors of n be p, q respectively (so p is a
prime and q is either a prime or q = p2 ). Then ξ(n) = 1 + p + q and ϑ(n) = n + n/p. So after multiplying by p, the
condition from the statement can be rewritten as

n(p + 1) = p(1 + p + q)4 .

The right-hand side is divisible by p + 1 and since p and p + 1 are coprime, p + 1 | (1 + p + q)4 . If both p, q were odd,
p + 1 would be even and (1 + p + q)4 odd, which is not possible, therefore p = 2 (2 has to be a divisor of n and p is
the smallest prime divisor). Now expanding (1 + p + q)4 = (3 + q)4 via the binomial formula and omitting the terms
divisible by 3 shows that 3 | q 4 , therefore 3 | q. The option q = p2 being clearly impossible, we conclude that q is a
prime and so q = 3. Finally, n · 3 = 2 · 64 , so n = 25 · 33 = 864.

Problem 38. On a 3 × 3 board, a mouse sits in the bottom left square. Its goal is to get to a piece of cheese in the
top right square using only moves between adjacent squares. In how many ways can we fill some (possibly none) of the
free squares with obstacles so that the mouse is still able to reach the cheese?

Result. 51
Solution. Firstly, let us consider the case when we fill the centre square. In such a case the mouse can reach the cheese
only by moving along one of the “border corridors”, so we may place further obstacles to only one of these corridors.
Each corridor consists of three squares, thus there are 23 = 8 ways to fill one of them and 8 + 8 − 1 = 15 ways to leave
at least one corridor free (the −1 is due to the situation with both corridors free being counted twice).
Let us now focus on the case with the centre square free. The path to cheese exists if and only if at least one of the
squares adjacent to the mouse and at least one adjacent to the cheese is free. This gives three options for the two
squares adjacent to the mouse and three options for the two squares adjacent to the cheese (i.e. at most one filled
square) and two options for each of the remaining two corners, so the number of placements plausible for the mouse in
this case is 3 · 3 · 2 · 2 = 36.
We conclude that there are 15 + 36 = 51 ways in total.

Problem 39. Among all the pairs (x, y) of real numbers satisfying

x2 y 2 + 6x2 y + 10x2 + y 2 + 6y = 42,

let (x0 , y0 ) be the one with minimal x0 . Find y0 .


Result. −3
Solution. Firstly, we observe that if we add 10 to both sides, the binomial x2 + 1 can be factored out from the left-hand
side, so
(x2 + 1)(y 2 + 6y + 10) = 52.
The expression on the left-hand side can be viewed as a product of two quadratic functions f (x) = x2 + 1 and
g(y) = y 2 + 6y + 10. Now since the graphs of quadratic functions are symmetric and in this case, both functions are
first decreasing and then increasing, the minimal value x0 gives the maximal value of f , which in turn forces g(y0 )
to be minimal possible (the product f (x)g(y) is a positive constant). So it suffices to find where g(y) = (y + 3)2 + 1
attains its minimum and we conclude that the answer is y0 = −3.

13
Problem 40. Let 4ABC be a right-angled triangle with right angle at C. Let D and E be points on AB with D
between A and E such that CD and CE trisect ∠ACB. If DE : BE = 8 : 15, find AC : BC.

4 3
Result. 11
Solution. Let P be a point on BC such that DP k AC. As the triangles ABC and DBP are similar, AC : BC =
DP : BP .
A
D
E

B C
P
Since CE is the angle bisector in triangle BCD, the Angle bisector theorem implies CD : CB = ED : EB = 8 : 15.
Further, DP = CD · sin 60◦ and BP = CB − CP = CB − CD · cos 60◦ , therefore
√ √
3
DP 2 CD 4 3
= 15 1 = .
BP 8 CD − 2 CD
11

Problem 41. Mike plays the following game: His task is to find an integer between 1 and N (inclusive). In each
turn, he picks an integer from this interval; if it is the correct one, the game ends, otherwise he is told if his choice was
too large or too small. However, if the picked number is too large, he has to pay 1 e, and if it is too small, he pays 2 e
(and he pays nothing if his choice is correct). What is the largest integer N such that Mike can always finish the game
provided that he can spend at most 10 e?
Result. 232
Solution. Denote by Nk the maximal N such that with k Euros Mike can always find the integer from the range
1, . . . , N (or, equivalently, any set of integers of length N ); our goal is to compute N10 . Clearly N0 = 1 and N1 = 2.
Let us show that the numbers satisfy the recurrence relation

Nk+2 = Nk+1 + Nk + 1.

Indeed, if Mike has k + 2 euros and picks the number Nk+1 + 1, then either it is the correct one, or it is too large (in
which situation he proceeds with k + 1 money and a set of length Nk+1 ) or too small (leading to k money and Nk
choices). This shows Nk+2 ≥ Nk+1 + Nk + 1. On the other hand, if there are more than Nk+1 + Nk + 1 numbers to
choose from, then selecting a number greater than Nk+1 + 1 may lead to more than Nk+1 choices but only k + 1 money
left (too large choice) and similarly, selecting a number less than Nk+1 + 2 may result in more than Nk choices but
only k money left (too small choice). Thus the recurrence relation is proved.
The number N10 = 232 now may be computed via the recurrence in a straightforward way.

Problem 42. Positive integers a, b, c satisfy a ≥ b ≥ c and

a + b + c + 2ab + 2bc + 2ca + 4abc = 2017.

Find all possible values of a.


Result. 134
Solution. Observe that if we multiply the left-hand side by 2 and add 1, we get

1 + 2a + 2b + 2c + 4ab + 4bc + 4ca + 8abc = (2a + 1)(2b + 1)(2c + 1).

The same operation applied on the right-hand side yields 2·2017+1 = 4035. Factorising into primes gives 4035 = 3·5·269,
and since a ≥ b ≥ c ≥ 1, the equalities 2a + 1 = 269, 2b + 1 = 5, and 2c + 1 = 3 must hold. Therefore the result is
a = 134.

14
Problem 43. An alien spacecraft has the shape of a perfect ball of radius R, supported by three parallel vertical
straight legs of length 1 af (alien fathom) and negligible width. The lower ends of the legs form an equilateral triangle
of side length 9 af and when the spacecraft rests on a flat surface, the lowest point of the ball precisely touches the
surface. What is the radius R (in af)?

Result. 14
Solution. Denote lower ends of the legs by A, B, C and their upper ends by A0 , B 0 , C 0 , respectively. Let O be the
center of triangle ABC (i.e. the point of tangency the ball to the ground), S be the center of the spacecraft and
√ √
D = AO ∩ BC. We have AO = AB · 33 = 3 3 af, AA0 = 1 af and SO = SA0 = R. Pythagorean theorem yields

(SO − AA0 )2 + AO2 = SA02 or (R − 1)2 + 27 = R2 ,

from which we get R = 14 af.


S

A
14

9 3 3
O

9
B 2 D C
A0 √
2 7

A 3 3
O D

Problem 44. Four brothers, Allan, Bert, Charlie, and Daniel, had collected a large amount of hazelnuts in a forest.
At night, Allan woke up with the irrestible urge to eat, so he decided to eat some of them. After counting the hazelnuts,
he found out that if one hazelnut was removed, the rest could have been divided into four equal parts, so he discarded
the extra hazelnut and ate his share (one fourth of the rest); then he returned to bed. Later that night, Bert woke
up because of hunger, too, and found out that there was again an extra nut, which he removed and ate one fourth
of the remaining nuts. Until morning, exactly the same happened to Charile and Daniel. When all brothers met in
the morning, they found out that the number of the remaining nuts was still such that it was divisible by four after
removing one nut. What was the minimal number of hazelnuts the brothers could have collected?
Result. 1021
Solution. Let us add three additional, “fake” nuts at the beginning of the night. Then the number of all nuts is a
multiple of four. It is enough to observe that after Allan’s supper the number of the remaining nuts in the bag is also
a multiple of four and there are still the three fake nuts among them. By repeating this reasoning we get that the
number of nuts with the fake ones included is equal to 45 k for some positive integer k, so the actual number of nuts is
45 k − 3. Plugging k = 1 gives the desired minimum 45 − 3 = 1021.

Problem 45. We call a positive integer convenient if all its prime divisors are among 2, 3, and 7. How many
convenient numbers are among 1000, 1001, . . . , 2000?
Result. 19
Solution. We begin with a general observation: If x is a real number with x > 1/2, then there is exactly one (integer)
power of 2 in the half-open interval [x, 2x).
Further, call a positive integer strongly convenient if all its prime divisors are among 2 and 3. The number of
strongly convenient numbers in the interval [x, 2x) can be computed as follows: Firstly, there is exactly one power of
two in the interval [x, 2x). Further, in the interval [x, 2x) there is at most one strongly convenient number (say c1 )
divisible by 3, but not by 32 , for c1 /3 is the only power of two in the interval [x/3, 2x/3), provided that x > 1/6. This

15
way we proceed and find a strongly convenient number c2 divisible by 32 but not by 33 etc., until [x/3k , 2x/3k ) ∩ N = ∅,
i.e., x < 3−k /2. We infer that the number of strongly convenient numbers in the interval [x, 2x) is 1 + the greatest k
satisfying 3k < 2x; denote this number by `3 (x).
To obtain the number of convenient numbers in the interval [x, 2x), we employ a similar technique: The sought
number is the sum of numbers of strongly convenient numbers in the intervals [x, 2x), [x/7, 2x/7), [x/72 , 2x/72 ) etc.
Thus the quantity can be computed as `3 (x) + `3 (x/7) + `3 (x/72 ) + . . . , the sum having `7 (x) terms, with the symbol
`7 defined in a similar fashion as `3 .
Finally, as 2000 is not a convenient number, our task is to compute
`3 (1000) + `3 (1000/7) + `3 (1000/49) + `3 (1000/343) = 7 + 6 + 4 + 2 = 19,
which is the desired quantity.

Problem 46. Emperor Decimus has banned using the digit 0 (introduced by his predecessor Nullus) and ordered the
digit D, representing the quantity 10, to be used instead, thereby founding the Decimus notation. Each positive integer
still has a unique representation, e.g.
3DD6 = 3 · 1000 + 10 · 100 + 10 · 10 + 6 · 1 = 4106.
To smoothen the transfer to the new system, a list of all integers from 1 to DDD (inclusive) has been written down.
How many occurrences of the new digit D are there in the list? Multiple occurrences in a single number are counted
with multiplicity, so e.g. DD is counted as two D’s.
Result. 321
Solution. Observe that all k-digit numbers (in Decimus notation) are represented exactly by the strings
1. . .1}, . . . , D.
| {z . .D} .
| {z
k k

Therefore, to count the total number of D’s occurring in the k-digit numbers, we can group together the numbers
having D on the first position, the second etc. up to the k-th position. There are exactly 10k−1 such numbers for each
position—we have to fill the remaining k − 1 positions with symbols 1, . . . , 9, D. Since for each number we count all
occurrences of D, we may distribute the ones with multiple occurrences into all corresponding groups and the result
stays correct, so there are k · 10k−1 digits D among the k-digit numbers.
It is easy to see that only numbers consisting of one, two, or three digits appear in the list 1, . . . , DDD. We conclude
that among them the digit D appears 1 · 100 + 2 · 101 + 3 · 102 = 321 times.

Problem 47. Square ABCD has inscribed circle ω, which touches the square at points W , X, Y , Z lying on sides
AB, BC, CD, DA respectively. Let E be an inner point of the (shorter) arc of ω between W and X and F the
intersection of BC and EY . Provided that EF = 5 and EY = 7, determine the area of triangle F Y C.
Result. 21
Solution. By the (limit case of the) Inscribed angle theorem, ∠EXF = ∠EY X. Consequently, triangles F EX and
F Y X are similar (they also share the angle at F ), therefore EF : XF = XF : Y F or XF 2 = EF · Y F = 5 · 12 = 60 (a
fact also known as the Power of a point theorem).
A Z D

W Y

F
B X C

Let t = 12 AB be one half of the side length of the square. Then the Pythagorean theorem gives
Y F 2 = t2 + (t + XF )2 = 2t2 + 2t · XF + XF 2 = 2t(t + XF ) + XF 2 ,
hence the sought area of triangle can be computed as
1
2 · Y C · CF = 1
2 · t · (t + XF ) = 14 (Y F 2 − XF 2 ) = 21.

16
Problem 48. In the cryptogram
WE · LIKE = NABOJ
different letters stand for different digits. Furthermore, we are given S(WE) = 11, S(LIKE) = 23, and S(NABOJ) = 19,
where S(n) denotes the sum of digits of a positive integer n. None of the three numbers may start with zero. Find the
five-digit number NABOJ.
Result. 60 724
Solution. Since in the given cryptogram ten different letters show up, all ten digits 0, 1, . . . , 9 are involved. The sum
of all digits is 45. By adding up the three given digital sums, we get 11 + 23 + 19 = 53, which is equal to 45 + E. This
yields E = 8, and by squaring it, J = 4. Using S(WE) = 11, we also get W = 3.
Next, observe that
100000
LIKE < < 2636.
38
Therefore, we have L = 1 or L = 2. In case L = 2, the digital sum of LIKE leads to I + K = 13, which means that
(I, K) or (K, I), respectively, can be one of the pairs (4, 9), (5, 8), or (6, 7). But none of them is possible any more, since
the digits 4 and 8 are already taken and in case of (6, 7), the factor LIKE would exceed the upper bound above. From
that we conclude L = 1. Then the digital sum of LIKE leads to I + K = 14, which restricts the options for (I, K) or
(K, I), respectively, to the pair (5, 9). An easy calculation shows that only (I, K) = (5, 9) solves the cryptogram. From
38 · 1598 = 60 724 we obtain the desired number NABOJ = 60 724.

Problem 49. Find all positive integers n with the property that the sum of all non-trivial divisors is 63.
Note: A non-trivial divisor d of n fulfils 1 < d < n.
Result. 56, 76, 122
Solution. Denote by s(n) the sum of all non-trivial divisors of a positive integer n. It is easy to analyse that there
is no solution of the problem, if n has three or more different prime factors: s(2 · 3 · 5) = 41, s(2 · 3 · 7) = 53 and
higher values of n give s(n) greater than 63. Furthermore, if n were a power of a single prime number p, s(n) would be
divisible by p, so p = 3 or p = 7. However, we may easily check that no power of 3 or 7 satisfies the given condition.
Therefore n possesses exactly two different prime factors p, q and a possible solution can be expressed as n = pα · q β .
If α = β = 2, then p = 2, q = 3 gives s(n) = 54 and all other combinations of prime factors or greater exponents result
in s(n) > 63. Therefore at least one of the two exponents has to be smaller than 2; in particular, n is not a square of
an integer, and as such, it has an even number of (non-trivial) divisors. Thus if all the divisors were odd, s(n) would
be an even number, which is not the case—we infer that (w.l.o.g.) q = 2.
For the same reasons, the number of odd non-trivial divisors (which are precisely p, p2 , . . . , pα ) is odd, hence α is
odd. However, even for p = 3, α = 3, and β = 1, s(n) exceeds 63, therefore α = 1. We conclude that

s(n) = 2 + 22 + · · · + 2β + p + 2p + · · · + 2β−1 p = (2β − 1)(p + 2).

Out of all divisors of 63, only 1, 3, and 7 are of the form 2β − 1. These yield 61, 19, and 7 as corresponding values of p
and so the possible values of n are 2 · 61 = 122, 22 · 19 = 76, and 23 · 7 = 56.

Problem 50. Bob has a cool car with square-shaped rear wheels (the front wheels are the standard round ones).
Such a car would usually be rather unpleasant to drive, but Bob has installed very good shock absorbers for the rear
wheels so that the car stays in a fixed position parallel to the surface while driving on a flat road. The side length of
the rear wheel is 40 cm and its axle is fixed with respect to the car in the horizontal direction. What is the radius of
the front wheel (in cm) provided that when the car is moving with constant forward speed, the axle of the rear wheel is
exactly half of the time lower and half of the time higher from the surface than the axle of the front wheel?


Result. 10 7

17
Solution. Consider the trajectory of the center of the square as the car moves forward—it consists of quarter-circles
centered in a vertex of the square.


2r
4

If the car maintains constant forward speed, this quarter-circle can also be viewed as the graph of the height of the
rear axle as a function of time. Therefore the sought radius of the circular wheel is the height
√ precisely in one quarter
of the horizontal distance. Let r be the radius of the arc; then this one quarter equals 2r/4. Using the Pythagoras
theorem, we obtain that the height at that moment is
r  √  q 2
2 7
r2 − 4 r = 8 r.


The result follows by plugging in r = 20 2.

Problem 51. The City of the Future has the shape of a regular 2017-gon. In vertices of the city there are 2017
metro stations, labelled 1, 2, . . . , 2017 counterclockwise on the city map. There are two metro lines: side and diagonal.
The side line provides direct connection from station a to b (but not in the opposite direction) if and only if a − b + 1 is
divisible by 2017 and one such ride lasts 1 minute. The diagonal line provides direct connection from station a to b if
and only if 2b − 2a + 1 is divisible by 2017 and one ride lasts 15 minutes. Ferdinand is an avid metro traveller and
starting from station 1 he wants to get to the station n with the following property: the shortest possible time needed
for getting to this station is the longest among all the stations. Find values n of all possible destinations of Ferdinand.
Result. 1984, 1985
Solution. Whenever Ferdinand travels one stop by diagonal line and then one stop by side line, he reaches the same
destination in the same time as if he first took side and then diagonal line. Therefore we may w.l.o.g. assume that first
he uses only diagonal line and then only side line. Further, if his journey consisted of at least two diagonal rides and at
least one side ride, it could be replaced by a less time-consuming one (while keeping the destination) by omitting the
last two diagonal and the first side ride, which together form an empty move in the metro network. Hence we may
consider only journeys either using only diagonal line or using it at most once.
Note that if n ≥ 1009, then using diagonal line once and side line n − 1009 times, Ferdinand will get to station n
in 15 + (n − 1009) minutes. Obviously, using side line only cannot result in a shorter journey. On the other hand, if
travelling only by diagonal line, he can reach station n in 2 · (2018 − n) · 15 minutes. Thus we want to find n ≥ 1009 for
which M (n) = min{n − 994, 30 · 2018 − 30n} is the largest possible. We have
30 · 2018 + 994 31 · 2018 − 1024 1 1
n − 994 ≤ 30 · 2018 − 30n ⇐⇒ n ≤ = = 2018 − 33 − = 1985 − .
31 31 31 31
Hence, for n ≤ 1984: M (n) = n−994 ≤ 1984−994 = 990 and for n ≥ 1985: M (n) = 30·(2018−n) ≤ 30·(2018−1985) =
990, so the desired largest minimum equals 990 and is achievable for n = 1984 and n = 1985.
It remains to check that for n < 1009 it is possible to get to station n in less than 990 minutes: for n ≤ 990
Ferdinand can travel in n − 1 minutes only by side and for n ≥ 991 he can travel in 15 · (2 · (1009 − n) + 1) ≤ 15 · 37 = 555
minutes only by diagonal.

Problem 52. Let f (n) be the number of positive integers that have exactly n digits and whose digits have a sum of
5. Determine, how many of the 2017 integers f (1), f (2), . . . , f (2017) have the units digit equal to 1.
Result. 202
Solution. Each n-digit number with digit sum of 5 can be represented as 5 ones assigned to some of n places of that
number. Each place can have assigned more ones and the first place (from the left) has to have assigned at least 1 one.
So the count of n-digit numbers with digit sum of 5 is equal to number of ways to distribute 4 ones to n places. In
other words, we are computing number of 4-combinations of n elements with repetition, therefore
 
n+4−1 (n + 3)(n + 2)(n + 1)n
f (n) = = .
4 24

18
In the following, f will refer to this expression instead of the original combinatorial definition.
Let us now count the number of f (n) whose unit digit is 1. To start with, if n has remainder after division by five
0, 2, 3, or 4, then n, n + 3, n + 2 or n + 1 is divisible by 5. Since 24 and 5 are coprime, f (n) is also divisible by 5, so its
last digit is 0 or 5. Therefore the last digit of f (n) can be 1 only when n has remainder 1 after division by 5.
Further, let us observe that f (n) and f (n + 40) have the same units digit. Indeed, when computing f (n + 40) − f (n)
and looking only at the numerators, we may proceed by expanding the expression
   
24f (n + 40) = 40 + (n) 40 + (n + 1) 40 + (n + 2) 40 + (n + 3) ,

leaving the inner brackets unexpanded. After subtracting the term 24f (n) = n(n + 1)(n + 2)(n + 3), the remaining
terms are products of four numbers, out of which either at least two are 40, or exactly three are the brackets containing
n. In the former case the term is divisible by 402 , in the latter at least two of the brackets are consecutive integers,
hence the term is divisible by 80. Therefore, after dividing by 24 = 8 · 3, the difference is divisible by 10 as desired.
Consequently, it suffices to check the units digit of f (n) only for any set of 40 consecutive integers n.
Finally, direct verification shows that
f (n) = f (−3 − n). (?)
Taking into account all the facts, we need only to compute f (1) = 1, f (6) = 126, f (11) = 1001, and f (16) = 3876.
Identity (?) now implies that among the remaining four numbers of the form f (5k + 1), i.e. f (−4), f (−9), f (−14), and
f (−19), two have units digit 1 and two have 6. Therefore 4 · 2000/40 = 200 of the numbers f (1), . . . , f (2000) have units
digit of 1 and among f (2001), . . . , f (2017), this holds for f (2001) and f (2011). In total, there are 202 such numbers.

Problem 53. The picture shows two (indirectly) similar hexangular figures having some sides of length a, b, c, d and
A, B, C, D, respectively. If these two figures are put together as in the picture, we get a new hexangular figure being
similar to each of the two smaller ones (directly similar to the right-hand one). Find the ratio A : a.

B
b D

c
a
d A
q √
1+ 5
Result. 2
Solution. Let us call the three similar figures small, middle and large, respectively. Denote by p the sought ratio. Due
to the similarity of the small and the middle figure we get

p = A : a = B : b = C : c = D : d.

Furthermore we easily see the equations D = B + a and C + b = A + d. The ratio B : A in the middle figure corresponds
to the ratio C : c in the large one. Hence we have B : A = p and B = p2 a as a consequence. By analogous observations
in the middle and the large figures we get b : a = C : B = D : C and therefore
d c b
d:a= · · = p3 .
c b a
Plugging in all these results into equation D = B + a we get pd = p2 a + a = a(p2 + 1) and thus p4 = p2 +√
1. The
only positive solution of the equation p4 − p2 − 1 = 0 as a quadratic equation with unknown p2 is p2 = 1+2 5 . As a
consequence we get s √
1+ 5
p= ,
2
the square root of the golden ratio as the sought result.

Problem 54. Determine all pairs of positive integers (a, b) such that all the roots of both the equations

x2 − ax + a + b − 3 = 0,
x2 − bx + a + b − 3 = 0

are also positive integers.


Result. (2, 2), (6, 6), (7, 8), (8, 7)

19
Solution. Let k, l be the roots of the first equation and m, n the roots of the second one. It is easy to see that if we
have one solution (k, l, m, n), we can swap k and l or m and n or both and get another solution—therefore we will
write only one of these solutions. According to Vieta’s formulas,

k + l = a, m + n = b, kl = mn = a + b − 3.

Combining these we obtain


kl + mn = 2a + 2b − 6 = 2k + 2l + 2m + 2n − 6,
which can be rearranged to
(k − 2)(l − 2) + (m − 2)(n − 2) = 2.
If both summands (k −2)(l −2) and (m−2)(n−2) are positive, i.e. equal to 1, we obtain solutions (k, l, m, n) = (3, 3, 3, 3)
and (1, 1, 1, 1). If one of the summands is zero, there are solutions (k, l, m, n) = (2, 6, 3, 4) and (3, 4, 2, 6).
The remaining case is that one of the summands is negative; for this to happen, one of k, l, m, n has to equal 1.
W.l.o.g. let k = 1, then l = mn and we can modify the equation to

2 = −(l − 2) + (m − 2)(n − 2) = −mn + 2 + mn − 2m − 2n + 4 = −2m − 2n + 6

or m + n = 2, which yields m = n = 1 and l = mn = 1. We infer that there in no solution with a negative summand.
Finally, the possible values of (a, b) = (k + l, m + n) are (6, 6), (8, 7), (7, 8), (2, 2). We can easily check that all of
these values of (a, b) satisfy the conditions from the statement.

Problem 55. Triangle ABC with AB = 3, BC = 7, and AC = 5 is inscribed in circle ω. The bisector of angle
∠BAC meets side BC at D and circle ω at a second point E. Let γ be the circle with diameter DE. Circles ω and γ
meet at E and a second point F . Determine the length of AF .
Result. √30
19
Solution. It is a well-known fact that the perpendicular bisector of segment BC passes through point E. Let that
bisector meet circle ω at second point G (so that EG is a diameter of ω) and M the midpoint of BC.

B A
G
ω

γ D δ
M

F
C

Using Thales’ theorem in circles ω and γ, respectively, we infer that ∠GF E = ∠DF E = 90◦ , implying that points G,
D, F are collinear. Further, ∠GM D = 90◦ and ∠GAD = ∠GAE = 90◦ (Thales’ theorem again) shows that the points
D, M , G, A are concyclic; denote their common circle by δ. Now using inscribed angles (in the circles written above
the equality sign) leads to
ω
∠AF D = ∠AF G = ∠AEG = ∠AEM
and
ω δ
∠F AD = ∠F AE = ∠F GE = ∠DGM = ∠DAM = ∠EAM.
It follows that triangles AF D and AEM are similar. In the same fashion we deduce
ω
∠ACD = ∠ACB = ∠AEB

which, together with ∠DAC = ∠EAB (AE is angle bisector), proves the similarity of triangles CAD and EAB.
Therefore
AC
AD AB · AE AB · AC
AF = AE · = AE · = .
AM AM AM

20
The length of AM can be computed via Median formula as
1p 1√
AM = 2(AB 2 + AC 2 ) − BC 2 = 19
2 2
and we conclude that
3·5 30
AF = 1
√ =√ .
2 19 19

Problem 56. Determine the number of ordered triples (x, y, z), where x, y, z are non-negative integers smaller than
2017 such that
(x + y + z)2 − 704xyz
is divisible by 2017.
Result. 20172 + 1 = 4 068 290
Solution. Condition 2017 | (x + y + z)2 − 704xyz can be rewritten as (x + y + z)2 ≡ 704xyz (mod 2017). In the
following, all congruences are considered modulo 2017. Since 2017 is a prime, for each positive integer a < 2017 there
exist exactly one positive integer smaller than 2017, denoted here by a−1 , satisfying a · a−1 ≡ 1.
First, let us find the triplets (x, y, z) where all three x, y, and z are non-zero. Let y ≡ kx and z ≡ lx (such k and l
always exist: k ≡ y · x−1 , l ≡ z · x−1 ). Substituting in the condition from the statement yields (x + kx + lx)2 ≡ 704klx3
and after multiplying by (x−1 )2 we get (1 + k + l)2 ≡ 704klx. Finally, we multiply the congruence by (704kl)−1 ,
resulting in
x ≡ (704kl)−1 (1 + k + l)2 .
This implies that for each k, l ∈ {1, 2, . . . , 2016} there exists exactly one x satisfying the condition from the statement
(all adjustments were invertible). We can choose such k and l in 20162 ways. However, x has to be non-zero—this
occurs if and only if k + l 6≡ 2016. There are 2015 such pairs (k, l), one for each non-zero k excluding 2016. Thus there
are 20162 − 2015 desired triplets (x, y, z) with x, y, z 6= 0.
If x = 0 and y and z are non-zero, then we get (y + z)2 ≡ 0 which holds if and only if y ≡ −z; there are 2016 such
triplets (0, y, z). In the same fashion we get 2016 triplets for y = 0 and z = 0. If two of the integers x, y, z equal zero,
then the third one has to be zero, too, so we get one more triplet (x, y, z) = (0, 0, 0).
To sum up, the total number of desired triplets (x, y, z) is

20162 − 2015 + 3 · 2016 + 1 = 20162 + 2 · 2016 + 1 + 1 = 20172 + 1.

Problem 57. Jane and Thomas play a game with a fair dice, which has two faces painted red, two green, and two
blue. They alternately roll the dice until one of them has seen all three colors during his/her turns, this player being
declared the winner. With what probability does Jane win the game, provided that she is the first one to roll the dice?
Result. 81/140
Solution. Denote by P1 (x, y) the probability that the player who is to roll the dice wins the whole game, provided
that he/she has seen x colors and the other player y colors so far. Let P2 (x, y) be the same probability for the currently
non-rolling player (so that P1 (x, y) + P2 (x, y) = 1). Our goal is to compute P1 (1, 1), i.e. the probability of the starting
player to win right after both players took their first turn.
Since P2 (2, 2) = 23 P1 (2, 2), it follows that P1 (2, 2) = 53 , P2 (2, 2) = 25 . Furthermore, from

P2 (2, 1) = 23 P1 (1, 2),


P1 (1, 2) = 13 P2 (2, 1) + 23 P2 (2, 2)
12 23 27 8
we obtain P1 (1, 2) = 35 , P2 (1, 2) = 35 , P1 (2, 1) = 35 , and P2 (2, 1) = 35 . Finally, we have

P1 (1, 1) = 13 P2 (1, 1) + 23 P2 (1, 2),


81
yielding P1 (1, 1) = 140 .

21
Problem 58. Given the equation
k(k + 1)(k + 3)(k + 6) = n(n + 1),
find the largest integer n for which there exists an integer solution (k, n).
Result. 104
Solution. Firstly, observe that given a solution (k, n), the only different solution with the same value of k is (k, −1 − n).
Of the two integers n, −1 − n, one is always non-negative and the other one negative. As we are interested in the
largest value of n, we may further assume that n ≥ 0. For these numbers the expression n(n + 1) is increasing as a
function of n, hence to get n as large as possible, we have to maximize the left-hand side of the equality.
Denote by P (k) the left-hand side polynomial k(k + 1)(k + 3)(k + 6) = k 4 + 10k 3 + 27k 2 + 18k. We will use the fact
that between two subsequent numbers n(n + 1) and (n + 1)(n + 2) of the right-hand side there is no other number of
this kind. Let us therefore try to approximate P (k) by a polynomial (k 2 + ak + b)(k 2 + ak + (b + 1)) with variable k
and integer coefficients a and b. Comparing the coefficient of k 3 , we start with a = 5. Expanding yields

(k 2 + ak + b)(k 2 + ak + (b + 1)) = (k 2 + 5k + b)(k 2 + 5k + (b + 1))


= k 4 + 10k 3 + (26 + 2b)k 2 + (10b + 5)k + (b2 + b).

Now there is no appropriate integer value for b fulfilling 26 + 2b = 27. Therefore, plugging in b = 0, b = 1 respectively,
we get for sufficiently large absolute values of k the inequalities
(1) (2)
k 4 + 10k 3 + 26k 2 + 5k < k 4 + 10k 3 + 27k 2 + 18k < k 4 + 10k 3 + 28k 2 + 15k + 2.

For numbers of this kind, P (k) is strictly between two consecutive numbers of the form n(n + 1). Hence we have to
analyze for which values of k the two inequalities are valid. Inequality (1) simplifies to 0 < k 2 + 13k = k(k + 13), hence
k > 0 or k < −13. Similarly, (2) gives 0 < k 2 − 3k + 2 = (k − 1)(k − 2), so k > 2 or k < 1. Thus for k < −13 or k > 2
both inequalities are fulfilled and P (k) is between two consecutive numbers of the form n(n + 1). Therefore we have to
analyze only the situations with −13 ≤ k ≤ 2.
Replacing the inequalities (1), (2) above by equalities, one gets satisfactory values of k, since then P (k) equals a
number of the form n(n + 1); therefore there exist solutions (k, n) for k = −13, 0, 1, 2. As P (k) is increasing for k ≥ 0
and we are searching for the maximal appropriate value of P (k), P (0) and P (1) are not significant. Similarly, since
P (k) is clearly decreasing for k ≤ −6, it remains to check k = −5, −4, −3, −2, −1. However, P (k) ≤ 0 for −6 ≤ k ≤ −3
and −1 ≤ k ≤ 0, so only P (−2) is of interest. It is easy to see that out of values P (−13), P (−2), P (2), the first one is
the greatest. For k = −13, (1) becomes equality, therefore n = k 2 + 5k = 104 is the sought number.

Problem 59. The Quarter-Pizzeria delivers pizza in special pentagonal boxes, which are suitable both for a quarter
of a big pizza and for three quarters of a small pizza (as shown in the picture). What is the radius of a small pizza (in
cm) if the radius of a big pizza is 30 cm?

√ p √
Result. 5(1 + 7− 2 7 − 4)
Solution. Let ABDEF be a pentagon similar to the described box with EF = 1 (taking 30 cm as a unit, we receive
the problem’s conditions). Note that AF = EF (radii of a big pizza) and ∠AF E = ∠BAF = 90◦ (central angles in
circle quarters), so there exists a point C such that ACEF is a square. Let K, M be the midpoints of segments BC,

22
BD, respectively and let T be the point of tangency of the big pizza quarter to the segment BD.
F 1 E F E

2x+2y−1

D
D

1 x

T
2−2x−2y
M
x
1−x−y
x
x

A B A 1−2y y y C
B K

Let x be the sought radius of a small pizza, i.e. BD = 2x. Several tangencies give AM = 2x and AB = BT , DE = DT ,
so AB + DE = BD. If BC = 2y, then AB = 1 − 2y, DE = BD − AB = 2x + 2y − 1, CD = 2 − 2x − 2y and
KM = CD/2 = 1 − x − y. Applying Pythagorean theorem to BKM and AKM , we get

y 2 + (1 − x − y)2 = x2 and (1 − y)2 + (1 − x − y)2 = 4x2 .

Thus,
1 − 3x2
y 2 + 4x2 − (1 − y)2 = x2 , so y= .
2
Plugging this into one of the above equations, we get after simplification

9x4 − 6x3 − 2x + 1 = 0.

Note that

9x4 − 6x3 − 2x + 1 = (3x2 )2 + (x − 1)2 − 2 · 3x2 (x − 1) − 7x2


= (3x2 − x + 1)2 − 7x2
√ √
= (3x2 + ( 7 − 1)x + 1)(3x2 − ( 7 + 1)x + 1),

so we obtain an equation √ √
(3x2 + ( 7 − 1)x + 1)(3x2 − ( 7 + 1)x + 1) = 0.
It is now easy to see that the first factor has no roots over the reals and the second one has roots of the form
√ √
 q 
1
1+ 7± 2 7−4 .
6

However, the larger of the two roots is readily seen to be greater than 1/2 which contradicts the inequality

2BD = BD + AB + DE < BC + CD + AB + DE = 2.
√ p √
This means that x = 61 (1 + 7 − 2 7 − 4) and so the answer is

√ √
 q 
30x = 5 1 + 7 − 2 7 − 4 cm.

23
Problem 1. The picture shows a decagon with all sides meeting at right angle. Lengths of some of the sides (shown
as dashed in the picture) are known and given in centimeters.
1000
52 134

45 70
2018

What is the perimeter of the decagon in cm?


Result. 4444
Solution. By swapping the “inner” corners into “outer” corners, the decagon can be transformed into a rectangle of
dimensions 2018 and 70 + 134 = 204. Therefore the perimeter is 2 · (2018 + 204) = 4444.

134

70
2018

Problem 2. The minute hand of this clock is missing. How many minutes have passed since the last full hour, if the
angle between the hour hand and twelve o’clock is 137◦ ?

12

137◦
9 3

Result. 34
Solution. Since the hour hand passes 360◦ : 12 = 30◦ in one hour, it takes 2 minutes to pass 1◦ . Therefore, it passed
137◦ − 4 · 30◦ = 17◦ after four o’clock, which took 17 · 2 = 34 minutes.

Problem 3. Four students, Kevin, Liam, Madison, and Natalie, took an exam. We know that their scores were 2, 12,
86, and 6 in some order. We also know that
• Kevin’s score was pampam than Madison’s score,
• Madison’s score was pampam than Liam’s score,
• Natalie’s score was pampam than Liam’s score,

• Kevin’s score was pampam than Natalie’s score.


where pampam means either “greater” or “smaller” (the same meaning in all four cases). What was the sum of
Madison’s and Natalie’s scores?
Result. 18
Solution. One can see that if pampam stands for greater, then Kevin has the largest score and Liam has the smallest
score, and if pampam means smaller, it is just the other way round. In any case, Madison and Natalie always have the
middle two scores, namely 6 and 12. Therefore the sought sum is 18.

1
Problem 4. Jack and John are standing in a square and counting houses around. However, each starts counting
(clockwise) at a different house, so Jack’s house no. 4 is John’s 16, and Jack’s 12 is John’s 7. How many houses are
there in the square?
Result. 17
Solution. Since Jack’s 4 is John’s 16, there is a segment of houses where John’s numbering is larger by 12. However,
this segment has to end before Jack reaches 12, since otherwise John would get 12 + 12 = 24. Since the number always
drops by the total number of houses when the end is reached, we see that there are 24 − 7 = 17 houses on the square.

Problem 5. Doris has to decalcify the coffee machine. According to the instruction manual, she should mix four
parts of water and one part of a 10% vinegar concentrate. Unfortunately, she can only find a bottle of 40% vinegar
concentrate in her cupboard. How many parts of water have to be mixed with one part of 40% vinegar concentrate in
order to get the prescribed concentration for decalcifying the coffee machine?
Note: Vinegar concentrate of n% consists of n parts of vinegar and 100 − n parts of water.
Result. 19
Solution. In the original recipe, the vinegar makes 10% of one of 5 parts. The same concentration is achieved if the
vinegar makes 40% = 4 · 10% of one of 4 · 5 = 20 parts. Hence we need 19 extra parts of water.

Problem 6. If g is parallel to h and the angles at A and C are 105◦ and 145◦ as indicated in the picture, what is the
measure of angle ∠CBA?
A g

105◦
B
145◦
h
C

Result. 110◦
Solution. We can add points D, E lying on h, g, respectively, so that the known angles and the angle in question
become the internal angles of pentagon ABCDE. Since the sum of the newly added angles is 180◦ (we can make them
right as in the figure below, but that is not necessary) and the sum of internal angles of any pentagon is 540◦ , we infer
that the sought value is 540◦ − 180◦ − 105◦ − 145◦ = 110◦ .
A E g

105◦
B 110◦
145◦
h
C D

Problem 7. If ABCD is a square, what is the measure of angle ε (in degrees)?

A D

ε
ε

B C

Result. 67.5◦
Solution. Let X, Y be the two points with angle ε. Then ∠AXY = ∠AY X = ε. Further, since ∠XAY = ∠CAB = 45◦ ,
the interior angles of triangle XY A satisfy
45◦ + ε + ε = 180◦
or ε = 67.5◦ .

2
Problem 8. Cederic was born to his mother when she celebrated her 27th birthday. At most how many times can it
happen that Cederic’s age is the same as his mother’s age read backwards?
Note: Possible leading zeros of numbers are ignored, e.g. 470 read backwards is 74.
Result. 7
Solution. Let Cederic be c years old and his mother m years old, c being equal to m reversed. The numbers c and m
have the same number of digits (with c possibly having a leading zero, if m ends with a zero), which is at least 2. Let a
and b be the units digit of c and m, respectively. As Cederic’s mother is 27 years older, we see that either a + 7 = b or
a + 7 = 10 + b. If the mother was at least 100 years old, the difference of the first digits of their ages could be at most
1, which is not possible, as those are precisely the digits b and a. Thus, both numbers c and m have 2 digits.
Therefore, we want to find all numbers ab such that

ab = ba + 27.

We know that a > b, so the condition a + 7 = b cannot hold. Hence we consider the condition a + 7 = 10 + b or
a = b + 3. Since a ≤ 9, we have b ≤ 6. For every digit b ∈ {0, 1, . . . , 6}, we get digit a as a = b + 3. It is easy to see
that for these digits the equality (b + 3)b = b(b + 3) + 27 holds. So the desired situation can happen 7 times: When
Cederic has 3, 14, 25, 36, 47, 58, and 69 years.

Problem 9. Julia uses 32 white and 32 black cubes of side length 1 to form one big cube of dimensions 4 × 4 × 4.
She wants that the surface of the new cube contains as many white faces of the unit cubes as possible. What is the
maximum possible proportion of the big cube’s surface area that is white?
Result. 3/4
Solution. If a unit cube is placed in a corner of the big cube, then three of its faces are visible, if it is on one of the
edges, two faces are visible, otherwise only one face is shown. There are eight corners and each of the twelve edges of
the big cube consists of two unit cubes—altogether 32 places, and it is clear that the highest proportion of white area is
achieved if the white cubes are placed precisely to these places. In this placement every face of the big cube looks the
same, showing twelve white and four black faces. Hence the proportion for the whole surface is equal to 12/16 = 3/4.

Problem 10. One hundred people took part in the selection of the crew for a flight to Mercury. Each potential
astronaut underwent three tests checking certain health, psychological, and experience criteria. Only twenty-six people
passed the health check successfully. Moreover, sixty participants failed more than one of the three tests. Finally,
there were eighty-three people failing either the psychological or the experience test, but nobody failed these two
simultaneously. How many of the participants were chosen for the mission, i.e. passed all the three tests?
Result. 3
Solution. Since nobody failed in both the psychological and the experience test, all the participants failing in at least
two tests must have failed because of their health. This gives us (100 − 26) − 60 = 14 people failing only the health
check. Together with 83 people failing in psychology or experience, there were 97 participants dismissed, hence only 3
astronauts from the whole group were selected.

Problem 11. Square A has two sides that coincide with the radii of a circle, square B has two vertices on the same
circle and shares a part of its edge with square A. Find the ratio of the area of square A to the area of square B.

Result. 5 : 4
Solution. Denote by s the side length of square B. Observe that by symmetry, the midpoint of the circle divides the
side of square B which lies on the diameter in two equal parts of length s/2. Then the Pythagorean theorem yields
 s 2 5 2
r2 = + s2 = s
2 4

3
and therefore the ratio is 5 : 4.

r
s
2

s r

Problem 12. Determine the last two digits of the product


2 · 3 · 5 · 7 · 11 · 13 · 17 · 19 · 23 · 29 · 31 · 37.

Result. 10
Solution. Since there is 2 · 5 in the product, the units digit will be 0. We obtain the tens digit as the last digit of the
product 3 · 7 · 11 · . . . · 37. It is sufficient to consider only the units digits of multiplicands. Moreover, we can ignore
ones. So we have to determine the last digit of the product
3 · 7 · 3 · 7 · 9 · 3 · 9 · 7 = 3 · 7 · 3 · 7 · 3 · 7 · 9 · 9.
As 3 · 7 = 21 has the units digit 1, we can remove pairs of 3 and 7. After this, there is only 9 · 9 left, whose units digit
is again 1. Therefore, the last two digits of the given product are 10.

Problem 13. A detective interrogated first five of six suspects of a crime. He found out that they have 1, 2, 3, 4,
and 5 friends among all the six suspects, respectively. He knows that friendship is symmetric and decided to figure out
the number of friends of the last suspect before his interrogation. How many was it?
Result. 3
Solution. Let n be the number of friends of the last suspect. The suspect with five friends is a friend of everyone else,
so omitting him from the group simply decreases everyone’s number of friends by one. Then we can also omit the
suspect with one friend only, since his number of friends dropped to zero and he is no longer significant in any way.
This way we obtain a group of four suspects, about which we know that they have 1, 2, 3, and n − 1 friends among
themselves, respectively. Repeating the two steps produces yet smaller group with values 1 and n − 2; it is clear then
that n − 2 = 1 or n = 3.
Note: This solution can be turned to a way of producing such a group of suspects. The result is shown in the following
diagram:

Problem 14. The die in the picture has a positive integer written on each of its faces. Moreover, the product of the
numbers on opposite faces is the same for all such pairs. The numbers on the faces do not have to be distinct. What is
the smallest possible value of the sum of all the numbers on the die?
12

9 6

4
Result. 40
Solution. Let P be the product of numbers on opposite faces. Clearly, the higher P , the higher the total sum. Since
P has to be divisible by all the three shown numbers, its smallest value of P is their least common multiplier, P = 36.
Under these circumstances, the numbers not shown are 3, 4, and 6 and the sought sum is 6 + 9 + 12 + 6 + 4 + 3 = 40.

Problem 15. If the grey octagon and the striped pentagon are regular, and the striped quadrilateral is a square,
determine the measure of the angle between the thick segments.

Result. 99◦
Solution. Let us denote the points as in the picture.

B
A
C

Angle CBD is the difference of interior angles of octagon and pentagon, hence ∠CBD = 135◦ − 108◦ = 27◦ . We also
easily obtain that ∠ABD = 135◦ . Since both triangles ABD and CBD are isosceles,

∠CDB = 12 (180◦ − ∠CBD) = 76.5◦ ,


∠BDA = 12 (180◦ − ∠ABD) = 22.5◦ .

Therefore
∠CDA = ∠CDB + ∠BDA = 99◦ .

Problem 16. The minister has a personal driver who leaves the ministry at fixed time in the morning to pick up the
minister at his place and take him to the ministry. The minister wakes up at the same time every day and the car
comes exactly when he is ready to go. Today, the minister woke up early and he was ready to leave one hour earlier
than usual, so he decided to walk towards the car (which departed from the ministry as usual). He met the car, got in
and arrived at the ministry twenty minutes earlier than usual. How many minutes did he spend walking? Assume that
the car moves always at the same speed and that it takes no time for the minister to get in the car.
Result. 50
Solution. The time the minister gained by getting up earlier (1 hour) splits into the unknown length t of the walk and
the time which would remain for the car to get from the meeting point to the minister’s house, which is obviously half
of the total saved time, therefore
20
60 = t +
2
or t = 50.

5
Problem 17. What is the smallest positive integer, which has at least two digits and when its first (i.e. leftmost)
digit is erased, the value drops 29 times?
Result. 725
Solution. Let d be the first digit of the number, k the number obtained after erasing the first digit, and n the number
of digits of k. Then the original number equals 10n d + k and the assertion can be rewritten as

10n d + k = 29k

or
28k = 10n d.
Since 28 = 22 · 7, in order for the right-hand side to be divisible by 28, d = 7 and n ≥ 2 has to hold. Finally, the choice
n = 2 (implying k = 25) gives the smallest possible number 725.

Problem 18. How often in 24 hours is the minute hand of a clock perpendicular to its hour hand?
Result. 44
Solution. The minute hand does 24 revolutions in 24 hours, the hour hand does 2 revolutions in 24 hours. Therefore
the minute hand laps the hour hand 22 times in 24 hours. In each of this 22 times the minute hand and the hour hand
are perpendicular 2 times, therefore the answer is 44.

Problem 19. Find all four-digit palindrome numbers that can be written as a sum of two three-digit palindromes.
Note: A palindrome is a number which stays the same when the order of its digits is reversed, e.g. 2018102 is a
palindrome. A number cannot begin with a zero.
Result. 1111, 1221
Solution. Let abba be such a palindrome. Since it is a sum of two three-digit numbers, it does not exceed 1998, so
a = 1. Let 1bb1 be equal to cdc + xyx, then

1001 + 110b = 101(c + x) + 10(d + y).

Since the left-hand side ends with 1, c + x also ends with one. As both c and x are at least one and at most 9, c + x = 11.
Plugging in and simplifying produces
11(b − 1) = d + y.
Since d and y are digits, the right-hand side does not exceed 18, so b − 1 is either 0 or 1. Both options are possible:
1111 = 505 + 606, 1221 = 565 + 656.

Problem 20. The sides of an equilateral triangle are divided into two segments that are in the ratio of 6 to 1 in such
a way that the dividing points also form an equilateral triangle (see figure). Determine the ratio of the area of the
smaller equilateral triangle to the area of the larger equilateral triangle.

Result. 31/49
Solution. The area of each of the three small triangles is
1 6 6
· =
7 7 49
of the large equilateral triangle, because the height is 1/7 and the base 6/7 of the corresponding lines in the large
equilateral triangle. Therefore the ratio of the area of the smaller equilateral triangle to the area of the larger equilateral
triangle is
6 31
1−3· = .
49 49

6
Problem 21. Find all quadruples (a, b, c, d) of positive integers such that when we replace the letters in the table
below by the assigned values, then a, b, c, d will be exactly the number of ones, twos, threes, and fours present in the
table, respectively.

1 2 3 4

a b c d

Result. (2, 3, 2, 1), (3, 1, 3, 1)


Solution. No number can appear in the table more than five times; however, the number five cannot appear, since it
would take a position used by the number appearing five times. Hence only the numbers 1, 2, 3, and 4 can be filled in.
Let us show that d = 1: If d = 2, then one of a, b, c has to be 4, and because there are only two free slots, it
has to be b. However, (2, 4, 2, 2) is clearly not a valid quadruple. The options d = 3 and d = 4 lead even faster to a
contradiction.
We now know that a ∈ {2, 3}. Assuming that a = 2, we get b, c ∈ {2, 3} (there cannot be any other ones and fours),
but b = 2 would contradict the condition on b (there would be three twos), hence b = 3, and c = 2 follows directly. If
a = 3, there has to be one more one in the table, and that cannot be c (there is already another three), so b = 1, and
again c = 3 is immediate.

Problem 22. Peter forgot his password. He only remembers that the password consisted of nine lower-case latin
letters and contained words ‘math’ and ‘drama’. How many passwords do satisfy these requirements?
Note: The words are contained as substrings, therefore, for example, ‘martha’ does not contain ‘math’. There are
altogether 26 letters in the alphabet.
Result. 2030
Solution. Firstly, consider the case when the subwords ‘math’ and ‘drama’ do not overlap. Then there are two ways
to order them: ‘dramamath’ and ‘mathdrama’.
If they do overlap, there is only one possible way to arrange them: ‘dramath’. There are three ways to choose
places for the two remaining letters: ‘∗∗dramath’, ‘∗dramath∗’, ‘dramath∗∗’. In every case, there are 262 = 676 ways
to choose these two letters. Hence, in these cases, there are 676 · 3 = 2028 possible passwords.
In total, there are 2028 + 2 = 2030 possible passwords.

Problem 23. If one chooses two arbitrary distinct numbers from the set {1, 2, 3, . . . , n − 1, n}, the probability that
1
the numbers are consecutive positive integers is 21 . Determine n.
Result. 42
1
Solution. There are n − 1 pairs of consecutive numbers in the set {1, 2, 3, . . . , n − 1, n} and there are 2 n(n − 1)
possibilities to choose two arbitrary different numbers. Therefore we have
n−1 2 1
1 = =
− 1)
2 n(n
n 21

yielding n = 42.

Problem 24. Arthur, Ben and Charlie were playing table tennis using the following rules: Each round, two players
played against each other and the remaining one rested. The winner of the round then played in the next round with
the rested player. In the first round, Arthur played against Ben. After several rounds, Arthur had scored 17 victories
and Ben 22. How many times did Arthur and Ben play against each other?
Result. 20
Solution. Observe that whenever Charlie wins a round, it has no impact on the number of rounds won by Arthur
or Ben, neither has it any impact on the number of rounds when Charlie does not play. Hence we may assume that
Charlie always loses. In other words, every victory of Arthur over Ben increases Arthur’s overall score by two (unless it
happened in the last round), and vice versa. Since the number of Arthur’s victories is odd, we see that the last round
had to be Arthur vs. Ben, won by Arthur. Therefore, if we add one more round (Arthur vs. Charlie won by Arthur),
the total number of rounds when Charlie did not play would be one half of the sum of final scores of Arthur and Ben,
i.e. (18 + 22)/2 = 20.

7
Problem 25. E-shop customers can express their satisfaction with a purchased item by rating it online using a
five-point rating scale (1 star = poor, 5 stars = excellent). The average rating of a newly released smartphone was 3.46
stars last week, however, as two more people submitted their ratings earlier this week, it rose to the current average of
3.5 stars. How many people have rated the smartphone so far?
Result. 52
Solution. Denote k the original number of ratings and x their sum. Further denote a, b the two this week’s ratings.
Then
x x+a+b
= 3.46 and = 3.5
k k+2
or
23

x= 3+ 50 k, (1)
1

x+a+b= 3+ 2 k+ 7. (2)

Equation (1) implies that k is a multiple of 50. Moreover, after subtracting (1) from (2), we get

k
a+b−7= .
25
As a, b ≤ 5, the left-hand side is a positive integer not exceeding 3, hence k ≤ 75. We conclude that k = 50 and after
adding the two customers rating this week, we see that 52 people have rated so far.

Problem 26. Juliette has four pairs of socks with Monday, Tuesday, Wednesday, Thursday written on each single
sock. How many ways are there to wear all of these socks from Monday to Thursday, if the two socks on Juliette’s feet
should be different and neither of them showing the current day? None of the socks can be worn repeatedly.
Note: Any sock can be worn on any foot, i.e. there are no “right” and “left” socks. Furthermore, wearing a sock on the
right foot and another sock on the left foot counts the same as wearing them reversed.
Result. 9
Solution. For the sake of brevity, we will use numbers 1, 2, 3, 4 instead of the names of the days. Observe that each
day is assigned three distinct numbers: The actual number of the day and the two numbers of the socks being worn.
Therefore, we may equivalently describe the assignment of socks by a single number for each day—the only number out
of the four not appearing in the aforementioned triple. We infer that the valid assignments of socks correspond to the
rearrangements of (1, 2, 3, 4), i.e. permutations which leave none of the numbers at its original position.
The number of rearrangements can be computed as follows: There are three options of placing 1, let n 6= 1 be its
position. Now n has also three options where to be put. It is easy to see that the remaining two numbers are now
assigned in an unique way, hence there are 3 · 3 = 9 rearrangements and the same number of choices of Juliette’s socks.

Problem 27. A jury of 26 mathematicians was to nominate (at least) five films for awards at a festival of math-themed
films. There were 16 films to choose from. The jury had chosen the following procedure: Each jury member voted for
five distinct films and the five films with most votes were nominated; if there was a tie on the fifth place, all these films
were nominated. What is the smallest number of votes that a film could have received so that it was nominated no
matter the results of other films?
Result. 21
Solution. In total, 26 · 5 = 130 votes were distributed among the films. On one hand, if a film received 20 or less votes,
the remaining 110 votes can easily be distributed so that there are five films getting 21 votes each. On the other hand,
if the film received at least 21 votes, it not being nominated would imply at least five films getting at least 22 votes,
resulting in at least 21 + 5 · 22 = 131 votes cast, a contradiction.

Problem 28. A real function f satisfies f (x) + xf (1 − x) = x for every real value of x. Find f (−2).
Result. 4/7
Solution. From the equation f (−2) − 2f (3) = −2 we see that it is equivalent to determine f (3) instead. Since
f (3) + 3f (−2) = 3, we have two linear equations for the unknown values f (−2) and f (3). Multiplying the second
equation by 2 and adding both equations we get f (−2) = 4/7.

8
Problem 29. Two-digit numbers n, a, b, o, j are such that their product naboj is divisible by 4420. Determine the
greatest possible value of their sum n + a + b + o + j.
Result. 471
Solution. Firstly, let us factorize 4420 = 2 · 2 · 5 · 13 · 17. Since 13 and 17 are primes, one of the numbers n, a, b, o, j
has to be divisible by 13 and one by 17. As the smallest common multiple of 13 and 17 is 221, there is no two-digit
number divisible by both of them. Without loss on generality, we can assume that n is divisible by 17 and a is divisible
by 13. This means that n ≤ 85 = 5 · 17 and a ≤ 91 = 7 · 13.
Suppose now that n = 85 and a = 91. We see that n = 85 is divisible by 5, so we only need to guarantee the
divisibility by 4. As n and a are odd, 4 has to divide boj. Therefore, one of the numbers b, o, j is divisible by 4, or
there are two numbers divisible by 2. We get the greater sum in the second case, when b = o = 98 and j = 99. We
have found the sum n + a + b + o + j = 85 + 91 + 98 + 98 + 99 = 471.
Finally, let us check the possibility that n < 85 or a < 91. Since the numbers n and a have to be divisible by 17
and 13, respectively, this would mean n ≤ 68 = 85 − 17 or a ≤ 78 = 91 − 13. Then the sum n + a + b + o + j could be
at most 68 + 91 + 3 · 99 = 456 (in the former case) or 85 + 78 + 3 · 99 = 460 (in the latter case) and that is less than we
have previously achieved.

Problem 30. Naomi ordered eight tennis balls and one handball at an online sports shop. The balls (of a perfect
spherical shape) were packed in a cubic box so that each tennis ball was tangent to three of the six faces of the box
and to the handball. The radius of a handball is 10 cm and the radius of each tennis ball is 5 cm. Find the length of
the edge of the box in centimeters.

Result. 10(1 + 3)
Solution. The space diagonal of the box passes through the centers of the handball and two tennis balls, and also
through the points of tangency of these three balls. The only area where the diagonal is not inside any ball are the
segments between a tennis ball and corner of the box; the distance from the center of a tennis ball to the corner is one
half of the space diagonal of the circumscribed cube of the ball. So the length of the diagonal of the box is the sum of
• (2×) 1/2 of the diagonal of a cube circumscribed to a tennis ball,
• (2×) the radius of a tennis ball,

• the diameter of the handball.


Hence the length of the diagonal is equal to
√ √
10 3 + 10 + 20 = 30 + 10 3,

and the length of the edge is equal to √


30 + 10 3 √
√ = 10(1 + 3).
3

Problem 31. Written in the decimal system, the power 229 is a nine-digit number whose digits are pairwise distinct.
Which digit is missing?
Result. 4
Solution. On one hand, the power 229 can be computed by hand with reasonable effort: For example, use 210 = 1024,
compute 10242 and 10242 · 1024. Finally, divide the result by 2 to get 229 = 536 870 912.
On the other hand, you can use the fact that an integer and its digital sum have the same residue class modulo 9.
Moreover, the residue class of 2n modulo 9 is periodic with period of length 6. Since the sum of all digits is 45, we end
up having
45 − x ≡ 229 ≡ 25 ≡ 5 (mod 9)
where x denotes the missing digit in the decimal representation of 229 . This leads to x ≡ 4 (mod 9). Therefore the
missing digit is 4.

9
Problem 32. When cleaning the attic, Ben found an old calculator, which showed only √ the first two digits after the
decimal √
point for each result, but was able to compute square roots. So for example, for 4 the machine displayed 2.00
and for 6 = 2.44949 . . . it showed 2.44. What is the smallest positive integer, which is not a square of an integer, but
for whose square root would Ben’s calculator show two zeros after the decimal point?
Result. 2501

Solution. Denote by ftd(n) the first two digits after the decimal point of n. It is clear that as n increases from one
square number to the next one, ftd(n) increases as well; since we are looking for the smallest integer n, this has to be
of the form√k 2 + 1 for some positive integer k. √
When k 2 + 1 is rounded down to its integer part, the result is k, therefore k 2 + 1 − k is a number strictly
between 0 and 1. The assertion that ftd(k 2 + 1) = 0 can thus be equivalently restated as
p 1
k2 + 1 − k < .
100
Adding k to both sides, squaring (both sides are positive) and rearranging yields
 
1
k > 50 1 − .
1002

The right-hand side is a number strictly between 49 and 50; since k is an integer, k ≥ 50 follows. We conclude that
n = 502 + 1 = 2501 is the sought smallest number.

Problem 33. One of the numbers from 1 to 9 is to be written in each cell of a 2018 × 2018 board in such a way that
within any 3 × 3 square, the sum of the inputs is divisible by 9. In how many different ways can this be accomplished?
Result. 98068
Solution. Any filling of the 8068 cells forming the two bottom rows and the two leftmost columns defines the whole
filling explicitly as we can uniquely fill in the remaining numbers on consecutive diagonals—see the example in the
picture. On the other hand, obviously, each correct filling induces some filling of the 8068 cells. Therefore the number
of arbitrary fillings of those cells is the same as the sought number of correct fillings of the table.

9 8 9 8 9 8 9 8
9 7 9 7 9 7 9 7 2
5 2 5 2 5 2 5 5 2 5 1
→ → →
4 1 4 1 1 4 1 1 2 4 1 1 2 2
3 6 9 9 1 3 3 6 9 9 1 3 3 6 9 9 1 3 3 6 9 9 1 3
2 9 1 7 4 4 2 9 1 7 4 4 2 9 1 7 4 4 2 9 1 7 4 4

Problem 34. Find all pairs of positive integers (n, m) fulfilling the equation 4n + 260 = m2 .
Result. (3, 18), (6, 66)
Solution. The given equation is equivalent to m2 − (2n )2 = 260 and factorizing the left-hand side leads to (m −
2n )(m + 2n ) = 260. The prime factorization of 260 = 22 · 5 · 13 leads to the following possible decompositions:

260 = 1 · 260 = 2 · 130 = 4 · 65 = 5 · 52 = 10 · 26 = 13 · 20,

taking into account that (m − 2n ) < (m + 2n ). Due to (m + 2n ) − (m − 2n ) = 2n+1 we get the two possibilities
26 − 10 = 24 and 130 − 2 = 27 leading to the two pairs (3, 18) and (6, 66) fulfilling the the given equation.

Problem 35. In an equilateral triangle ABC, a light ray coming from B hit AC at point D satisfying DC : AC =
1 : 2018, and it reflected such that the angle of incidence was equal to the angle of reflection. Then it reflected again
every time it reached a side of 4ABC. How many times did it reflect (including the first reflection) until it reached
some vertex of 4ABC?
Result. 4033
Solution. Instead of reflecting the light ray, we shall let it proceed straight and subsequently reflect the triangle along
the side with which the ray is incident. Let us show that one row of these reflected triangles is sufficient for the light
ray to reach a vertex of one of the triangles.
Let E be the intersection of the ray BD with line through A parallel to BC, and F a point on BC such that
EF k AC. Then the triangles BCD and BF E are similar and BF = 2018BC. This implies that the point E may be
obtained by reflecting the triangle ABC and it is clearly first such point on BD.

10
It is easy to see that the segment BE intersects 2 · 2017 − 1 = 4033 segments in the row of triangles, which equals
the number of reflections of the light ray in the original problem. The picture illustrates the solution with the initial
condition changed to DC : AC = 1 : 5.

A B0 C0 A00 B 00 = E

B C F

Problem 36. A rectangular sheet of paper ABCD was folded so that (former) point A ended on side BC and point
M , where side CD met (former) side DA, was exactly in one third of CD, i.e. CD = 3CM . If the area of the gray
overlapping triangle is 1, what is the area of the striped triangle?

D M C

A B

Result. 9/4
Solution. Let us denote the various intersection points as in the figure.

D0

D K M C

A0

A L B

All the three triangles KM D0 , A0 M C, and LA0 B are right and similar to each other, as straightforward angle chasing
shows, hence we seek the ratio of similitude between the two triangles in question. Taking into account that KD = KD0
and LA = LA0 , we have
D0 K + KM = DM = 32 DC = 23 AB
and
A0 L + LB = AB.
Since A0 L corresponds to KM and LB corresponds to KD0 in the aforementioned similarity, this shows that the ratio
is 3/2. As we are interested in areas, the result is (3/2)2 = 9/4.

Problem 37. When dividing the polynomial x3 + x5 + x7 + x9 + x11 + x2017 + x2018 by the polynomial x2 − 1 there
is a remainder. Find the value of the variable x for which the numerical value of that remainder is 1111.
Result. 185

11
Solution. The polynomial can be rewritten as

x3 + x5 + x7 + x9 + x11 + x2017 + x2018 =


= x(x2 − 1) + x(x4 − 1) + x(x6 − 1) + x(x8 − 1) + x(x10 − 1) + x(x2016 − 1) + (x2018 − 1) + 6x + 1.

Now, since
x2k − 1 = (x2 − 1)(x2k−2 + x2k−4 + · · · + 1),
we see that all the brackets on the right-hand side of the equality are divisible by x2 − 1. As the degree of 6x + 1 is
smaller than the degree of x2 − 1, we infer that 6x + 1 is the sought remainder. Now solving 1111 = 6x + 1 produces
the answer x = 185.

Problem 38. There are ten towns in the country of Pentagonia, each one connected by three railway lines to another
three towns according to the diagram below. The antitrust laws of the country require that no two lines having a
common stop are operated by the same railway company. In how many ways can the lines be assigned to three railway
companies in a lawful way?

Result. 30
Solution. Observe that when assigning the lines on the “outer pentagon”, two of the companies (denoted by X and
Y ) have to get two lines and one company (Z) obtains one line. The companies also have to be in order XY XY Z
starting from some town. It is easy to show that when we have assigned these lines, the rest of the network is assigned
in a unique way: The lines of the “inner pentagon” are assigned to the same companies as their outer counterparts and
the “connecting” lines must each go to the sole unused company.
This means that the number of correct assignments of the whole network is equal to the number of assignments of
the outer pentagon. There are six ways to assign the three companies to X, Y , and Z, and five possibilities for the
town where the assignment XY XY Z starts from. This gives us 5 · 6 = 30 ways in total.

Problem 39. A right triangle contains 25 tangent circles of radius 1 as shown in the picture.

What is the radius of the incircle of the triangle?



Result. 25 − 13 2
Solution. Consider the triangle whose vertices are centers of the three circles in the corner. This is a right triangle
with legs of length 14 and 34, respectively; hence, by the Pythagorean theorem, the length of the hypotenuse is
p √
142 + 342 = 26 2.
√ √
Further, its inradius can be computed as (14 + 34 − 26 2)/2 = 24 − 13 2. Since the sides of the original triangle are
parallel to the new ones and their distance is 1, the incenters of the two triangles coincide and the sought inradius is

12

just the one we have computed plus the distance, i.e. 25 − 13 2.

Problem 40. Marge has invented the operation marging on a (finite) list of integers: given a list, she takes four
copies of it, increases their entries by 0, 2, 3, and 5, respectively, and concatenates the results, forming again a single
list. For example, given the list (8, 3), the result after marging is (8, 3, 10, 5, 11, 6, 13, 8). If Marge starts with the
one-element list (0) and keeps marging it until it has at least 2018 entries, what will be the 2018th entry? (The leftmost
entry is considered to be the first one.)
Result. 17
Solution. For convenience, let us number the positions in the list starting with zero instead. In that case, an easy
induction argument shows that the position of a number written in the base 4 system describes what operations (and
in what order) were applied to obtain the number on the given position. For example, this is what Marge’s list looks
like after two iterations of marging:

(0+0, 0+2, 0+3, 0+5, 2+0, 2+2, 2+3, 2+5, 3+0, 3+2, 3+3, 3+5, 5+0, 5+2, 5+3, 5+5).
00 01 02 03 10 11 12 13 20 21 22 23 30 31 32 33

(The numbers under the entries are their positions in base 4.) Since 2017 written in base 4 is 133201, the number on
position 2017 is
2 + 5 + 5 + 3 + 0 + 2 = 17.

Problem 41. Determine the smallest positive integer n such that the equation

(x2 + y 2 )2 + 2nx(x2 + y 2 ) = n2 y 2

has a solution (x, y) in positive integers.


Result. 25
Solution. The equation can be also viewed as a quadratic equation in n with solution
p
(x2 + y 2 ) x + x2 + y 2

n=
y2
p
(the other solution would lead to negative n since x2 + y 2 > x), or
p
ny 2 = (x2 + y 2 ) x + x2 + y 2 .


Let d = GCD(x, y) and let x = x0 d, y = y0 d. Plugging in and simplifying yields


 q 
ny02 = d(x20 + y02 ) x0 + x20 + y02 .

2 2 2 2 2
p since x0 and y0 are coprime, also y0 and x0 + y0 are coprime and x0 + y0 | n follows. Further, the presence of
Now
2 2 2 2 2
x0 + y0 forces x0 + y0 to be a square. It is well known that 5 = 25 is the smallest square which is the sum of two
squares, 32 + 42 . Hence n ≥ 25 and plugging in x = 4, y = 3 shows that n = 25 indeed gives a solution.

13
Problem 42. In a rectangular room with dimensions 6 m × 2.4 m × 2.4 m (length × width × height), a spider is
located on one 2.4 m × 2.4 m wall 20 cm away from the ceiling and with equal distance to the vertical edges. A fly,
sitting on the opposite wall, is on its vertical axis of symmetry, too, but 20 cm away from the floor. If the fly does not
move at all, what is the shortest total distance (in meters) the spider must crawl along the surfaces in order to capture
the fly?

20 cm
spider
fly 2.4 m

20 cm
2.4 m
6m

Result. 8
Solution. Let us examine the spider’s possible paths in the net of the cuboid. It is clear that the shortest path becomes
a straight line segment when the net is put into a plane. The white circle represents the fly, whereas the black circle
stands for the spider, its location in the plane dependent on the way we decompose the net of the cuboid.

B
C

There are (up to a symmetry) three possible ways for the spider to reach the fly, crossing one, two, or three of the long
faces of the cuboid; the paths are marked by A, B, and C in the picture. Clearly, a path using four of these faces could
be reduced to a shorter one. √ The length of path A is 8.4 m and using the Pythagorean theorem, we obtain that the
length of path B in meters is 66.32 and for path C it is 8. Therefore path C is the shortest one and the answer is 8 m.
The image below shows the shortest path in three dimensions:

Problem 43. Find the minimum of

(6 + 2 cos(x) − cos(y))2 + (8 + 2 sin(x) − sin(y))2

for x, y ∈ R.
Result. 49
Solution. Let us set V (x, y) = (6 + 2 cos(x) − cos(y))2 + (8 + 2 sin(x) − sin(y))2 . Recall that the circle with center
(C1 , C2 ) and radius R > 0 can be parametrized (i.e. coordinates of all points lying on it can be expressed) by angle
α as (x1 , x2 ) = (C1 + R cos(α), C2 + R sin(α)). Let us consider circles k1 , k2 with centers (0, 0), (6, 8) and radii 1, 2,
respectively. Then it follows from the Pythagorean theorem that V (x, y) = AB 2 where A ∈ k1 with angle x and B ∈ k2
with angle y. It follows that the minimum of V (x, y) is the square of the distance of the √ closest pair of points on k1
and k2 and we can compute it using the distance of the centers and the radii of k1 and k2 : 62 + 82 − 1 − 2 = 7. Thus
the minimum of V (x, y) equals 72 = 49.

14
Problem 44. What is the smallest positive integer such that its last (i.e. units) digit is 2, and if we move the last
digit in front of the first digit, we get the double of the original number?
Result. 105 263 157 894 736 842
Solution. Let us call N the number in question. When we remove the units digit, we should get all the digits of 2N
except for the leftmost one. Therefore, since N ends with 2, 2N has to end with 4, hence the tens digit of N is 4. Let
di be the i-th digit of N , this time counting from right to left (i.e. d1 is the units digit). Taking into account how
multiplying by two works digit-wise, we see that the digits of N have to satisfy
(
2di−1 mod 10 if di−2 < 5,
di =
2di−1 mod 10 + 1 if di−2 ≥ 5

for all i > 2. This way we can directly write down the digits of N . We stop when we obtain the digit 1 and in the next
step the digit 2: The number starting with this 1 is N , because multiplying by 2 precisely removes the last digit, but
puts 2 in front of the number. The result is

N = 105 263 157 894 736 842.

Problem 45. Mother Berta divides her triangular shaped area of land with two straight lines into four pieces and
gives the piece of size 6 to her daughter Betty, the one of size 4 to her daughter Barbara and the smallest one with size
3 to the youngest daughter Francis. She keeps the largest piece for herself. How big is this piece of land?

3
6

Result. 19/2
Solution. We use the notation as in the following picture.
C

Q
3
S 6
b

a 4
A B
P

Due to the area ratios, S divides QB in the ratio 1 : 2 and P C in the ratio 2 : 3. Introducing the straight line AS and
denoting the area of triangle ASQ by b and the area of triangle AP S by a leads to the following two equations:
b 1
=
a+4 2
b+3 3
=
a 2
These are equivalent to

2b = a + 4
2b + 6 = 3a,

which gives the solutions a = 5 and b = 29 . Therefore the area of quadrangle AP SQ is 19


2 .

15
Problem 46. Four brothers have altogether 2018 euros. It is known that the wealth of each of them is a positive
integer, no two possess the same amount of euros, and whenever one brother is richer than another one, the wealth of
the richer one is a multiple of the wealth of the poorer one. What is the smallest number of euros the richest brother
could have had?
Result. 1152
Solution. Since each brother’s wealth is a multiple of the wealth of the poorest one, their sum, 2018, has to be divisible
by this number as well. However, prime factorization 2018 = 2 · 1009 gives only three options for the poorest one: 1, 2,
or 1009. Clearly, 1009 is impossible, as this would be larger than any of the remaining three amounts. Further, if it
was just 1, the rest would be left with 2017 euros, which is a prime number, hence the second poorest brother would
have had only 1 as well—a contradiction. Therefore the poorest one has 2 euros and the remaining three have 2016
altogether.
Let a < b < c be the fortunes of the three brothers; these are subject to the conditions a | b | c and a + b + c = 2016.
The divisibility together with strict inequality implies that 2a ≤ b and 2b ≤ c; if we could achieve equalities, we would
clearly get a solution with the smallest value of c. Luckily, 1 + 2 + 4 = 7 divides 2016, therefore we can indeed divide
this sum as
2016 = 17 · 2016 + 72 · 2016 + 47 · 2016
4
and the answer is 7 · 2016 = 1152.

Problem 47. Andrew drew the symbol ♣ on the blackboard. Then he repeated the following procedure thirteen
times: He erased the blackboard and wrote a new sequence of symbols, having the pair ♣♥ instead of each ♥ and ♥♣
instead of each ♣ in the original sequence. For example, Andrew would replace the sequence ♣♥♥ with ♥♣♣♥♣♥.
How many pairs ♥♥ (with no other symbol in between) were there on the blackboard when Andrew was finished with
his task? The counted pairs may overlap, so e.g. in the sequence ♥♥♥♥, there are three ♥♥ pairs.
Result. 1365
Solution. Let An be the sequence on the blackboard after Andrew carried out the replacement procedure for the n-th
time (with A0 = (♣)) and hn the number of pairs ♥♥ in An . Since each pair ♥♥ in An arises only from the pair ♥♣
in An−1 , which, on the other hand, comes either from ♥♥ or ♣ in An−2 , we see that hn = hn−2 + 2n−3 for n ≥ 3, since
there are exactly 2n−3 symbols ♣ in An−2 . Therefore, for odd n, we have

hn = 2n−3 + 2n−5 + · · · + 20 + h1 = 31 (2n−1 − 1),

since h1 = 0. The desired result is h13 = 1365.

Problem 48. Let ABCDEF GHI be a regular nonagon with circumcircle % and center O. Let M be the midpoint
of the (shorter) arc AB of %, P the midpoint of M O, and N the midpoint of BC. Let lines OC and P N intersect at
Q. What is the measure of ∠N QC (in degrees)?
Result. 10◦
Solution. We will prove that the quadrilateral OCN P is cyclic; since ∠ON C = 90◦ , this is equivalent to ∠OP C = 90◦ .
This can be seen as follows: As both C and M lie on %, OC = OM . Easy computation also shows that ∠M OC = 60◦ ,
so 4OCM is equilateral. Now P , being the midpoint of OM , satisfies ∠OP C = 90◦ .
Another easy calculation reveals that ∠OCN = 70◦ , hence ∠OP N = 180◦ − ∠OCN = 110◦ . Using the triangle
OQP , we get that
∠N QC = ∠P QO = 180◦ − ∠P OQ − ∠QP O = 10◦ .
A
M
% B
P
N

O Q
C

16
Problem 49. Anna picked a triple (x, y, z) of positive integers such that x + y + z = 2018 and told x to Xena, y to
Yena, and z to Zena. None of the three knew the other two numbers, but they were told the information about their
sum. The following conversation followed:
• Xena: I know that Yena and Zena have different numbers.
• Yena: Thanks to Xena, now I know that all the three of us have different numbers!
• Zena: Now I can finally tell who was told what number.
Find the triple (x, y, z).
Result. (3, 2, 2013)
Solution. Xena’s statement means just that x is odd; if it was even, y and z could have been the same.
Assume now that y is odd; that means that Yena knew from the beginning that x and z are different. If, moreover,
y ≥ 1009, Yena would have already known that x and z were different from y and thus would not need Xena’s statement.
On the other hand, if y ≤ 1007, then despite Xena’s statement Yena still could not tell if her number was different
from x. We infer that y is even and consequently, z is odd.
If y was a multiple of 4, then x + z = 2018 − y ≡ 2 (mod 4), i.e. it could be a double of an odd number; in such a
case Yena could not deduce that x and z are different. However, if y ≡ 2 (mod 4), then x and z have to give distinct
remainders modulo 4 and Yena’s statement is justified.
Finally, let us examine Zena’s statement. Firstly, y = 2, for otherwise y could be decreased by 4 and x increased
by 4 and Zena could not tell the difference. For similar reasons, x ≤ 4, hence either x = 1 or x = 3. However, in the
former case, knowing 2018 − z = x + y = 3 Zena could have determined x and y without Yena’s statement (using just
what Xena had reported). We conclude that x = 3 and z = 2013.

Problem 50. Wizards Arithmetix and Combinatorica are engaged in a duel. Both wizards have 100 hit points (HP).
Arithmetix’s spell hits Combinatorica with probability 90% and deals 60 HP damage (if it succeeds), Combinatorica’s
spell hits Arithmetix with probability 60% and deals 130 HP damage. The wizards alternate in spell-casting, Arithmetix
starts. The duel ends when a participant runs out of his or her hit points, the remaining wizard being the winner.
Determine the probability that Arithmetix wins the duel.
Result. 45/128
Solution. The exact amount of HP is not important—it clearly suffices to know that Arithmetix loses after being hit
once and Combinatorica after being hit twice. Suppose that we are in the state of the duel when Combinatorica has
been already hit once and it is Arithmetix’s turn to cast. Denote the probability that Arithmetix wins by q. In this
state, Arithmetix can win either if his attack succeeds, which happens with probability 0.9, or if he misses and so does
Combinatorica in her turn—that happens with probability 0.1 · 0.4, and subsequently, Arithmetix wins again with
probability q. Therefore, we obtain the equation

q = 0.9 + 0.1 · 0.4 · q,

which yields q = 15/16.


Let us now compute the probability p that Arithmetix wins the duel. If Arithmetix hits and Combinatorica misses
(probability 0.9 · 0.4), the duel gets to the situation of the previous paragraph and Arithmetix wins with probability
q = 15/16. On the other hand, if Arithmetix misses and Combinatorica misses, too (probability 0.1 · 0.4), then
Arithmetix can again win with probability p. So we can write the equation
15
p = 0.9 · 0.4 · + 0.1 · 0.4 · p.
16
Solving it shows that Arithmetix wins the battle with probability p = 45/128.

Problem 51. Let a(1), a(2), . . . , a(n), . . . be an increasing sequence of positive integers satisfying a(a(n)) = 3n for
every positive integer n. Compute a(2018).
Note: A sequence is increasing if a(m) < a(n) whenever m < n.
Result. 3867
Solution. If a(1) = 1 we also have a(a(1)) = 1 6= 3 · 1 which is impossible. Since the sequence is increasing it follows
that 1 < a(1) < a(a(1)) = 3 and thus a(1) = 2. From the equation we deduce a(3n) = a(a(a(n))) = 3a(n) for all n.
We easily prove by induction (starting with a(1) = 2) that a(3m ) = 2 · 3m for every m. Using this we also obtain
a(2 · 3m ) = a(a(3m )) = 3m+1 .
There are 3n − 1 integers i such that 3n < i < 2 · 3n and there are 3n − 1 integers j such that a(3n ) = 2 · 3n < j <
n+1
3 = a(2 · 3n ). Since a(n) is increasing there is no other option than a(3n + b) = 2 · 3n + b for all 0 < b < 3n . Therefore
a(2 · 3n + b) = a(a(3n + b)) = 3n+1 + 3b for all 0 < b < 3n . Since 2018 = 2 · 36 + 560 we have a(2018) = 37 + 3 · 560 = 3867.

17
Problem 52. Equilateral triangle T of side length 2018 is divided into 20182 small equilateral triangles of side length
1. We call a set M of vertices of these small triangles independent if for any two distinct points A, B ∈ M the segment
AB is not parallel to any side of T . What is the largest number of elements of an independent set?
Result. 1346
Solution. Each vertex in the grid can be assigned its distances to the three sides of T (taking as the unit the height
of a small triangle); it is easy to see that for each vertex, these three integers add up to 2018. On the other hand,
given a triple of non-negative integers with sum 2018, there is a unique vertex of the grid with these numbers being its
distances from the sides, thus we may equivalently consider such triples instead of the vertices. We will refer to the
three numbers as coordinates.
The independence condition translates to the assertion that no two triples in the set have equal the first, the second,
or the third coordinate. Let
M = {(x1 , y1 , z1 ), (x2 , y2 , z2 ), . . . , (xk , yk , zk )}
be an independent set. Since the numbers x1 , . . . , xk are distinct non-negative integers, their sum is at least

k(k − 1)
0 + 1 + · · · + (k − 1) = .
2
The same holds for the sums y1 + · · · + yk and z1 + · · · + zk . On the other hand, we have xi + yi + zi = 2018 for each
i = 1, . . . , k, and so

k(k − 1)
3· ≤ (x1 + · · · + xk ) + (y1 + · · · + yk ) + (z1 + · · · + zk ) = 2018k.
2
It follows that
2
k ≤1+ · 2018
3
or k ≤ 1346.
The following two sequences of points form together an independent set of size 1346:

(0, 672, 1346), (2, 671, 1345), (4, 670, 1344), . . . , (1344, 0, 674);
(1, 1345, 672), (3, 1344, 671), (5, 1343, 670), . . . , (1345, 673, 0).

We conclude that the sought maximal number of elements is 1346.


The following picture illustrates the construction of an independent set for a triangle of side length 11 instead of
2018:

Problem 53. Let ABC be a triangle with AB = 5, AC = 6 and ω its circumcircle. Let F , G be points on AC such
that AF = 1, F G = 3, and GC = 2, and let BF and BG intersect ω in D and E, respectively. Given that AC and
DE are parallel, what is the length of BC?
p
Result. 5 5/2
Solution. Denote x = BC. Since ACED is an isosceles trapezoid, we may put y = AE = CD. Finally, let p = BF ,
q = DF , u = BG, and v = GE.
Angles BAC and BDC are inscribed in the same circle and hence of the same size. Consequently, triangles ABF
and DCF are similar, which implies
y q 5
= = .
5 1 p
Furthermore, in the same way we obtain the similarity between triangles BCG and AEG, from which
y v 4
= =
x 2 u
follows. Finally, as AC and DE are parallel,
p u
=
q v

18
and combining with the preceding equations yields
25 4x
y y
y = 2y
5 x
p
or x2 = 125/2. Therefore, x = 5 5/2.

Problem 54. We know that


222000 = |4569878 {z
. . . 229376} .
6623 digits

For how many positive integers n < 22000 is it also true that the first digit of 2n is 4?
Result. 2132
Solution. If the first digit of a k-digit number N is c, then c10k−1 ≤ N < (c + 1)10k−1 . This implies that
2c10k−1 ≤ 2N < (2c + 2)10k−1 , i.e. the first digit of 2N is at least the first digit of 2c and at most the first digit of
2c + 1. We apply this to the first digits of powers of two: Having a power of two with the first digit equal to 1, there
are these five possibilities for the first digits of the following powers of two: (1) 1, 2, 4, 8, 1; (2) 1, 2, 4, 9, 1; (3) 1, 2, 5, 1;
(4) 1, 3, 6, 1; (5) 1, 3, 7, 1.
Let k be a non-negative integer such that 2k begins with 1 and has d digits. Then there is a unique power of two
beginning with 1 and having d + 1 digits, and it is either 2k+3 (if we are in one of the situations (3), (4), (5) above)
or 2k+4 (given that the case (1) or (2) occurs). As 20 (having 1 digit) and 221998 (having 6623 digits) begin with
1, we can compute how many times does (1) or (2) occur when computing successive powers of two: It is exactly
21998 − 3 · 6622 = 2132 times.
Finally, observe that the cases (1) and (2) are precisely those giving rise to a power of two starting with 4, therefore
there are exactly 2132 such numbers in the given range.

Problem 55. Find rational numbers a, b, c such that


q
3 √
3 √ √
3 √
2 − 1 = 3 a + b + 3 c.

Note: A rational number is a quotient of two integers.


Result. (1/9, −2/9, 4/9)
p √
3 3

Solution. Let us set x = 2 − 1 and y = 3 2. The idea is to use the fact that the numbers y 3 ± 1 are integers and
employ the factorization identities for A3 ± B 3 to obtain a relation between x and y in a form suitable to express x as
sum of three rational cube roots. Firstly note that

1 = y 3 − 1 = (y − 1)(y 2 + y + 1)

and since 3 = y 3 + 1 we have

3y 2 + 3y + 3 y 3 + 3y 2 + 3y + 1 (y + 1)3
y2 + y + 1 = = = .
3 3 3
Hence
1 3
x3 = y − 1 = = .
y2 + y + 1 (y + 1)3
Secondly, since
3 = y 3 + 1 = (y + 1)(y 2 − y + 1)
we have
1 y2 − y + 1
=
y+1 3
and finally √
1 √ √
3
r
3 3 3 3
x= = ( 4 − 2 + 1).
y+1 9
We have proved that the triple (a, b, c) = ( 49 , − 29 , 19 ) works.
It is possible to prove that this representation of x as sum of three cube roots of rationals is unique up to a
reordering.

19
Problem 1. Three years ago the age of Punto’s mum was three times the age of Punto. Now the age of Punto’s dad
is three times the age of Punto. What is the difference of ages of Punto’s parents in years?
Result. 6
Solution. If x denotes the current age of Punto, then the current age of Punto’s mum is 3(x − 3) + 3, which is 3x − 6.
According to the second statement, Punto’s dad is currently 3x years old. Therefore, the difference of the parents’ ages
is 6 years.

Problem 2. How many triangles are there in the picture?

Result. 30
Solution. All the triangles in the picture have the topmost point as a vertex. Further, one side of all the triangles
lies completely on one of the two parallel horizontal line segments. Therefore, each triangle is determined by a choice
of one of the two horizontal segments and two distinct vertices on that segment. Since there are six vertices on each
segment, the number of triangles is
2 · (5 + 4 + 3 + 2 + 1) = 30.

Problem 3. A point E inside a square ABCD is such that ABE is an equilateral triangle. What is the size of the
angle DCE in degrees?
D C
?
E

A B

Result. 15
Solution. Since in an equilateral triangle each internal angle is 60◦ , we get ∠CBE = 90◦ − ∠EBA = 30◦ . Due to
EB = AB = BC, the triangle BCE is isosceles and we obtain

∠ECB = ∠BEC = 21 (180◦ − ∠CBE) = 75◦ .

Finally, the size of the angle sought after is ∠DCE = 90◦ − ∠ECB = 15◦ .

Problem 4. Little Sandi owns a big treasure chest containing a lot of labelled coins: one coin labelled with 1, two
coins labelled with 2, ..., eighteen coins labelled with 18, and nineteen coins labelled with 19. Sandi takes one coin after
the other out of her treasure chest without being able to read its label. What is the minimum number of coins Sandi
has to take out if she wants to make sure she has ten identically labelled coins?
Result. 136
Solution. By chance Sandi might have taken all coins with labels less than 10 and nine coins each of those with labels
from 10 to 19. In total, this amount of coins is (1 + 2 + · · · + 9) + 9 · 10 = 135. Therefore, taking out 135 coins does not
guarantee getting ten equally labelled ones.
However, if Sandi takes out 136 coins, at least 91 coins show labels greater than 9. By the pigeonhole principle, one
of the ten types of coins with labels 10 to 19 has been taken out at least ten times. Therefore, the minimum number of
coins is 136.

1
Problem 5. Mr. Sugar bought a big box of his favourite Halloween candies to hand out to trick-or-treaters. However,
he ate half of all the candies himself before the first child came and took her share. He ate half of what was left before
the second child came, and half of what was left again before the third child came and took all of the remaining candies.
If each child received exactly three candies, how many candies did Mr. Sugar originally buy?
Result. 42
Solution. If n is the number of candies bought originally, then we can rewrite the distribution of the candies via the
equation  
n  1 1
− 3 · − 3 · − 3 = 0.
2 2 2
Solving for n yields n = 42.

Problem 6. Let ABCD be a quadrilateral with right angles at A and C. Given the lengths BC = 6, CD = 8, and
DA = 2, find the area of the quadrilateral ABCD.
C
D

A B

Result. 24 + 4 6

Solution. By the Pythagorean theorem, we get BD = 62 + 82 = 10 and using this result we obtain
p p √ √
AB = BD2 − AD2 = 102 − 22 = 96 = 4 6.
Hence, the area of the quadrilateral ABCD is
1 √ √
(6 · 8 + 2 · 4 6) = 24 + 4 6.
2

Problem 7. An office printer can print either on one side or on both sides. One-sided printing takes three seconds
per page, whereas two-sided takes nine seconds per sheet. Kate wants to print a research paper eighteen pages long,
printing on both sides of the paper. She can either print all the pages two-sided or print only the odd pages, manually
put the sheets back into the printer, and print the even pages. She quickly realizes that both procedures would take
the same time. How many seconds does it take her to put the sheets back into the printer?
Result. 27
Solution. We know that Kate wants to print on nine sheets of paper. When using the duplex capability of the printer,
printing takes 9 · 9 = 81 seconds. On the other hand, when printing the sides separately, every sheet gets printed twice
so that the printing itself takes 2 · 3 · 9 = 54 seconds. Hence, the manual part of the job has to take 81 − 54 = 27
seconds.

Problem 8. Find all 9-digit numbers A which satisfy the following conditions:
• It contains each digit 1, . . . , 9 exactly once.
• Any two-digit number which is formed of neighbouring digits of A while keeping their order is divisible by 7 or 13.

Result. 784913526
Solution. Let us arrange the digits 1, . . . , 9 into a diagram, adding an arrow from x to y if the two-digit number xy is
divisible by 7 or 13.
3

9 1

7 8 4 5 6

2
(The solid arrows stand for divisibility by 7, the dashed ones for 13, and the dash-dotted one for divisibility by both
numbers.) It is clear from the diagram that the number in question has to begin with digits 784. If the next digit is 9,
then we have to go on with 1, 3, and 5, and the remaining two digits have to be in the order 2, 6. This way we obtain
the solution 784913526.
If, on the other hand, we carry on as 7842, there are two cases two consider. Firstly, continuing to 1 and subsequently
3, there is no way for the number to contain both 5 and 9. Secondly, we run into the same issue when carrying on as
784263, so the solution above is the only one.

Problem 9. Two squares are inside a larger square as in the picture. Find the area of the square A if the area of the
square B is 48.

Result. 54
Solution. Since the triangles adjacent to the sides of square B are isosceles, the side of B lying on the diagonal is
precisely
√ the middle third of the diagonal. Therefore, if s is the side length of the large square, the side length of B is
1
3 · 2 · s and the one of A is 12 · s. Therefore, the area ratio of the inscribed squares is
s2 2 · s2 9
: =
4 9 8
9
so the area of square A is 48 · 8 = 54.

Problem 10. Fiona has two cubes, one of side length 9 cm composed of white unit cubes (i.e. cubes of side length
1 cm), and one of side length 10 cm composed of black unit cubes. With these unit cubes she wants to build up a cube
of side length 12 cm. At least how many cm2 of the surface have to be black?
Result. 0
Solution. Fiona has 93 = 729 white and 103 = 1000 black unit cubes. In order to build up the surface of a cube of
side length 12 she needs 123 − 103 = 1728 − 1000 = 728 unit cubes. As a consequence, she can compose a cube of side
length 12 with all six faces being completely white. Therefore, the answer is 0.

Problem 11. After marking a mathematics test, the teacher found out that exactly ten of his pupils could not
multiply fractions, fourteen of them could not add them, and seventeen could not rationalise them. Moreover, every
student lacked at least one of the skills and there were exactly six pupils who lacked all three of them. At most how
many pupils attended the class?
Result. 29
Solution. In order to find out the size of the class precisely, we would also need the information about how many
students lack two of the skills for each pair of the skills. However, it is easy to see that we obtain the largest number if
we assume that everyone who lacks at least two skills in fact lacks all three of them. In that scenario, the total number
of pupils is
10 + 14 + 17 − 2 · 6 = 29.
We have to subtract the number of totally unskilled pupils twice, since when adding the sizes of the three other groups,
we count these pupils three times.

Problem 12. One of the angles of a right-angled triangle is 23◦ . Find the angle (in degrees) between the median
and the altitude lines both drawn from the right angle.
Result. 44
Solution. Let ABC be a triangle with ∠BAC = 90◦ . Connect A with the midpoint M of BC to get the median, draw
the altitude from A, and denote its intersection with BC by H.
C
H

M
23◦

23◦ 23◦
A B

3
By the theorem of Thales, the vertices of triangle ABC lie on a circle with center M . W.l.o.g. let ∠CBA = 23◦ . Since
triangle ABM is isosceles, we also have ∠BAM = 23◦ . Furthermore, triangle AHC is similar to triangle ABC. This
yields ∠HAC = 23◦ . Finally, we get ∠M AH = 90◦ − 2 · 23◦ = 44◦ for the measure of the desired angle.

Problem 13. Positive integers a and b satisfy 20a + 19b = 365. Find the value of 20b + 19a.
Result. 376
Solution. Clearly, we have a, b ≤ 20. Adding b to both sides of the given equation we get 20(a + b) = 365 + b. Now
the left-hand side is divisible by 20, hence the right-hand side must equal 380. This yields b = 15, hence a = 4 and we
just need to compute 20b + 19a = 380 − a = 376.

Problem 14. A regular polygon with 2018 vertices has 2,033,135 diagonals. How many more diagonals are there in
a regular polygon with 2019 vertices? Note that in both cases, we do not count the sides of the polygon as diagonals.
Result. 2017
Solution. The regularity of the polygons involved plays no role and we may assume that the 2019-gon is made from
the 2018-gon by dividing one of the edges by a new vertex. This new vertex is connected by 2016 new diagonals to
2016 non-neighbor vertices and there is also one new diagonal connecting its two neighbors, so in total, the numbers of
diagonals increases by 2017.
2
Problem 15. Find all real solutions of the equation (x2 − 4x + 5)x +x−30 = 1.
Result. 2, 5, −6
Solution. Since x2 − 4x + 5 = (x − 2)2 + 1 ≥ 1, the base is always a positive real number. Hence the only possibilities
to fulfill the equation are the base equals 1 or the exponent equals 0. In the first case, x2 − 4x + 5 = 1 is equivalent to
(x − 2)2 = 0 which yields x = 2. In the second case, x2 + x − 30 = (x − 5)(x + 6) = 0 has the two solutions x = 5 and
x = −6. All in all, there are three solutions.

Problem 16. How many permutations of numbers 1, 2, 3, 4 are such that whenever we erase one of the numbers,
the remaining sequence of numbers is neither increasing nor decreasing?
Note: A permutation is a sequence containing each of the numbers exactly once.
Result. 4
Solution. Suppose 1 is the starting number. In order to comply with the condition about increasing, the permutation
has to be (1, 4, 3, 2). But since erasing 1 yields a decreasing sequence of numbers, this permutation does not fulfill the
given conditions. Therefore, 1 cannot be the starting number, and by symmetry it cannot be the ending number, too.
Similarly, 4 cannot be neither the first nor the last number. Hence 1 and 4 must be in the middle. Both orders (1, 4)
and (4, 1) yield two valid permutations:
(2, 1, 4, 3), (3, 1, 4, 2), (2, 4, 1, 3), (3, 4, 1, 2).
Therefore there are four permutations which satisfy the conditions.

Problem 17. Let ABCD be a rectangle with AB = 8 cm and BC = 6 cm. Further let X and Y be the intersections
of the perpendicular bisector of AC with AB and CD, respectively. What is the length of XY (in centimeters)?
Result. 152
√ √
Solution. By the Pythagorean theorem, we get AC = AB 2 + BC 2 = 82 + 62 = 10. Let S be the intersection of
AC and its perpendicular bisector. Clearly, S is the midpoint of the diagonal, i.e. we have AS = 5.
Y
D C

A B
X
Since ∠CAB = ∠SAX and ∠XSA = ∠CBA = 90◦ , triangles ASX and ABC are similar, yielding SX : AS = BC : AB
or
BC · AS 15
SX = = .
AB 4
Finally, the length we are looking for is XY = 2 · SX = 15
2 .

4
Problem 18. In the cryptarithm F OU R + F IV E = N IN E each letter represents only one digit throughout the
problem and different letters represent different digits. Furthermore, it is known that
• F OU R is divisible by four,
• F IV E is divisible by five,
• N IN E is divisible by three.
Find all possible values of N IN E.
Result. 3435
Solution. By looking at the units digits, we immediately get R = 0. Since F IV E is divisible by 5 and R = 0, we must
have E = 5. From the hundreds digits and knowing that the digit 0 already is taken, we conclude O = 9 and get a
carry of 1 from the tens digits and one into the thousands digits. Therefore, U + V must be greater than 10 and N has
to be an odd digit greater than 1. On the other hand, U + V ≤ 7 + 8 = 15 because the digit 9 is already taken. This
limits the possibilities for N to N = 3. Consequently, U = 6 (because of the divisibility by 4), V = 7 and F = 1.
Since N IN E is divisible by 3, its digit sum N + I + N + E = 3 + I + 3 + 5 = 11 + I must be divisible by 3, too.
Out of the three digits 1, 4 and 7 which would be suitable for I, only I = 4 is still available. Therefore, the answer is
N IN E = 3435 and the cryptarithm translates into 1960 + 1475 = 3435.

Problem 19. Let ABC be an equilateral triangle and CDEF a square such that E lies on segment AB and F on
segment BC. If the perimeter of the square equals 4, what is the perimeter of triangle ABC?
A

E D

B C
F

Result. 3 + 3
Solution. Denote further by G the intersection of AC and DE. Note that the two right-angled triangles having interior
angles 30◦ , 60◦ , and 90◦ , i.e. BEF and GCD are congruent, since they both have one side of length 1 of the square in
common. Such a right triangle is half of an equilateral triangle with its side length being the length of the hypotenuse
of the triangle. Therefore √ BE = 2BF and by the Pythagorean theorem, BE 2 = EF 2 2
√ + BF . Together with the √ fact
EF = 1 we obtain BF = 3/3. Therefore, the side length of triangle ABC is 1 + 3/3 and its perimeter is 3 + 3.

Problem 20. Let a, b be real numbers. If x3 − ax2 + 588x − b = 0 has a triple real root, what are the possible values
of a?
Result. 42, −42
Solution. If r is the triple root, then

(x − r)3 = x3 − 3rx2 + 3r2 x − r3 = x3 − ax2 + 588x − b.

Comparing the coefficients yields 3r2 = 588 or r = ±14. Hence we get a = ±42.

Problem 21. Simon is on a trip to islands which are connected by toll bridges as shown in the diagram. There is a
unique view from each bridge, so he wants to cross every bridge. Since he likes to save money, he intends to cross every
bridge exactly once. In how many ways can he plan the trip if he starts on the square-island? Simon cannot jump from
one bridge to another when he is not on an island and he can visit any island any number of times.

Result. 120

5
Solution. Note that the square island and the rightmost island in the middle are two ’special’ islands: All bridges
start in one of them, and from each ’usual’ island there are bridges to both of the special islands. So, we always go
from one of the special islands to the other through one of the usual islands. Therefore, all we need to determine the
journey is to order the usual islands, which can be done in 5! = 120 ways.

Problem 22. How many ordered pairs of positive integers (m, n) are there such that their least common multiple is
equal to 2000?
Result. 63
Solution. Let us distinguish two cases: Firstly, assume that none of the numbers equals 2000 = 24 · 53 . Then one of
the number is equal to 24 · 5k for any k ∈ {0, 1, 2} and the other one to 2l · 53 for any l ∈ {0, 1, 2, 3}, hence there are 24
such pairs (counting both possible orderings). Secondly, if one of the numbers is 2000, then the other number can be
any divisor of 2000 and there are (4 + 1) · (3 + 1) = 20 of them, so when taking the ordering into account, we obtain
2 · 20 − 1 = 39 pairs; we subtract one for the pair (2000, 2000), which is counted twice. In total, there are 24 + 39 = 63
pairs.

Problem 23. Let ABCDEF GH be a regular octagon with AC = 7 2. Determine its area.

Result. 98 2
Solution. Let M be the center of the circumcircle of the given octagon. Since ∠AM C = 2
8 · 360◦ = 90◦ , the radius of
the circumcircle has to be 7 and the diameter is 14.

C A

Due to the area rearrangement


√ shown
√ in the picture we get the area by multiplying AC with the diameter. Therefore
the area sought is 14 · 7 2 = 98 2.

C A

Problem 24. Four friends decided to start learning languages. The local language schools offers courses of Arabic,
Bengali, Chinese, and Dutch, and each friend wants to learn exactly three of those languages. In how many ways can
they pick the courses such that they all attend at least one course together?
Result. 232
Solution. Firstly note that a triple of languages can be assigned to a single person in exactly four ways—we can
equivalently choose the language the person is not going to learn. Hence there are 44 = 256 ways of assigning the
triples of languages to four people when disregarding the extra condition.
Let us now count the assignments causing that the four people have no course in common. The only possibility is
that for each of the four languages, there is a person not attending the course. Hence there are 4! = 24 assignments.
We conclude that the number of assignments satisfying the given condition is 256 − 24 = 232.

Problem 25. Having been told a positive integer n with non-zero digits, Abigail multiplied n by a number containing
the same digits, but in reversed order. She noticed that the result was one thousand more than the product of the
digits of n. Find all possible values of n.
Result. 24, 42

6
Solution. Clearly, n has at least two digits. If n has precisely two non-zero digits a and b, then the given statement
translates into
(10a + b)(10b + a) = 1000 + ab
or
a2 + b2 = 10(10 − ab).
Since the right hand side has to be positive, we get ab < 10. Therefore we have to work out only a couple of cases
obtaining that a, b are 2 and 4 in some order.
Finally, if n has k ≥ 3 digits, then the left-hand side of the equality is at least (10k−1 )2 , whereas the right-hand
side is less than 1000 + 10k . As a consequence, n cannot have more than two digits.

Problem 26. Let ABCD be a parallelogram and let T be an interior point of AD such that the straight line through
T and C is the bisector of ∠BCD. Further, let E be a point on AB such that ∠AET = 40◦ . If ∠CT E = 75◦ , what is
∠CDA (in degrees)?
Result. 110
Solution. Let S be a point on BC such that T S is parallel to AB. Then ∠ET S = 40◦ and

∠DCT = ∠ST C = ∠CT E − ∠ST E = 35◦ .

Since CT is the bisector of ∠DCB, triangle CT S is isosceles, ∠DCT = ∠T CS and ∠DCB = 2 · 35◦ = 70◦ . Hence
∠CDA = 180◦ − ∠DCB = 110◦ .
D C
◦ 35◦
110
35◦

110◦
T 35◦
40◦ S

40◦
A B
E

Problem 27. Two large noble houses met at a feast, both represented by at least one male and at least one female
member. Each member of a house greeted each member of the other house: When two men greeted each other, they
shook hands, whereas two women or a woman and a man bowed to each other. When the greeting was over, altogether
85 handshakes and 162 bows had happened. How many women were present at the feast? We count it as one bow
when two people bow to each other.
Result. 10
Solution. Let m1 , m2 , w1 , w2 be the number of men and women in the houses, respectively. Since m1 m2 = 85 = 5 · 17,
we see that w.l.o.g. m1 = 5 and m2 = 17 (the possibility with one of the numbers being 1 is easily excluded).
Furthermore, there were 85 + 162 = 247 greetings altogether, and from

(m1 + w1 )(m2 + w2 ) = 247 = 13 · 19

we obtain m1 + w1 = 13, m2 + w2 = 19 (the other factorizations are again easily excluded), hence w1 = 8, w2 = 2 and
the answer is 8 + 2 = 10.

Problem 28. Consider a triangle with side lengths 10, 24, and 26. Let c be a circle with its centre on the longest
side and tangent to the two shorter sides. Find the radius of the circle c.
Result. 120/17
Solution. The Pythagorean theorem shows that the triangle is a right triangle. The segment connecting the vertex of
the right angle with the centre of the circle splits the original triangle into two triangles. The radii to the tangent
points are perpendicular to the shorter sides of the triangle, hence they are altitudes in the smaller triangles. Let r be

7
the radius of circle c. We calculate the area of the original triangle in two ways, using the known dimension of the
original triangle and the smaller triangles:
1 1 1
S= · 24 · 10 = · 24 · r + · 10 · r,
2 2 2
which implies r = 120/17.

r
10
r

24

Problem 29. Margarita came to a casino with 10 e. A slot machine in the casino works as follows: The gambler
inserts 1 e using any coins and with probability p she wins the jackpot; otherwise it returns 0.5 e. Help Margarita find
the smallest probability p such that if she decides to play this slot machine as long as possible or until winning the
jackpot, she will have at least 50% chance of winning the jackpot.

Result. 1 − 19 0.5
Solution. Note that Margarita can play the machine unsuccessfully at most 19 times, since after spending her last 1 e
she will have only 0.5 e left, which is less than the machine accepts. The probability of each loss is 1 − p, therefore
the probability of not winning the jackpot is (1 − p)19 . In order
√ to have at√least 50% chance of winning the jackpot,
(1 − p)19 ≤ 0.5 has to hold, which is equivalent to p ≥ 1 − 19 0.5 and 1 − 19 0.5 is the sought smallest value of p.

Problem 30. Find all four-digit positive integers abcd which are equal to aa + bb + cc + dd . None of the digits may
be zero.
Result. 3435
Solution. As 66 ≥ 10000, none of the digits may exceed 5. On the other hand, if all the digits were 4, the equation
would not hold, and if there were at most three 4’s, the number would be at most 3 · 44 + 33 < 1000. Therefore one of
the digits has to be 5. Since 55 = 3125, we see that the digit 5 has to appear exactly once, for if not, the first digit of
the number would have been greater than 5. Now 3000 < 55 < 55 + 3 · 44 < 4000, hence the first digit of the number
is 3.
At this moment we know that the number in question is at least 55 + 33 + 2 · 11 = 3154, which does not work and
neither does 3155. The next number not containing zeros and digits exceeding 5 is 3211 > 55 + 3 · 33 , therefore one of
the digits has to be 4. There cannot be another 4 since that would make the second digit exceed 5 and working out the
three remaining cases, we conclude that the only solution is 3435.

Problem 31. How many quintuples of two-digit numbers are there such that each of the digits 0 to 9 appears exactly
once and each of the two-digit numbers is even, but not divisible by three? Quintuples which differ only by the order of
numbers are considered equal.
Result. 16
Solution. All the five numbers have to end with an even digit. To satisfy the non-divisibility by three, numbers ending
with 0 or 6 have to start with one of 1, 5, 7, those ending with 2 or 8 have to start with one of 3, 5, 9, and the one
ending with 4 has to start with one of 1, 3, 7, 9. If we pick the number ending with 4 to be 14, then there are only two
choices for the tens digit for 0 and 6 (either 50, 76, or 56, 70), and consequently, there are only two choices for the tens
digit for 2 and 8 (either 32, 98, or 38, 92), hence four options in total. A similar argument applies for whatever choice
of the tens digit for 4, and since there are four digits to choose from, the total number of quintuples is 4 · 4 = 16.

Problem 32. Find all positive integers n such that


jnk jnk j n k
+ + = 2019.
5 7 35
Note: The expression bxc denotes the integer part of x, i.e. the greatest integer not exceeding x.
Result. 5439

8
Solution. Let jnk jnkjnk
f (n) = + + .
5 7 35
Clearly, f is a non-decreasing function of n. Further, f (n) − f (n − 1) = 1 if n is divisible by exactly one of the numbers
5 or 7, and f (n) − f (n − 1) = 3 if n is divisible by 35; in all the other cases f (n) = f (n − 1) holds. Since
 
1 1 1 13
f (n) ≤ + + n= n,
5 7 35 35
we obtain an estimate for the solution
35
n≥ · 2019
13
and since n is an integer, n ≥ 5436. Now f (5436) = 2018; the closest larger number divisible by 5 is 5440 and the one
divisible by 7 is 5439, hence by the above, f (5439) = 2019, f (5440) = 2020, and 5439 is the only solution.

Problem 33. For how many positive integers n can one find positive integers x, y ≤ 1 000 000 (not necessarily
distinct) such that
n = S(x) = S(y) = S(x + y),
where S(a) denotes the sum of digits of a?
Result. 6
Solution. Since for any positive integer a, S(a) and a leave the same remainder after division by 9, we infer from the
statement that the numbers n, x, y, and x + y leave the same remainder after division by 9. This implies that x and
consequently n have to be a multiple of 9. If n = 9m for a positive integer m, then for the choice x = y = 10m − 1 we
obtain the equality from the statement. The largest digit sum for numbers at most one million is 54, so there are six
such n: 9, 18, 27, 36, 45, and 54.

Problem 34. A pentomino composed of five squares of side length a is inscribed in a rectangle of dimensions 7 × 8
as in the picture:
7

Find a.

Result. 5
Solution. Let c and d be the length of the longer and shorter projection of a side of a square onto a side of the box,
respectively.

d
c c c d
Then

3c + 2d = 8,
3c + d = 7,

therefore c = 2 and d = 1. Using the Pythagorean theorem we deduce that


p √
a = c2 + d2 = 5.

9
Problem 35. Paul has a rectangular 5 by 3 bar of chocolate. He has put extra sugar on the top left square to make
it sweeter. He proceeds to eat the chocolate in the following way: In each step he randomly chooses either eating the
rightmost column, or the lowest row, both options having the same probability 1/2. He repeats these steps until he has
eaten the whole chocolate. What is the probability that in the very last step, he eats only the sweeter square?
Result. 15/64
Solution. Assume that the chocolate has three columns and five rows. We may view the process as follows: Paul picks
a sequence of C’s and R’s of total length (5 − 1) + (3 − 1) = 6, and based on this sequence, he eats the columns or rows
of the chocolate. There are two possibilities: Either the sequence contains exactly two C’s (and four R’s), in which case
only the sweeter square remains after the process, or the number of C’s or R’s is at least the number of columns or
rows, respectively, which leads to the whole chocolate being eaten. In the latter case, the last step cannot consist of
eating solely the sweeter square, since there are not enough columns or rows eaten to reduce the last row or column to
a single square.
There are 26 sequences of C’s and R’s (of length 6) in total, out of which 62 contain exactly two C’s, hence the


sought probability is
6

2 15
= .
26 64

Problem 36. Using each digit 1, . . . , 9 exactly twice, Greg formed several pairwise distinct prime numbers so that
the sum of these primes was the minimal possible. What was the value of the sum?
Result. 477
Solution. No prime numbers except 2 and 5 may end with a 5 or an even digit, therefore each of the digits

2, 5, 4, 4, 6, 6, 8, 8

has to appear at least on the tens position of some number in Greg’s set. Further, each of the remaining digits

2, 5, 1, 1, 3, 3, 7, 7, 9, 9

has to appear at least on the units position of some number in Greg’s set. Therefore the sum is at least

10(2 + 5 + 4 + 4 + 6 + 6 + 8 + 8) + 2 + 5 + 1 + 1 + 3 + 3 + 7 + 7 + 9 + 9 = 477.

On the other hand, this sum is achievable by forming the prime numbers as follows:

2, 5, 29, 53, 41, 47, 61, 67, 83, 89.

Problem 37. Tom and Jerry have two polynomials T (x) = x2 + 2x + 10 and J(x) = x2 − 8x + 25. When each of
them substituted their favourite positive integer t, j into their respective polynomial they got the same result, i.e.
T (t) = J(j). Find all possible values of |t − j|.
Result. 1, 5
Solution. Factoring the left-hand side of the equation T (t) − J(j) = 0 produces (t + j − 3)(t − j + 5) = 0 and therefore
either {t, j} = {1, 2} or |t − j| = 5.

Problem 38. The altitude from vertex A of triangle ABC has the same length as the median from vertex B.
Moreover, we know that the angle ABC equals 75◦ . Find the ratio AB : BC.

2 √1
Result. 2 = 2
Solution. Define E as refection of B with respect to the midpoint M of AC and F the perpendicular projection of
EF
E on line BC. Then it follows from the first given condition that sin(∠EBC) = BE = 12 and thus ∠EBC = 30◦ (it
cannot be obtuse because of the second given condition). Hence ∠ABM = ∠ABE = 75◦ − 30◦ = 45◦ .

A E

45◦
B 30◦
F
C

10
From the law of sines in triangles ABM and CBM we compute
BM AM AM
= = √
sin(∠BAC) sin 45◦ 2
2

and
BM CM AM
= = 1 .
sin(∠BCA) sin 30◦ 2

By dividing these equalities (note that AM = CM ) and using the law of sines in triangle ABC we finally get

AB sin(∠BCA) 2
= = .
BC sin(∠BAC) 2

Problem 39. In noughts and crosses, two players alternately put noughts and crosses into a 3 by 3 table. One of
them wins if there are three of his symbols in a row, column, or a diagonal. The game ends in a draw if the table is
completely filled and neither of the players won. How many different arrangements of the table are possible in the case
of a draw? We do not identify arrangements differing by rotating or reversing the table. Either of the players can start
the game.
Result. 32
Solution. We consider four separate cases according to the symbol in the middle square and the symbol which appears
five times in the table. If there is a cross in the middle square and there are five crosses in the table (we call this the
case “Cross-Cross”) we must put four more crosses into the table. We can not use any whole diagonal and any whole
“axis”. Consequently we end up with a pattern (Figure 1) which is not symmetric with respect to any rotation or axial
symmetry and therefore this case contributes with 8 different draw endings.

Figure 1

In the case “Cross-Nought” we have to place five noughts in the table with cross in the middle. We can not use all
four corners since the fifth would make noughts win. On the other hand we have to put at least one nought on both
diagonals, otherwise the crosses win. Hence we have to put either exactly three noughts or exactly two noughts in the
corner squares and in both these subcases we have exactly one possibility to complete the table to a draw ending. Both
of them are one of the 4 rotations (both have an axis of symmetry) of one of the patterns (Figure 2 and Figure 3).

Figure 2 Figure 3

The cases “Nought-Nought” and “Nought-Cross” are analogous to the two cases discussed above and thus the total
number of draw endings is 2(8 + 4 + 4) = 32.

Problem 40. Find the largest positive integer a such that no positive integer b satisfies
4 a 25
< < .
3 b 18

Result. 32

11
Solution. Taking reciprocals we obtain
b
0.72 < < 0.75.
a
The interval between 0.72 and 0.75 has length 0.03 > 1/34, hence for all a ≥ 34 there is a solution. For a = 33 there is
a solution b = 24 (giving b/a = 0.7272 . . . ). For a = 32 there is no solution as 24/32 = 0.75 and 1/32 > 0.03.

Problem 41. On a bicycle tour of length 110 km from Passau to Linz, Heiko and Eva have to overcome three climbs.
During their first stop, Heiko, who is excellent at mental arithmetic, says: “If you multiply the three distances from
Passau to the peak of each climb, you get a multiple of 2261.” After thinking about that for a while, Eva responds:
“You also get a multiple of 2261 if you multiply the distances measured from Linz instead of Passau.” After riding
80 km from the start they make a second stop and Heiko remarks: “Now there is only one climb ahead before we arrive
in Linz.” Assuming that all the distances are road distances in kilometres and that they are integers, find the distances
from Passau to the peak of each climb in kilometres.
Result. 68, 76, 91
Solution. Let A, B, and C be the three distances of the peaks from Passau measured in km. These distances have to
satisfy 2261 | ABC and 2261 | (110 − A)(110 − B)(110 − C). Due to 2261 = 7 · 323 = 7 · 17 · 19 and 7 · 17 = 119 > 110
none of the distances can obtain more than one prime factor of 2261.
W.l.o.g. 7 | A, 17 | B, and 19 | C. Since the same considerations apply to the distances 110 − A, 110 − B and
110 − C, we get the two possibilities 7 | (110 − B) and 7 | (110 − C) due to 7 - (110 − A). In the case 7 | (110 − B) we
get 19 | (110 − A) from 19 - (110 − C), and finally 17 | (110 − C). In the case 7 | (110 − C) we get 17 | (110 − A) and
19 | (110 − B) in a similar way.
Since GCD(7, 19) = 1, the only way to decompose 110 as a·7+b·19 with a, b non-negative integers is 110 = 13·7+1·19
(all the decompositions are of the form 110 = (13 + 19k) · 7 + (1 − 7k) · 19 for k ∈ Z and the coefficients are non-negative
only for k = 0). Similarly, we get the decompositions 110 = 4 · 17 + 6 · 7 and 110 = 4 · 19 + 2 · 17. This leads to the two
solutions
A = 13 · 7 = 91, B = 4 · 17 = 68, C = 4 · 19 = 76
and
A = 6 · 7 = 42, B = 2 · 17 = 34, C = 19.
Heiko’s remark during the second stop indicates that the third peak is at least 80 km away from Passau. As a
consequence, the sought distances are 68, 76, and 91.
√ √
Problem 42. Let ABC be a right-angled triangle with right angle at C such that AC = 4 − 3 and BC = 3.
Furthermore, let D and E be points such that ABDE is a square not containing point C in its interior, and let J be a
point on DE such that ∠ACJ = 45◦ . Finally, let K be a point on CJ such that AK k BC. What is the area of the
triangle JKE?

Result. 3 3/8
Solution. Firstly, note that ∠EKA = 90◦ ; in fact, triangles AEK and ABC are congruent since AK = AC, AE = AB,
and ∠EAK = ∠BAC. Further, the centre S of square ABDE lies on the circumcircle of triangle ABC because
ASB and ACB are right angles. Since AS = BS, the corresponding inscribed angles ACS and BCS are equal as
well. Therefore S lies on the angle bisector CJ. Let us reflect the triangle JKE across S; then E is mapped to B,
J is mapped to H, which is the intersection of AB and CJ, and K is mapped to I, which lies on CJ and satisfies
∠IBC = 90◦ .
D

K
E
B I S

C A

Now IBC is an isosceles right-angled triangle with right angle at B, so its area is ( 3)2 /2 = 3/2. Denoting by square
brackets the area of triangle, we have
[IBC] IC IH + HC HC
= = =1+ .
[IBH] IH IH IH

12
Furthermore, triangles ACH and BIH are clearly similar, the coefficient of similarity being
HC AC AC
= = .
IH IB BC
We conclude that √ √
BC 3 3 3 3
[JKE] = [IBH] = [IBC] · = · = .
AC + BC 2 4 8

Problem 43. Two prisoners are standing in front of two boxes. They know that one box contains two white marbles
and one black marble and the other box contains one white marble and two black marbles; however, they do not know
which box is which. Each prisoner is to choose a box and randomly draw a marble from it (without replacement). If
they draw a white marble, they are free, otherwise they are executed. If the second prisoner witnesses the first one’s
choice and its outcome and proceeds in the most logical way, what is the probability of his survival before the first
marble is drawn? We assume that the first prisoner picks the box randomly.
Result. 5/9
Solution. Let us denote c the colour the first prisoner drew and o the other colour. Then the probability that he drew
from box with c, c, o is 2/3 and the probability that he drew from the box with c, o, o is 1/3. If the second prisoner
chooses the same box as the first one, he will draw colour c with probability
2 1 1 1
· + ·0= ,
3 2 3 3
and thus colour o with probability
1 2
= .
1−
3 3
On the other hand if he chooses the other box, he will draw colour c with probability
2 1 1 2 4
· + · = ,
3 3 3 3 9
and colour o with probability
4 5
1− = .
9 9
In case c is white, the second prisoner will survive if he draws c. Since 49 > 13 , it is better for him to choose the
other box, and he will survive with probability 49 . In case c is black, the second prisoner will survive if he draws o.
Since 23 > 59 , it is better for him to choose the same box, and he will survive with probability 23 . Clearly c is white with
probability 21 and black with probability 12 , hence the second prisoner will survive with probability

4 1 2 1 5
· + · = .
9 2 3 2 9

Problem 44. What is the smallest positive integer n such that among any n (not necessarily distinct) real numbers
from the interval [1, 2019], there are three numbers representing lengths of sides of a nondegenerate triangle?
Result. 18
Solution. For n < 18 take the first n elements of the Fibonacci sequence given by a1 = a2 = 1, ak+2 = ak+1 + ak :

1, 1, 2, 3, 5, 8, 13, 21, 34, 55, 89, 144, 233, 377, 610, 987, 1597.

Clearly the greatest number of any triple formed from these numbers is greater or equal to the sum of the two others
and hence no such triple can form a nondegenerate triangle. For n = 18, let x1 ≤ · · · ≤ x18 be the chosen numbers. If
no three of them form a nondegenerate triangle, we get

x1 , x2 ≥ 1, x3 ≥ x1 + x2 ≥ 2, x4 ≥ x3 + x2 ≥ 2 + 1 = 3, ...

obtaining in each step a term of the mentioned Fibonacci sequence and thus we end with x18 ≥ 987 + 1597 > 2019,
which is impossible. The desired number is therefore n = 18.

13
Problem 45. Let σ(k) denote the number of all (positive) divisors of a positive integer k. Find the smallest positive
integer n such that the highest common factor of σ(n) and σ(n3 ) is not a power of two (including 1).
Result. 432 = 24 · 33
Solution. Recall that if
n = pα α2 αt
1 · p2 · · · pt
1

is the prime factorization of n, then


σ(n) = (α1 + 1)(α2 + 1) · · · (αt + 1).
The condition that the highest common factor is not a power of two can be equivalently restated that there is an odd
prime number q dividing both σ(n) and σ(n3 ). Since
σ(n3 ) = (3α1 + 1)(3α2 + 1) · · · (3αt + 1),
this number is never divisible by 3, therefore the smallest possible value of q is 5. Further note that q cannot divide
αi + 1 and 3αi + 1 at the same time, for otherwise it would have to divide
3(αi + 1) − (3αi + 1) = 2.
Therefore there are distinct i, j ∈ {1, . . . , t} such that q | αi + 1 and q | 3αj + 1. Since we are looking for the smallest
number, we may assume that t = 2, i = 1, and j = 2.
If q = 5, the smallest possible values for α1 , α2 are α1 = 4, α2 = 3, and taking the smallest possible primes, i.e.
p1 = 2, p2 = 3, we obtain n = 24 · 33 = 432.
If q ≥ 7, then α1 ≥ 6 and α2 ≥ 2, yielding
n ≥ 26 · 32 = 576 > 432,
showing that 432 is indeed the smallest possible value of n.

Problem 46. Let ABC be a right-angled triangle with ∠ACB = 90◦ , AC = 15, BC = 20. Let D be the point on
the line AB such that CD ⊥ AB. The circle t inscribed in the triangle ACD touches the line CD at T . Another circle
c also touches the line CD at T , and it touches the line segment BC. Denote the two intersections of the circle c with
the line AB by X and Y . What is the length of XY ?

Result. 3 5
Solution. Using the formulas for the segments of the hypotenuse, we compute
AC 2 BC 2 √
AD = = 9, BD = = 16, CD = AD · BD = 12.
AB AB
The radius of circle t (which is equal to DT ) can be computed using the well-known formula by dividing the area of the
triangle ACD by the half of its perimeter: We get that DT = 54/(36/2) = 3. Let the circle inscribed in the triangle
BCD, denoted by ω, touch the line CD at S. Similarly, its radius is equal to DS = 4. The homothety with the center
at C and ratio CT /CS = 9/8 sends the circle ω to the circle c. Therefore, the radius of the circle c is 4 · 9/8 = 9/2. Let
M be the midpoint of the segment XY and O the center of the circle c. We know that XO = 9/2 and OM = DT = 3.
Henceforth, using the Pythagorean theorem we conclude that
r

r
92 2
9
XY = 2 · XM = 2 −3 =6 − 1 = 3 5.
22 4
A

D
X

T c
M
S
t
Y
O

ω
C B

14
Problem 47. Every square of a 6 × 7 chessboard is coloured white, green, red, or blue. We call such a colouring
attractive if the four squares of any 2 × 2 square have distinct colours. How many attractive colourings are there?
Result. 1128
Solution. Let us make two observation. Firstly, since the colours of any two consecutive squares are clearly different
then if there are more than two colours present in a row then there are three consecutive cells coloured by three different
colours in that row. Secondly, such a triple completely determines the colouring of the respective three columns, e.g.
triple 1 − 2 − 3 forces the vertically neighbouring triples to be 3 − 4 − 1 and these two triples have to alternately fill the
respective three columns making them all contain just two colours. The same holds for columns instead of rows as well.
Hence there cannot be more than two colours used in some row and more than two colours used in some column at the
same time.
Assume now that the table has 6 rows and 7 columns. Let us compute the number of colourings using only
two colours in every row: We choose the two colours for the first row (then the pair of colours for any other row is
also determined) and then choose the starting colour in each of the 6 rows. This yields 42 · 26 = 6 · 26 colourings.


Analogously the number of colourings using only two colours in every column is 6 · 27 . Finally we have to subtract the
number of colourings using only two colours in every row and every column. Since such a colouring is determined by
the upper left 2 × 2 square there are 4 · 3 · 2 = 24 of them.
The result is 6 · 26 + 6 · 27 − 24 = 1128.

Problem 48. One hundred children stand in a row. The first child has 4 grams of chocolate, the second child has 8
grams of chocolate and so on until the 100th child has 400 grams of chocolate. The first child gives one third of their
chocolate to the second child (so the second child now has 283 grams of chocolate). The second child gives one third of
their chocolate to the third child and so on until the 99th child gives one third of their chocolate to the 100th child.
How many grams of chocolate will the 100th child have in the end?
Result. 597 + 3−99
Solution. All weights are in grams. After the first step, the second child has 8 + 4/3. In the second step, it gives one
third of their chocolate to the third child which has 12 + 8/3 + 4/32 then. After the third step, the fourth child has
16 + 12/3 + 8/32 + 4/33 . It is easy to observe that in the end, the 100th child has
 
99 98 97 2 1
4 100 + 1 + 2 + 3 + · · · + 98 + 99 .
3 3 3 3 3
Let us denote
99 98 97 2 1
S = 100 + + 2 + 3 + · · · + 98 + 99 .
31 3 3 3 3
Then we have
1 1 1 1
S= 1+
+ 2 + ··· + 98
+ 99 +
3 3 3 3
1 1 1
+ 1 + + 2 + ··· + +
3 3 398
..
.
1
+1+ +
3
+ 1.
Using the sum of geometric series,
1
1 − 3n+1
 
1 1 1 3 1
1+ + 2 + ··· + n = = 1− ,
3 3 3 1 − 13 2 3n+1
we get
  
3 1 1 1
S= 100 − + + · · · +
2 3100 399 3
  
3 1 1 1
= 100 − + + · · · + 1
2 3 399 398
  
3 1 1
= 100 − 1 − 100 .
2 2 3
Finally we obtain   
3 1 1 1
4S = 4 · 100 − 1− = 600 − 3 + = 597 + 3−99 .
2 2 3100 399

15
Problem 49. Find all integers n ≥ 3 for which
(n − 1)n−1 − n2 + 2019 · (n − 1)
(n − 2)2
is an integer.
Result. 3, 4, 5, 6, 8, 14
Solution. We would like n to satisfy (n − 2)2 | (n − 1)n−1 − n2 + 2019 · (n − 1). This is not influenced by adding
(n − 2)2 to the right-hand side and it allows us to get rid of the term n2 , since n2 − 4n + 4 − n2 = −4(n − 1). We
obtain an equivalent condition
(n − 2)2 | (n − 1)n−1 + 2015 · (n − 1).
Since n−1 and n−2 are coprime, we can divide the right-hand side by n−1. Substitute t = n−2. Then t2 | (t+1)t +2015.
Finally, we use the binomial expansion to get
     
2 t t t−1 t 2 t
t |t + t + ··· + t + t + 1 + 2015,
t−1 2 1
so all what is needed is t2 | 2016. The prime factorization of 2016 is 25 · 32 · 7, so the values 1, 2, 3, 4, 6, 12 are the only
possible (and satisfying) for t. Substituting back into n = t + 2 leads to the result 3, 4, 5, 6, 8, 14.

Problem 50. Let ABC be an equilateral triangle with each vertex on one of three concentric circles with radii 3, 4,
and 5, respectively. Find all possible side lengths of the triangle.
p √ p √
Result. 25 − 12 3, 25 + 12 3
Solution. Let A be on the circle with radius 3, B on the one with radius 4, C on the circle with radius 5, and let S be
the centre of the circles. There are two cases to consider.
First assume that S lies outside triangle ABC. Rotating C and S by 60◦ around B we obtain points C 0 = A and
S . Then triangle SBS 0 is equilateral with side length 4, and S 0 C 0 = SC = 5. Triangle SS 0 C 0 has side lengths 3, 4,
0

5, hence ∠S 0 SC 0
p = 90 √

. Hence ∠BSA = ∠S 0 SC 0 − ∠S 0 SB = 30◦ . Using the law of cosines in the triangle BSA we
obtain AB = 25 − 12 3.

S0

60◦
B
S
A = C0
C

If S lies inside triangle ABC, we can rotate A and S by 60◦ around B to get points A0 = C and S 0 . Similarly,
triangle SS 0 A0 is right with ∠SS 0 A0 = 90◦ . Hence ∠BSA = p
∠BS 0 A0 = ∠SS 0 A0 + ∠SS 0 B = 150◦ . Using the law of

cosines in triangle BSA we obtain the second solution AB = 25 + 12 3.
C = A0

S0
60◦

B
S
A

16
Problem 51. Seven people are sitting (equally-spaced) around a round table, on which seven arrows are drawn so
that exactly one starts at each seat and exactly one points to each seat (the starting and the ending seat do not have
to be distinct). Every minute, people change their seats, exchanging their current seat for the one where the arrow
from their seat points, and the table is rotated by one seat clockwise. What is the maximal number of minutes that
will take all the people to get back to their initial seats simultaneously and for the first time?
Result. 84
Solution. Note that every 7 rounds people permute their seats around the table, but the table gets back to its
initial position. Therefore, we see that looking at every 7th minute people will just change their places as in normal
permutation. The minimal number of times a permutation on seven elements can be applied before elements return to
their initial positions (i.e. the order of the permutation) can be at most 12: This is obtained for a permutation which
permutes elements like
1 → 2 → 3 → 1; 4 → 5 → 6 → 7 → 4.
In general, the order is the least common multiple of lengths of the cycles in the decomposition of the form above; it is
easy to see that 12 is indeed the maximal order. This gives an upper bound of 7 · 12 = 84 minutes required to get
people back to their initial seats.
If the arrows are drawn on the table so that initially only people 1 and 4 exchange their seats, then after seven
minutes, the people are permuted according to the permutation

1 → 2 → 3 → 1; 4 → 7 → 6 → 5 → 4,

so if we consider only every 7th minute, then the procedure indeed takes 84 minutes. It remains to observe that the
people cannot get to their initial seats in the meantime: Every 7th minute the seats 1, 2, and 3 are occupied by their
(possibly permuted) original occupants, but in the subsequent six minutes, at least one of these people sits away from
these three seats.

Problem 52. Let a1 , a2 , a3 , . . . be a sequence of positive real numbers. Starting from the second term a2 each
number is half the sum of the arithmetic and geometric mean of the two neighbour numbers. Find a333 if it is known
that a1 = 27 and a11 = 72 .

Note: The geometric mean of positive reals x and y is defined as xy.
Result. 2016
Solution. The condition translates into
ak−1 +ak+1 √ √ √ 2
2 + ak−1 · ak+1 ak−1 + ak+1
ak = =
2 4
for every k ≥ 2. Since the terms of the sequence are positive, this can be rewritten as
√ √
√ ak−1 + ak+1
ak = .
2

This pimplies that the sequence
p b1 , b2 , . . . where bk = ak is arithmetic; let d be its common difference. We know that
b1 = 2/7 and b11 = 7/2, so p p
7/2 − 2/7 1
d= = √ .
10 2 14
We infer that

r
2 332 4 + 332
b333 = b1 + 332 · d = + √ = √ = 12 14.
7 2 14 2 14
It follows a333 = b2333 = 2016.

Problem 53. Adam has a rectangle with perimeter 444 and sides positive integers a, b satisfying a > b. He tried to
cover it by squares with side length a − b putting the first square into the upper left corner and then following the
pattern of a square grid with axes parallel to the sides of the rectangle and the origin being the top left corner of the
rectangle. At some point (after putting at least one square) he had to stop because no more squares lying completely
in the rectangle could be added. Area of the uncovered part of the rectangle is 1296. Summing over all such rectangles,
find the sum of all possible side lengths of the square used for filling the rectangle.
Result. 166

17
Solution. We have a ≡ b ≡ r (mod a − b) where 0 ≤ r ≤ a − b − 1. Thus the area of the uncovered part is
ra + rb − r2 = −r2 + 222r = 1296 which is equivalent to (r − 6)(r − 216) = 0. Obviously a > r and b > r so we get
r = 6.
If we remove the uncovered area and set x = a − r, y = b − r, we obtain an x × y rectangle covered by (x − y) × (x − y)
squares (since x − y = a − b) and x + y = a + b − 2r = 210 = 2 · 3 · 5 · 7. Now x − y has to be a divisor of both x and y,
and so of x + y, too. Let us pick a divisor d | 210 and set x − y = d. Then using x + y = 210 we solve for x and y:

210 + d 210 − d
x= , y= .
2 2
Since the solutions must be positive integers (there is at least one square of side length a − b = x − y, therefore positive)
and as x − y > 6 (because of the maximality of the almost-cover), one can see that d yields a solution if only if it is an
even divisor of 210 and satisfies 6 < d < 210. Hence d is one of the numbers 10, 14, 30, 42, 70 the sum of which is 166.

Problem 54. Let P be a point inside a triangle ABC. Denote by A0 , B 0 , C 0 the intersections of AP , BP , CP with
BC, CA, AB, respectively. Assume that
A0 P = B 0 P = C 0 P = 3
and
AP + BP + CP = 25.
Find AP · BP · CP .
Result. 279
Solution. We shall denote the area of triangle XY Z by [XY Z].
Put
a = AP, b = BP, c = CP.
Then
[P BC] P A0 3
= =
[ABC] AA0 a+3
and likewise
[P CA] 3 [P AB] 3
= , = .
[ABC] b+3 [ABC] c+3
Furthermore, [P BC] + [P CA] + [P AB] = [ABC], so
3 3 3
+ + = 1,
a+3 b+3 c+3
which after clearing the denominators and rearranging yields

54 + 9(a + b + c) = abc.

The result follows by plugging in a + b + c = 25.

Problem 55. Fourteen points A1 , . . . , A14 were chosen on a circle c counter-clockwise in this order so that no three
distinct segments with endpoints among the given points meet in the interior of c. Christine drew all these segments,
but seeing that the picture had got too messy, she decided to erase all the sides and all the diagonals of heptagons
A1 A3 A5 A7 A9 A11 A13 and A2 A4 A6 A8 A10 A12 A14 . Into how many regions do the remaining segments divide the interior
of c?
Result. 295
Solution. Consider the segments being added to the configuration one after another: It is easy to see that when a
segment is added, the total number of regions increases by 1 + how many of already added segments does the new
segment intersect. Therefore the sought number of regions equals

1 + number of segments + number of intersections of segments.

Let us call points A1 , A3 , . . . , A13 odd and the remaining ones even. The existing segments are precisely those connecting
an odd point with an even point, hence there are 7 · 7 = 49 segments.
To compute the total number of intersections, note that the endpoints of intersecting segments have to lie on
the circle so that the odd ones are next to each other and the same for the even ones. On the other hand, every
such a quadruple of points gives rise to a unique intersection point, therefore we are to compute the number of these
quadruples. Assume that the points are ordered counter-clockwise and w.l.o.g. let A1 be the first odd point in the
quadruple; then dividing the points into seven pairs (A2 , A3 ), (A4 , A5 ), . . . , (A14 , A1 ), we observe that the remaining

18
three points have to be in distinct pairs. On the other hand, every choice of three of these pairs produces a desired
quadruple of points: Pick the odd point from the first pair and the even points from the other two pairs. Since there
are seven choices for the first odd point, we infer that there are
 
7
7· = 245
3

intersections in total. We conclude that the circle is divided into 1 + 49 + 245 = 295 regions.

Problem 56. Find the number of ordered 4-tuples (a, b, c, d) of positive integers satisfying

a + b + c + d = 505 and ab = cd.

Result. 800
Solution. Let us multiply the first equation by a and use the second one to get (a + c)(a + d) = 505a = 5 · 101 · a,
noting that both 5 and 101 are primes. As both of the brackets are bigger than a we need one of them to be equal
to 5k, and the other equal to 101l with kl = a. Assuming a + c = 5k and a + d = 101l for some fixed k, l satisfying
kl = a, we compute that c = k(5 − l), d = l(101 − k) and b = 505 − a − d − c = (101 − k)(5 − l). One can easily check
that the quadruple
(a, b, c, d) = (kl, (101 − k)(5 − l), k(5 − l), l(101 − k))
satisfies the condition ab = cd and thus is a valid solution of the given system for any l = 1, 2, 3, 4 and any
k = 1, 2, . . . , 100. All these 400 solutions are different since if two pairs (k1 , l1 ) and (k2 , l2 ) give the same solution
mentioned above we see that k1 l1 = k2 l2 and (5 − l1 )k1 = (5 − l2 )k2 implies k1 = k2 and l1 = l2 . Similarly for the case
a + c = 101l and a + d = 5k we obtain 400 pairwise different solutions

(a, b, c, d) = (kl, (101 − k)(5 − l), l(101 − k), k(5 − l))

any for l = 1, 2, 3, 4 and any k = 1, 2, . . . , 100. No solution from the second case coincides with a solution from the first
case since 5k = a + c = 101l does not hold for any k, l in our range. Therefore there are 400 + 400 = 800 solutions in
total.

19

Anda mungkin juga menyukai